Logical Reasoning
Logical Reasoning
What is Logic?
Logic is defined as the study of methods and principles used to distinguish good (correct) reasoning
from bad (incorrect) reasoning.
Since most questions in Logical reasoning are based on whether the student is capable of testing the
validity of an Argument, the first thing one has to clearly understand is the concept of the Argument .
For the purposes of understanding the concept of the Argument fully, it would help to get acquainted
with a few key related terms—
Elements of an Argument
Proposition: Propositions are basically the units of an Argument. A typical proposition has a
relationship spelled out between a subject and an object in the form of a simple (usually) sentence.
Example: All thieves are criminals
Here ‗thieves‘ is the subject and ‗criminals‘ the object of the predicate ‗are criminals.
Conclusion: The Conclusion of an Argument is the final proposition that is affirmed on the basis of
other propositions of the same Argument, and leading to the same. So basically the conclusion is
also a proposition, but one that is derived from another.
Premiss: The term Premiss is applied to that proposition that gives rise to the conclusion i.e. that
occurs first in sequence and has to be valid for the conclusion to be valid too. For the purposes of
explanation, frequently the terms ‗Proposition‘ and ‗Premise‘ are used interchangeably. So the
student should not get confused.
Inference: The term Inference is applied to that proposition that is a result of or derivation from the
Premiss. So, basically it means the same as Conclusion but is used usually in cases where there is
only one premiss from which it is derived.
When a conclusion is drawn from only one premiss, the inference is said to be immediate. When
there is more than one premiss involved, as in a syllogism, the inference is said to be mediate.
Argument : Once the above terms are clear, we can proceed to define an Argument—
An Argument is any group of propositions (two or more) of which one proposition, known as
conclusion/ inference, is claimed to follow from the other(s), known as Premiss(es), which is/are
regarded as providing support for the truth of former proposition.
In the above example of an Argument, the first part of the sentence i.e. ‗Wisdom is the principal thing‘
is the premiss and ‗get wisdom‘ is the inference/conclusion.
In this example, there are two propositions, as labeled. The conclusion (in this case, a better term
than ‘inference‘) is derived from a combination of both the propositions I and II. One important thing
to be noted here is that the sequence of the two propositions (or premisses, if you please), do not
matter. Proposition II can be stated first equally without affecting the conclusion in any way at all.
Example 3: Neither a borrower, nor a lender be; for loan oft loses both itself and friend.
In this example, it is obvious that, unlike 1, the second part of the sentence (For loan…) is the
premiss. The difference is made palpable by the use of the connective ‗for‘ here, instead of
‗therefore‘, as in the first case.
Types of Argument
1. Deductive Logic: A deductive argument is one whose conclusion is claimed to follow from its
premises with absolute necessity or certainty, this certainty not being a matter of degree and
not depending in any way on anything else. Therefore a deductive argument has to be either
valid or invalid. There is no grey area in between.
From the above example, another fact that comes out is that deductive arguments are frequently in
the form of Syllogisms. A Syllogism is any deductive argument where a conclusion is inferred from
two premisses.
A standard deductive argument with a mediate conclusion has two premisses and a derived
conclusion. So, in lay terms, it would be appropriate to say that an argument completely depends
upon its propositions. So, our first step will be to study the various kinds of propositions and what
each entails. For the purposes of the Management Entrance Examinations, a glance at the three
major types is sufficient—
TYPES OF PROPOSITIONS
1. CATEGORICAL PROPOSITION - Here the relationship between the subjective and the
predicative term is categorical i.e. the predicate either affirms or denies. For Example:
Preeti is a girl.
All guns are dangerous.
Some trees are bushes.
Some snakes are not poisonous.
There are four types of Categorical Propositions. Two of them describe the subject (hereafter
called ‗X‘) wholly, and are therefore called Universal Propositions. The remaining two describe
‗x‘ partially, and are therefore called Particular Propositions.
The most important thing here is that each proposition can and will be represented by Euler‘s
Circles (or Venn diagrams as they are more commonly known) and that will help us get
immediate inferences from each proposition. It will also help us in deciding which possible
inferences are necessarily true in all cases, which are necessarily false and which are
uncertain. The propositions are:
Note that we will be using ‗X‘ and ‗Y‘ to denote the subjective and predicative terms of the
propositions to minimize confusion.
(i) The Universal Affirmative: It states that every member of the first class is also a
member of the second class. This proposition takes the form All Xs are Ys. The
possible diagrams as shown by the Euler‘s Circles for this proposition are :
1 2
Immediate Inferences—
(ii) Some Ys are not Xs – Uncertain. This is because though diagram 1 shows it is
possible, diagram 2 shows another equally probable case fulfilling the Universal
Affirmative proposition. In this case, X and Y are co-incidental circles. We see that the
first inference still stands valid since the fact is that is that some Ys have to be Xs in all
cases without exception. But the second inference is uncertain since it is true in one
case and false in the second case, as shown by the two diagrams. Thus ‗Some Ys are
not Xs‘ is a possibility, but not a certainty.
(ii) The Universal Negative: It states that no member of the first class is a member of the
second class. This proposition takes the form— No X is a Y. The Euler‘s circle diagram
for this proposition would be two mutually exclusive circles thus:
X Y
Immediate Inference-
(iii) The Particular Affirmative: It states that at least one member of the term designated by
the class ‗X‘ is also a member of the class designated by the term ‗Y‘. This proposition
takes the form Some Xs are Ys. The possible diagrams as shown by the Euler‘s Circles
for this proposition are :
X Y
Y X
1 2
Immediate Inferences-
(ii) Some Xs are not Ys – True also, since according to modern Boolean Logic, when we
include a specific part of one class into another, we are by definition, excluding the rest
(although, as we will see later, the vice versa does not hold true).
(iv) The Particular Negative: It states that at least one member of the class designated by
the term ‗X‘ is excluded from the whole of the class designated by the term ‗Y‘. This
proposition takes the form Some Xs are not Ys. There is no specific diagram for the
Particular Negative. The Euler‘s circle diagram for this proposition would be two mutually
exclusive circles thus:
Y X Y X X
1 2 3
Immediate Inferences –
There are no immediate inferences for a Particular Negative. The reasons are given below:
Unlike the last Particular Affirmative proposition, the exclusion of a part of X from Y does not in
any way give us any information about the rest of the Xs. So we cannot ever say anything for
certain about the rest of the Xs. Therefore, for a Particular Negative proposition, there are no
immediate inferences OR, in other words, all immediate inferences are uncertain. This is amply
borne out by the three possible diagrams for the Particular Negative shown below. The shaded
portion in each is that part of X that is not Y.
In each of the above cases, the diagrams fulfill the condition that there are some Xs that are
not Ys. In Diagram 1, even the rest of the Xs are not Ys. Equally possible is Diagram 2, where
the rest of the Xs are Ys, as in Diagram 3, where all Ys are Xs. Thus, no immediate inference,
because none of the three are certainties.
There are some propositions that structurally do not seen to conform to the four types
described above and indeed, are the cause of a lot of confusion among students. They are
delineated below with explanations that prove that they are just variants of the four basic
types.
1. All Xs are not Ys. This means the same as Some Xs are not Ys .
Although it seems difficult to believe at first glance, This is a quirk of the English Language. A
close inspection of the proposition will show that its function is the same as those of the
Particular Negative i.e. both describe the subject ‗X‘ in part (and not universally—in which
case we would have said ‗No X is a Y‘ instead. There is a major difference between ‗No X is a
Y‘ and ‗All Xs are not Ys‘).
An example from real life should quell all doubts about the usage of ‗All‘ here—
Now, obviously, this does not mean that no graduates are MBAs. What it does is, like the
Particular Negative, denote a part of graduates who are not MBAs.
7 Pankaj Gandhi’s Academy/Logical Reasoning
Hence, to avoid confusion, students are advised to change the sentence type All Xs are not
Ys to the standard Particular Negative type Some Xs are not Ys whenever they come across
it.
2. Only Xs are Ys. This means the same as All Ys are Xs.
We find that what it means is that all MBAs are graduates. This is also borne out by drawing a
diagram for the same:
GRAD
MBA
Therefore students are advised to change the premise to a Universal Affirmative to facilitate
problen solving.
X X
X X X X
X X X X
X X
We find that there is no way that we can know which of the Xs is the student who is not
present. Therefore, due to the inability to pinpoint the particular student, we have to consider
this proposition as a Particular Affirmative i.e.
Some students are present.
In contrast, in (ii), we know exactly which student (i.e. Bahar) is present. Therefore here it
would be inappropriate to say that some students are present. In this case, after isolating that
particular student, the statement is turned into a Universal Affirmative i.e.
All students are present.
Example:
Example:
1. CATEGORICAL SYLLOGISM
A categorical syllogism is a deductive argument consisting of three categorical propositions
that together contain exactly three terms, each of which occurs in exactly two of the constituent
propositions.
The rule which follows from the above definition may be loosely termed as the Common
Factor Rule and it says that the conclusion will not have the common factor of the two
premisses. For example in a typical syllogistic argument of the given type:
P1 - All Xs are Ys
P2 - All Ys are Zs
C - All Xs are Zs
we find that the conclusion i.e. all Xs are Zs does not have that common factor of the two
premisses, which is the factor ‗Y‘.
Even if the sequence or structure of the subject and object in the premisses is changed, it does
not make any difference to the outcome.
MBA
Note: There is another conclusion i.e. Some Students are MBAs following the Immediate
Inference rule for the Universal Affirmative as given earlier.
Remember that ‗If X, then Y.‘ tells us only two things: (1) If X is true, then Y is true as well. (2)
If Y is false, then B is false as well (contra-positive). All other inferences are either false or
uncertain.
Affirming the Conclusion Fallacy:
Premiss : ‗If X, then Y.‘
Wrong Inference : If Y, then X.
If we know the conclusion is true, the if-then statement tells us nothing about the premise.
(a) If Azharuddin does not bat well, India will not win.
(b) If Azharuddin bats well, India will not win.
(c) If India wins, Azharuddin has batted well.
(d) If India did not win, Azharuddin did not bat well.
(e) None of these
The answer is (d).
Sometimes, the questions are rather more complicated, involving a number of similar
propositions. The student has to take care to identify the crucial one which relates to the
inference given or asked.
Example: If Simi has a talk show, then Priya will also have a talk show. If Priya does not have
a talk show, Oprah will not have a talk show. If Donahue has a talk show, then Leno also has a
talk shoe. Which of the following is applicable if Oprah has a talk show?
Here we see that beyond the obvious confusion, there is really very little problem in the
question. There are three If-clause propositions, of which the third is redundant as far as the
question is concerned (Donahue and Leno). It has just been put there to confuse the students.
Pankaj Gandhi’s Academy/Logical Reasoning 10
According to the first condition, if Priya does not have a talk show, then Simi does not have a
talk show because it is the obvious conclusion according to the formula. According to the
second condition, if Oprah has a talk show, Priya has a talk show .
The second conclusion is option (b) of the answer choices. The answer, therefore, is (b).
It is not necessary that X and Y be both positive or both negative. It is perfectly possible to
have a proposition like ‘If I read a lot, I will not pass my exams’. In this case the conclusion
would be ‘If I have passed my exams, I have not read a lot’.
Transitive Property
Premiss : If X, then Y.
Premiss : If Y, then Z.
Inference : So if X, then Z.
These arguments are rarely difficult, provided you step back and take a bird's-eye view. It may
be helpful to view this structure as an inequality in mathematics.
Notice that the conclusion in the transitive property is also an if-then statement. So we don't
know that Z is true unless we know that X is true. However, if we add the premise "X is true" to
the diagram, then we can conclude that Z is true:
Premiss : If X, then Y.
Premiss : If Y, then Z.
Inference : So if not Z, then not X.
Example: "If you work hard, you will be successful in America. If you are successful in
America, you can lead a life of leisure‖
Usually, arguments involve an if-then statement. Unfortunately, the if-then thought is often
embedded in other equivalent structures.
Example 1 : ―If Iqbal goes to the circus, Radha and Mary will also go.‖
Inference : If Radha or Mary do not go to the circus, Iqbal will not go.
Example 2 : ―If Iqbal goes to the circus, Radha or Mary will also go.‖
Inference : If Radha and Mary do not go to the circus, Iqbal will not go.
If you have taken a course in logic, you are probably familiar with these formulae. Their validity
is intuitively clear: The conjunction X&Y is false when either, or both, of its parts are false.
And the disjunction X or Y is false only when both A and B are false.
Given this statement, we know that if Danielle is accepted to graduate school, then she must
have done well on the GMAT.
Students often wrongly interpret this statement to mean. "If Danielle does well on the GMAT,
then she will be accepted to graduate school." There is no such guarantee. The only
guarantee is that if she does not do well on the GMAT, then she will not be accepted to
graduate school.
―X and Y cannot both " is logically equivalent to "if X, then not Y" or "if Y, then not X"
At first glance, this sentence does not appear to contain an if-then statement. But it essentially
says: "If John goes to the party, then Ken does not".
iv. ―X unless Y‖
Example : "Biff is at the beach unless it is raining."
If we symbolize "Biff is at the beach" as X, and "it is raining" as Y, we know that if it is not
raining, then Biff is at the beach.
3. THE DILEMMA
The disjunctive proposition contains two component propositions. It will take the form—
Either X or Y
1. If not X, then Y
and
2. If not Y, then X.
Note that like the If-clause X and Y could either be positive or negative sentences i.e. the
sentence ‗Either he does not run or he sleeps‘ will also fall under the format of Either X or y.
The only difference is that in this case ‗X‘ stands for ‗he does not run‘.
In this case the conclusions could be either ‗If he runs, thnm he sleeps‘ or ‗If he does not sleep,
he does not run‘.
Therefore, If one of the disjuncts is the proposition and if the other proposition is the
contradiction of one of the two disjuncts, then the other disjunct is true. The important thing is
that for a disjunct to be true, the other premiss has to be a negative i.e. contradictory.
An invalid dilemma:
Example:
Disjuncts/ Premises : Either Rasputin ruled the country or he was executed.
Conclusion : Rasputin ruled the country. Therefore he was not executed.
This kind of conclusion is incorrect because the fact that Rasputin ruled has nothing to do with
the possibility of his execution.
1. CONCLUSIONS
Most argument questions hinge, either directly or indirectly, on determining the conclusion
of the argument. The conclusion is the main idea of the argument. It is what the writer tries to
persuade the reader to believe. Most often the conclusion comes at the end of the argument.
The writer organizes the facts and his opinions so that they build up to the conclusion.
Sometimes, however, the conclusion will come at the beginning of an argument, rarely does it
come in the middle, and occasionally, for rhetorical effect, the conclusion is not even stated.
Example:
"The police are the armed guardians of the social order. The blacks are the chief domestic
victims of the American social order. A conflict of interest exists, therefore, between the blacks
and the police.--Eldridge Cleaver, Soul on Ice"
Here the first two sentences anticipate or set up the conclusion. By changing the grammar
slightly, the conclusion can be placed at the beginning of the argument and still sound natural:
"A conflict of interest exists between the blacks and the police because the police are the
armed guardians of the social order and the blacks are the chief domestic victims of the
American social order."
"The police are the armed guardians of the social order. So a conflict of interest exists between
the blacks and the police because the blacks are the chief domestic victims of the American
social order."
It is generally awkward to place the conclusion in the middle of the argument because then it
cannot be fully anticipated by what comes before nor fully explained by what comes after. On
the rare occasion when a conclusion comes in the middle of an argument, most often either the
material that comes after it or the material that comes before it is not essential.
In Summary: To find the conclusion, check the last sentence of the argument. If that is not the
conclusion, check the first sentence. Rarely does the conclusion come in the middle of an
argument.
As mentioned before, the conclusion usually comes at the end of an argument, sometimes at
the beginning, and rarely in the middle. Writers use certain words to indicate that the
conclusion
is about to be stated. Following is a list of the most common conclusion indicators-
hence , so, thus, follows that, conclude that, as a result, therefore, accordingly , consequently,
shows that, implies, means that
Types of conclusions
1. The stated conclusion: Most often the conclusion of an argument is put in the form of a
statement. Sometimes, however, the conclusion is given as a command or obligation.
2. The rhetorical conclusion: The conclusion can even be put in the form of a question. This
rhetorical technique is quite effective in convincing people that a certain position is correct.
We are more likely to believe something if we feel that we concluded it on our own, or at
least if we feel that we were not told to believe it. A conclusion put in question form can
have this result.
Example: "The Nanuuts believe that they should not take from Nature anything She cannot
replenish during their lifetime. This assures that future generations can enjoy the same
riches of Nature that they have. At the current rate of destruction, the rain forests will
disappear during our lifetime. Do we have an obligation to future generations to prevent this
result?"
Here the author trusts that the power of her argument will persuade the reader to answer
the
question affirmatively.
3. The un-stated conclusion: Taking the rhetorical technique one step further, the writer may
build up to the conclusion but leave it unstated. This allows the reader to make up his own
mind. If the build-up is done skillfully, the reader will be more likely to agree with the author,
without feeling manipulated.
Example: "He who is without sin should cast the first stone. There is no one here who does
not have a skeleton in his closet."
The unstated but obvious conclusion here is that none of the people has the right to cast
the
first stone.
Quantifiers
When determining the conclusion's scope be careful not to read any more or less into it than
the author states. GMAT writers often create wrong answer-choices by slightly overstating or
understating the author's claim. Certain words limit the scope of a statement. These words are
called quantifiers--pay close attention to them. Following is a list of the most important
quantifiers-
all, some, only, never, probably, except, most, could, always, must, likely, many, no,
everywhere, alone
15 Pankaj Gandhi’s Academy/Logical Reasoning
Example: "Whether the world is Euclidean or non-Euclidean is still an open question.
However, if a star's position is predicted based on non-Euclidean geometry, then when a
telescope is pointed to where the star should be it will be there. Whereas, if the star's position
is predicted based on Euclidean geometry, then when a telescope is pointed to where the star
should be it won't be there. This strongly indicates that the world is non-Euclidean."
Which one of the following best expresses the main idea of the passage?
Choice (A) understates the main idea. Although the opening to the passage states that we
don't know whether the world is non-Euclidean, the author goes on to give evidence that it is
non-Euclidean. Choice (C) overstates the main idea. The author doesn't say that the world is
non-Euclidean, just that evidence strongly indicates that it is. In choice (B), the word "probably"
properly limits the scope of the main idea, namely, that the world is probably non-Euclidean,
but we can't yet state so definitively. The answer is (B).
2. PREMISES
Once you've found the conclusion, most often everything else in the argument will be either
premises or "noise". The premises provide evidence for the conclusion; they form the
foundation or infrastructure upon which the conclusion depends. To determine whether a
statement is a premise, ask yourself whether it supports the conclusion. If so, it's a premise.
Premise Indicators Earlier we saw that writers use certain words to flag conclusions; likewise
writers use certain words to flag premises. Following is a partial list of the most common
premise indicators-
because, since, if, as, suppose, assume, for, is evident that, in that, ,owing to, inasmuch as,
may be derived from
Example: "Since the incumbent's views are out of step with public opinion, he probably will not
be reelected."
Here "since" is used to flag the premise that the incumbent's positions are unpopular.
Suppressed Premises
Most arguments depend on one or more unstated premises. Sometimes this indicates a
weakness in the argument, an oversight by the writer. More often, however, certain premises
are
left tacit because they are too numerous, or the writer assumes that his audience is aware of
the
assumptions, or he wants the audience to fill in the premise themselves and therefore be more
likely to believe the conclusion.
Example:
Conclusion: "I knew he did it."
Premise: "Only a guilty person would accept immunity from prosecution."
The suppressed premise is that he did, in fact, accept immunity. The speaker assumes that his
audience is aware of this fact or at least is willing to believe it, so to state it would be
redundant and ponderous. If the unstated premise were false (that is, he did not accept
immunity), the argument would not technically be a lie; but it would be very deceptive. The
unscrupulous writer may use this ploy if he thinks that he can get away with it. That is, his
A common question on the GMAT asks you to find the suppressed premise of an argument.
Finding the suppressed premise, or assumption, of an argument can be difficult. However, on
the GMAT you have an advantage--the suppressed premise is listed as one of the five answer-
choices. To test whether an answer-choice is a suppressed premise, ask yourself whether it
would make the argument more plausible. If so, then it is very likely a suppressed premise.
Example: "American attitudes tend to be rather insular, but there is much we can learn from
other countries. In Japan, for example, workers set aside some time each day to exercise, and
many corporations provide elaborate exercise facilities for their employees. Few American
corporations have such exercise programs. Studies have shown that the Japanese worker is
more productive than the American worker. Thus it must be concluded that the productivity of
American workers will lag behind their Japanese counterparts, until mandatory exercise
programs are introduced."
The conclusion of the argument is valid if which one of the following is assumed?
A. Even if exercise programs do not increase productivity, they will improve the American
worker's health.
B. The productivity of all workers can be increased by exercise.
C. Exercise is an essential factor in the Japanese worker's superior productivity.
D. American workers can adapt to the longer Japanese work week.
E. American corporations don't have the funds to build elaborate exercise facilities.
The unstated essence of the argument is that exercise is an integral part of productivity and
that Japanese workers are more productive than American workers because they exercise
more. The answer is (C).
3. COUNTER-PREMISES
When presenting a position, you obviously don't want to argue against yourself. However, it is
often effective to concede certain minor points that weaken your argument. This shows that
you are open-minded and that your ideas are well considered. It also disarms potential
arguments against your position. For instance, in arguing for a strong, aggressive police
department, you may concede that in the past the police have at times acted too aggressively.
Of course, you will then need to state more convincing reasons to support your position.
Example: "I submit that the strikers should accept the management's offer. Admittedly, it is
less than what was demanded. But it does resolve the main grievance--inadequate health
care. Furthermore, an independent study shows that a wage increase greater than 5% would
leave the company unable to compete against Japan and Germany, forcing it into bankruptcy."
The conclusion, "the strikers should accept the management's offer", is stated in the first
sentence. Then "Admittedly" introduces a concession; namely, that the offer was less than
what was demanded. This weakens the speaker's case, but it addresses a potential criticism of
his position before it can be made. The last two sentences of the argument present more
compelling reasons to accept the offer and form the gist of the argument.
Counter-Premise Indicators
As you may have anticipated, the GMAT writers sometimes use counter-premises to bait wrong
We will classify the three major types of inductive reasoning--generalization, analogy, and
causal--and their associated fallacies.
1. GENERALIZATION
Generalization and analogy, which we consider in the next section, are the main tools by which
we accumulate knowledge and analyze our world. Many people define generalization as
"inductive reasoning". In colloquial speech, the phrase "to generalize" carries a negative
connotation. To argue by generalization, however, is neither inherently good nor bad. The
relative validity of a generalization depends on both the context of the argument and the
likelihood that its conclusion is true. Polling organizations make predictions by generalizing
information from a small sample of the population, which hopefully represents the general
population. The soundness of their predictions (arguments) depends on how representative
the sample is and on its size. Clearly, the less comprehensive a conclusion is the more likely it
is to be true.
Example: "During the late seventies when Japan was rapidly expanding its share of the
American auto market, GM surveyed owners of GM cars and asked them whether they would
be more willing to buy a large, powerful car or a small, economical car. Seventy percent of
those who responded said that they would prefer a large car. On the basis of this survey, GM
decided to continue building large cars. Yet during the '80s, GM lost even more of the market
to the Japanese."
Which one of the following, if it were determined to be true, would best explain this
discrepancy?
The argument generalizes from the survey to the general car-buying population, so the
reliability of the projection depends on how representative the sample is. At first glance, choice
(A) seems rather good, because 10 percent does not seem large enough. However, political
opinion polls are typically based on only .001 percent of the population. More importantly, we
don't know what percentage of GM car owners received the survey. Choice (B) simply states
that Ford made the same mistake that GM did. Choice (C) is irrelevant. Choice (D), rather than
explaining the discrepancy, gives even more reason for GM to continue making large cars.
Finally, choice (E) points out that part of the survey did not represent the entire public, so (E) is
the answer.
2. ANALOGY
To argue by analogy is to claim that because two things are similar in some respects, they will
be similar in others. Medical experimentation on animals is predicated on such reasoning. The
argument goes like this: the metabolism of pigs, for example, is similar to that of humans, and
high doses of saccharine cause cancer in pigs. Therefore, high doses of saccharine probably
Pankaj Gandhi’s Academy/Logical Reasoning 18
cause cancer in humans.
Clearly, the greater the similarity between the two things being compared the stronger the
argument will be. Also the less ambitious the conclusion the stronger the argument will be. The
argument above would be strengthened by changing "probably" to "may". It can be weakened
by
pointing out the dissimilarities between pigs and people.
Example: Just as the fishing line becomes too taut, so too the trials and tribulations of life in
the
city can become so stressful that one's mind can snap.
Which one of the following most closely parallels the reasoning used in the argument above?
A. Just as the bow may be drawn too taut, so too may one's life be wasted by pursuing self-
gratification.
B. Just as a gambler's fortunes change unpredictably, so too do one's career opportunities
come unexpectedly.
C. Just as a plant can be killed by over watering it, so too can drinking too much water lead to
lethargy.
D. Just as the engine may race too quickly, so too may life in the fast lane lead to an early
death.
E. Just as an actor may become stressed before a performance, so too may dwelling on the
negative cause depression.
The argument compares the tautness in a fishing line to the stress of city life; it then concludes
that the mind can snap just as the fishing line can. So we are looking for an answer-choice that
compares two things and draws a conclusion based on their similarity. Notice that we are
looking for an argument that uses similar reasoning, but not necessarily similar concepts. In
fact, an answer-choice that mentions either tautness or stress will probably be a same-
language trap.
Choice (A) uses the same-language trap--notice "too taut". The analogy between a taut bow
and self-gratification is weak, if existent. Choice (B) offers a good analogy but no conclusion.
Choice (C) offers both a good analogy and a conclusion; however, the conclusion, "leads to
lethargy", understates the scope of what the analogy implies. Choice (D) offers a strong
analogy and a conclusion with the same scope found in the original: "the engine blows, the
person dies"; "the line snaps, the mind snaps". This is probably the best answer, but still we
should check every choice. The last choice, (E), uses language from the original, "stressful", to
make its weak analogy more tempting. The best answer, therefore, is (D).
3. CAUSAL REASONING
Of the three types of inductive reasoning we will discuss, causal reasoning is both the weakest
and the most prone to fallacy. Nevertheless, it is a useful and common method of thought.
To argue by causation is to claim that one thing causes another. A causal argument can be
either weak or strong depending on the context. For example, to claim that you won the lottery
because you saw a shooting star the night before is clearly fallacious. However, most people
believe that smoking causes cancer because cancer often strikes those with a history of
cigarette use. Although the connection between smoking and cancer is virtually certain, as with
all inductive arguments it can never be 100 percent certain. Cigarette companies have claimed
that there may be a genetic predisposition in some people to both develop cancer and crave
nicotine. Although this claim is highly improbable, it is conceivable.
1. Confusing Correlation with Causation: To claim that A caused B merely because A occurred
immediately before B is clearly questionable. It may be only coincidental that they occurred
19 Pankaj Gandhi’s Academy/Logical Reasoning
together, or something else may have caused them to occur together. For example, the fact
that insomnia and lack of appetite often occur together does not mean that one necessarily
causes the other. They may both be symptoms of an underlying condition.
2. Confusing Necessary Conditions with Sufficient Conditions: A is necessary for B means "B
cannot occur without A". A is sufficient for B means "A causes B to occur, but B can still
occur without A". For example, a small tax base is sufficient to cause a budget deficit, but
excessive spending can cause a deficit even with a large tax base. A common fallacy is to
assume that a necessary condition is sufficient to cause a situation. For example, to win a
modern war it is necessary to have modern, high-tech equipment, but it is not sufficient, as
Iraq discovered in the Persian Gulf War.
1 CONTRADICTION
A Contradiction is committed when two opposing statements are simultaneously asserted. For
example, saying "it is raining and it is not raining" is a contradiction. Typically, however, the
arguer obscures the contradiction to the point that the argument can be quite compelling.
Take, for instance, the following argument:
Example: "We cannot know anything, because we intuitively realize that our thoughts are
unreliable."
This argument has an air of reasonableness to it. But "intuitively realize" means "to know".
Thus the arguer is in essence saying that we know that we don't know anything. This is self-
contradictory.
2. EQUIVOCATION
Equivocation is the use of a word in more than one sense during an argument. This technique
is
often used by politicians to leave themselves an "out". If someone objects to a particular
statement, the politician can simply claim the other meaning.
Example: "Individual rights must be championed by the government. It is right for one to
believe in God. So government should promote the belief in God."
In this argument, right is used ambiguously. In the phrase "individual rights" it is used in the
sense of a privilege, whereas in the second sentence right is used to mean proper or moral.
The
questionable conclusion is possible only if the arguer is allowed to play with the meaning of the
critical word right.
3. CIRCULAR REASONING
Circular reasoning involves assuming as a premise that which you are trying to prove.
Intuitively,
it may seem that no one would fall for such an argument. However, the conclusion may appear
to state something additional, or the argument may be so long that the reader may forget that
the conclusion was stated as a premise.
Example: "The death penalty is appropriate for traitors because it is right to execute those
who
betray their own country and thereby risk the lives of millions."
This argument is circular because "right" means essentially the same thing as "appropriate". In
effect, the writer is saying that the death penalty is appropriate because it is appropriate.
Pankaj Gandhi’s Academy/Logical Reasoning 20
4. SHIFTING THE BURDEN OF PROOF
It is incumbent on the writer to provide evidence or support for her position. To imply that a
position is true merely because no one has disproved it is to shift the burden of proof to others.
Example: "Since no one has been able to prove God's existence, there must not be a God."
There are two major weaknesses in this argument. First, the fact that God's existence has yet
to
be proven does not preclude any future proof of existence. Second, if there is a God, one
would
expect that his existence is independent of any proof by man.
5. UNWARRANTED ASSUMPTIONS
The fallacy of unwarranted assumption is committed when the conclusion of an argument is
based on a premise (implicit or explicit) that is false or unwarranted. An assumption is
unwarranted when it is false--these premises are usually suppressed or vaguely written. An
assumption is also unwarranted when it is true but does not apply in the given context--these
premises are usually explicit.
The arguer offers two options: either restrict freedom of speech, or lose the country. He hopes
the reader will assume that these are the only options available. This is unwarranted for. He
does not state how the so-called "subversive elements" would destroy the country, nor for
that matter, why they would want to destroy it. There may be a third option that the author did
not mention; namely, that society may be able to tolerate the "subversives" and it may even be
improved by the diversity of opinion they offer. The answer is (C).
6. APPEAL TO AUTHORITY
To appeal to authority is to cite an expert's opinion as support for one's own opinion. This
method of thought is not necessarily fallacious. Clearly, the reasonableness of the argument
depends on the "expertise" of the person being cited and whether she is an expert in a field
relevant to the argument. Appealing to a doctor's authority on a medical issue, for example,
would be reasonable; but if the issue is about dermatology and the doctor is an orthopaedist,
then the argument would be questionable.
7. PERSONAL ATTACK
In a personal attack (ad hominem), a person's character is challenged instead of his opinions.
Example: Politician: "How can i trust my opponent to be true to the voters? He isn't true to his
wife!"
This argument is weak because it attacks the opponent's character, not his positions. Some
people may consider fidelity a prerequisite for public office. History, however, shows no
correlation between fidelity and great political leadership.
For Example :
Prakash Mittal, owner of Venus Stationers , wants to open a store in Uran. A new engineering college
will open in Uran next year, and Prakash anticipates that the students of this college will provide a
good market for the stationery items. Prakash wants to find a store that is large enough so as to cater
to all stationary item needs, such as drawing boards, drafters, tee-squares etc. He also wants a store
with as low a monthly rent as possible. Finally, he also wants a store that is not too far from the
college campus. Prakash has located three stores for rent in the area. Their addresses are 10
Bhosale Heights, 14 Patil Plaza, and 17-18 Padale Palace. The monthly rentals are Rs. 3000, Rs.
2500 and Rs. 2000 respectively. The Bhosale Heights location is about 100 metres away from the
college campus, the Patil Plaza location is half a mile away, and the Padale Palace location is about
two miles away from the campus. The Bhosale Heights and Patil Plaza locations are about the same
size. The Padale Palace location is larger than the other two and it also includes a basement which
can be used for storage, thus making more space available in the main store.
Renting a store site in Uran is one of the goals of Prakash. therefore, the best classification for the
first item would be A, as it is a major objective.
Prakash wants a store at the lowest possible rental; therefore the rent he has to pay at each location
is an influencing factor in reaching the decision Thus, the classification for the second item would be
B, as it is a major factor.
The additional space in the basement of the Padale Palace store is a consideration for comparing the
options, but it is only a part of the space offered by that store. thus though it affects the decision, it is
not the only criterion upon which the decision would be based. As such the classification for the third
item would be C, it being a minor factor.
One strategy to tackle such problems is to follow the directions summarised below.
B D
1. IMMEDIATE INFERENCE
In this question type a statement is given and it is followed by an inference. One has to either—
Assume the statement to be true and work out if the inference is (a) True, (b) False or (c) Uncertain
OR
Assume the statement to be false and decide if the inference is (a) True, (b) False or (c) Uncertain.
Now, in the above question, if the statement is that the statement is true, then obviously the inference
is true (the Universal Affirmative rule) and the answer is (a).
But if the question says that the statement is false, then our first job is to make it true. In logic,
whatever is not hundred percent false is true. Therefore to convert the false statement to a true one,
we have to make only one exception to the statement (and resist the impulse to make it into an
opposite statement). The converted true statement now becomes:
Now the answer has to be (c) since all inferences from a Particular Negative (Some/All…not)
statements are uncertain.
2. VALIDITY OF ARGUMENT
This can cover two question types, both essentially asking the examinee to assess the validity of the
argument.
E.g. 1– In the following question, two statements are followed by two inferences. In the answer
mark
(a) if only Inference (I) follows, (b) if only Inference (II) follows,
(c) if both (I) and (II) follow, (d) if neither (I) nor (II) follows.
The answer, after checking with the rules, and Euler‘s circles, will be (a).
E.g. 2– Six propositions are given, of which three form a logically compact argument. You have
to identify that set of three from the given answer options. The basic rules of logic and the
Euler‘s Circles will give the answer.
3. CATEGORICAL SYLLOGISM
This question type is in the form of a sentence with two parts consisting of the conclusion and one of
the two premisses. The student is expected to identify the missing second premise from the given
answer choices.
The first step towards solving this type is identifying the premiss and the conclusion. This becomes
easy with the realization that temporally, the part that occurs first has to be the premiss i.e. He was a
murderer. (Another fast way of identifying the premiss and the conclusion would be to check the
connectives between the two parts of the sentence. E.g. if the connective is ‘because‘, the second
part of the sentence becomes the premise i.e. the cause of the conclusion. If the connective is ‗so‘ or
‗therefore‘, then the first part of the sentence is the premiss. Therefore in a sentence like ‗Women are
cruel, therefore they are dangerous‘, the premiss is ‗Women are cruel‘)
Now applying the rules it is clear that the missing premise has to be a Universal Affirmative; therefore
the answer has to be either (a) or (b). Now the Euler‘s circles will confirm that the answer is (b).
Sometimes, the questions are rather more complicated, involving a number of similar propositions.
The student has to take care to identify the crucial one which relates to the inference given or asked.
E.g. If Simi has a talk show, then Priya will also have a talk show. If Priya does not have a talk show,
Oprah will not have a talk show. If Donahue has a talk show, then Jay also has a talk shoe.
Which of the following is applicable if Oprah has a talk show?
If Priya does not have a talk show, then Simi does not have a talk show
If Oprah has a talk show, Priya has a talk show (because even if the premiss is in the form
if not X, then not Y,
The second conclusion is option (b) of the answer choices. The answer, therefore, is (b).
NOTE—It is not necessary that X and Y be both positive or both negative. It is perfectly possible to
have a proposition like ‗If I read a lot, I will not pass my exams’. In this case the conclusion
would be ‗If I have passed my exams, I have not read a lot’. Thus various combinations of
positive and negative sentences may be given to the students to confuse them, especially as
the statements seem to contradict their real-life experiences.
It is reiterated that general knowledge and real life has no bearing on Deductive Logical
Reasoning.
The disjunctive proposition contains two component propositions. It will take the form—
Either X or Y
2. If not Y, then X.
Note that like the If-clause X and Y could either be positive or negative sentences i.e. the sentence
‗Either he does not run or he sleeps‘ will also fall under the format of Either X or y. The only
difference is that in this case ‗X‘ stands for ‗he does not run’. In this case the conclusions could be
either ‗If he runs, them he sleeps’ or ‗If he does not sleep, he does not run’.
Therefore, if one of the disjuncts is the proposition and if the other proposition is the contradiction of
one of the two disjuncts, then the other disjunct is true. The important thing is that for a disjunct to be
true, the other premiss has to be a negative i.e. contradictory. Let us take an example of a valid
argument:
E.g. Disjuncts/ Proposition : Either Rasputin ruled the country or he was executed.
Conclusion : Rasputin did not rule the country.
Therefore Rasputin was executed.
To clarify the point about one premiss necessarily being a contradiction to the other to get a
conclusion, let us study an example of an invalid dilemma:
This kind of conclusion is incorrect because the fact that Rasputin ruled has nothing to do with the
possibility of his execution.
6. DATA SUFFICIENCY
In this question type, a conclusion is given first and two possible data statements follow it. One has to
study the data and the conclusion and mark
(a) if data statement 1 alone can bring us to the conclusion.
(b) if data statement 2 alone can bring us to the conclusion.
(c) if both statements 1and 2 together can get us the conclusion.
(d) if both statements takes separately or together cannot bring us the conclusion.
E.g Conclusion : The office needed Styrofoam cups to serve coffee to employees.
Data : 1. The office gave its employees three cups of coffee a day.
2. Everybody drank coffee in Styrofoam cups.
Now if we read the data statements carefully, we realise that neither of the two alone is capable of
reaching the conclusion, but that taken together, the statements do give us the conclusion that the
office needed Styrofoam cups to serve coffee to its employees. Therefore the correct answer is (c).
This type will have one or two statements followed by one or more inferences, which the student has
to validate.
This is one of the most common type of Inductive Reasoning questions, and in many ways, the most
important type. A basic comprehension of English is sometimes not enough as the subtle nuances in
the statements sometimes decide the fate of the answer a student selects. Thus, the first step is to
read the statement(s) very carefully, noting if there are any vague words, which potentially could
render a factual sounding statement uncertain. Then one should take care to note if the implication
statement(s) are actually implied by the problem statement beyond reasonable doubt or not. Only
then will the statements be implied.
Statement: Everyone thinks that I murdered Caesar and that I did not love him. But it is not that I
loved Caesar less, but that I loved Rome more.
If we read the statements very carefully, we find that the first implication is only alleged but not
confirmed. We cannot say for certain that the speaker killed Caesar in actuality, since people only
think, they do not know. The second implication is true because what the speaker means is that he
loved Rome more than he loved Caesar, but that he loved Caesar nonetheless.
The answer is (b).
This is another common question type. Here there is a question followed by two answers starting with
‗Yes‘ and ‗No‘ respectively. We have to decide whether the answers are forceful or not.
The first and most important thing here is that we are not supposed to consider an answer forceful at
the cost of the other i.e. we are not supposed to compare the two answers. Each answer is to be
considered to be an independent entity. Therefore it follows that there must be a few objective tools
to test forcefulness. There are two rules and both have to be applicable simultaneously.
1. The answer must address all the points specifically raised by the question e.g. if the question
mentions a country specifically, then the answer must mention or at least refer to it also
without any ambiguity as to the subject of the reference.
Answer 1 looks strong enough but it does not fulfil any of the conditions laid out above. An example
of a forceful answer would be—
Yes, because mechanisation will lead to increased agricultural production and India desperately
needs more food grains.
The second answer fulfils the second condition i.e. a supportive argument, but it is still not forceful
because of the ambiguity caused by the word ‗we‘. It is necessary to assume that ‗we‘ does not stand
automatically for Indians. It could stand for any country, in which case the statement would not make
any sense. Therefore, a forceful statement would be—
No, because machines need power and India is already short of it.
This question type is a favourite with a few specific institutes. There are four sentences given and the
student has to identify whether the sentences are facts, inferences or judgements. A brief explanation
of what generally constitute facts, inferences and judgements should help.
Facts
1. Categorical Statements—which are not judgmental, opinionated or inferred. The statement
need not be factually true i.e. the student‘s general knowledge should not encroach upon his
evaluation of the statement.
Judgement
2. Predictions—Words like ‗will‘ and ‗shall‘ usually identify these judgements (unless they are
also Universal truths).
3. Judgements—Words like ‗should‘ and ‗must‘ usually identify these (exception, again,
Universal Truths)
Note—Quantifiable modifiers are usually not opinions. An example should explain this. The modifier
in question is ‗only‘.
Directions : In the following question mark the correct sequence of Fact, Inference and Judgement.
This is one of the most mind-boggling questions and the student is requested to be extremely wary of
this type. There are usually five to ten statements following a short passage and the student is asked
to mark the degree of truthfulness or falsity of a statement. Remember
1. The statement is definitely true only when it definitely follows from the passage without any
ambiguity or exception whatsoever.
29 Pankaj Gandhi’s Academy/Logical Reasoning
2. The statement is probably true when it may not definitely follow but all indications are that it
will or the chances of it doing so are very high.
3. The statement is uncertain if it cannot be said to follow with any degree of certainty and there
are no indications to its truth or falsity
4. The statement is probably false when it is in all probability it is false but there might be
chances of its truth, albeit very little, and
5. The statement is definitely false when there is no way that the statement can have any
chances of being true at least from the information given in the passage.
E.g. Directions: Given below is a passage followed by several inferences. Examine the inferences
separately in the context of the passage, and determine whether they are true or false. Mark
(a) if the inference is definitely true, i.e. it properly follows from the statement of facts
given.
(b) if the inference is probably true, though not definitely true, in the light of statement of
facts given.
(c) if the inference is uncertain, i.e. data is insufficient to decide whether the inference is
true or false.
(d) if the inference is probably false, though not definitely false, in the light of statement of
facts given.
(e) if the inference is definitely false, i.e. it cannot possibly be inferred from the statement
of facts given.
In the last thirty years, the annual milk production has more than trebled to 69 million tonnes in 1996.
This extraordinary transformation of dairying in what has been described as a rags-to-riches story has
led to milk being compared to the ‘Cinderella’ of Indian agriculture. While what has been achieved is
impressive, the prospects ahead are challenging. In the Indian context, one dimension of dairy
development that has been equally significant as has been the phenomenal growth in milk production
is its impact in upgrading the life of the rural poor.
1. There has been a three hundred percent increase in milk production in India.
Here, the production is trebled, so the increase is two hundred percent. Therefore the
statement is definitely false i.e. (e).
5. India has been producing milk for the last thirty years.
Obviously, definitely true. Answer is (a).
This question type is relatively simpler and less subjective than the above question type. Basically, it
is a test of whether the student understands the logic in the sequencing of sentences and their
meanings in a paragraph.
Eg. Directions: Given below is a passage followed by a few statements. You have to read the
passage and on the basis of the information available, mark your answer as
(a) if the argument is an upstream argument i.e. if the passage is a direct conclusion or
inference of the question statement
(b) if the argument is a downstream argument i.e. if the question statement is a direct conclusion
or inference of the passage.
Pankaj Gandhi’s Academy/Logical Reasoning 30
(c) if the argument is a lateral argument i.e. if the statement supports the passage but is neither
upstream nor downstream.
(d) if the argument is irrelevant to the passage.
The municipal corporation has sanctioned an educational aid of Rs. one lakh to the families of all
poor, disabled students in the municipality.
(1) Poor, disabled students in municipal schools are largely ignored by their classmates and
teachers. (d)
(2) The municipal board wants to improve its image on the eve of the elections. (a)
(3) Public opinion has swung in favour of the present members of the municipal corporation and
the concerned families have heaved a collective sigh of relief. (b)
(4) The municipal corporation has also contributed to social organisations such as CRY (Child
Relief and You). (c)
The logic of a downstream argument (3 – b) follows exactly the reverse order. In this case, the
passage temporally precedes the question statement, which cannot exist, and does not make
complete sense without the passage.
However, a lateral argument (4 – c) neither follows from nor gives rise to the argument or information
given in the passage. The fact that the municipal corporation has donated to CRY has nothing to do
with the aid to the families of poor, disabled students in terms of sequentiality. The sentence stands
by itself and merely helps to strengthen or support the passage.
2. WEAKNESS / STRENGTH IN ARGUMENT
Eg -1 The municipal corporation has decided to sanction aid of Rs one lakh to the families of poor,
disabled students since it feels that the less privileged of their citizens will benefit from the
largesse.
Which of the following, if true, points out a serious weakness in / strengthens the above
argument?
(a) Many of the families benefiting from the aid might be tempted to spend the money on purposes
other than what it is meant for.
(b) The municipal corporation has often tried to donate to social causes.
(c) The members of the municipal corporation are not united and frequently disrupt the
corporation’s plans.
(d) The municipal corporation has set up a body to ensure that the money is spent on the
education of the disabled students and not frittered away by the families.
In such types, the correct answer to both the questions is often a hidden premise or what is
described as an implicit assumption. This implicit assumption is present in the answer choices. Often,
an idea (statement) will strengthen the argument by stating a true assumption that is implicit in the
argument.
In the passage above, the municipal corporation‘s argument has the underlying assumption that the
money donated by them might be used wrongly and the students might not ultimately benefit from it.
(d) identifies this assumption, thereby strengthening the argument.
An idea (statement) that weakens an argument can be located by –
1. Identifying a false implicit assumption in the argument, which, when attacked, weakens the
argument.
2. The presence of an ‗alternative causal linkage‘ in the options can undermine an argument.
This alternative causal linkage gives a causal explanation besides the one suggested in the
passage.
In example 2, (b) provides a causal explanation other than the one cited in the passage and
completes the argument. It explains the phenomenon of the FIIs withdrawing their funds from the
market.
Eg – 3 The children were asked to play after lunch break so that they would not feel drowsy.
(a) Most of the children wanted to merely indulge in idle talk after a good lunch.
(b) Some of the children spent a good deal of time reading comics.
(c) Some of the children developed stomach cramps due to excessive physical activity
immediately after a heavy lunch.
(d) The school authorities were conscientious in their attitude since they realised that the children
did not get enough time to play.
3. An idea that weakens could also be one that mentions consequences not taken into
consideration by the argument, and which, if considered (and if true), could undermine the
argument. Statement (d) provides a consequence that is not taken into consideration by the
argument and is therefore indicative of a weakness in the argument.
DIRECTIONS: In each of the following questions two or more statements are given under P and Q
respectively. These statements are followed by one inference each. Find out, on the basis of the
statements,
(a), if the conclusion in respect of `P' only is correct.
(b), if the conclusion in respect of `Q' only is correct.
(c), if the conclusions in respect of `P' as well as `Q' are correct.
(d), if both conclusions in respect of `P' and `Q' are incorrect.
4. P. No morning is night. No morning is noon. Therefore
No noon is night.
5. P. All radios are electric goods. All table lamps are electric goods. Therefore
All radios are table lamps.
Q. All electric goods are radios. All electric goods are table lamps. Therefore
All table lamps are radios.
DIRECTIONS: In the following questions one statement is followed by one inference. Assuming the
statement to be true point out if the inference is (a) True, (b) False or (c) Uncertain.
DIRECTIONS: In the following questions one statement is followed by two possible implications.
Study the and mark one of the following answer choices:
(a), Statement 1 is implied.
(b), Statement 2 is implied.
(c), Both the statements are implied
(d), Both the statements are not implied.
DIRECTIONS: In the following questions study that data and the conclusion and mark.
(a), If statement 1 alone can bring us to the conclusion.
(b), If statement 2 alone can bring us to the conclusion.
(c), If both the statements 1 and 2 taken together can bring us to the conclusion.
(d), If the statements taken separately or together cannot bring us to the conclusion.
22. Conclusion : Many children were injured when a bomb exploded in a class room.
Data : 1. India needs to progress and if talented people are neglected India
cannot make progress.
2. There is scarcity of talent in India.
DIRECTIONS: Each question contains six statements followed by four sets of combinations of three.
Choose the set in which the statements are logically related.
DIRECTIONS: In each of the following questions one interrogative statements followed by two
arguments, one beginning with `yes‘ and the other with `no‘. Mark:
(a), if only argument 1 is forceful
(b), if only argument 2 is forceful
(c), if both 1 and 2 are forceful
(d), if neither 1 nor 2 is forceful.
33. Should the people in India be legally forced to adopt family planning programme?
1. Yes, because population explosion is perhaps the greatest threat to India‘s economic
development.
2. No, because nothing in a democratic country like India should be implemented by force.
34. It is good for India to adopt a socialistic pattern of society?
1. Yes, because there is economic disparity and in a democratic state it should not be.
2. No, because it will come in conflict with our social structure.
35. Will democracy survive in India?
1. Yes, because people in India are democratic in outlook.
2. No, because corruption in the working of the government has made the people lose faith
in it.
DIRECTIONS: From the alternatives, choose the one which correctly classifies the four sentences as
F : Fact : if it relates to a known matter of direct observation, or an existing reality
or something known to be true.
38. 1. The urge for human fellowship must be regarded as a basic urge.
2. Indian works of art have had a very illustrious history for centuries.
3. Ganesh Pyne is one of the most prolific Indian painters.
4. Since he is very reclusive, fellowship is not one of his attributes.
(a) JJJI (b) JJFI (c) JFJI (d) JIJF
39. 1. We should see that traditional arts are preserved and carried on from age to age.
2. Industry must be regarded as partnership between management and labour.
3. Casablanca‘s success catapulted Bogart to cult status in Hollywood.
4. Agfa-Loveall Inc. posted huge profits because of incentives to labourers.
(a) JJFI (b) FJII (c) FIIJ (d) JJII
41. If I do not work 10 hours a day, I will not meet my target. If I work 12 hours a day, I will get a
bonus. If I work 10 hours a day, I do not work 12 hours a day. I work 12 hours a day.
Therefore…
(a) I will not meet my target.
(b) I do not work 10 hours a day.
(c) I get a bonus.
(d) Both (b) and (c).
42. If Amitabh Bachchan retires, then Govinda will be happy. If Jackie Shroff does not act, Anil
Kapoor will not act either. If Govinda is not happy, then Amitabh Bachchan will not act.
Which of the following is applicable if Amitabh Bachchan acts?
(a) Govinda will be happy.
(b) Amitabh Bachchan will not retire.
(c) Anil Kapoor will be happy.
(d) Jackie Shroff will not be happy
43. If we unite, we will succeed. If we don‘t unite, we will fall. If we are divided, we will not fall. If
we don‘t succeed, we will fail. Therefore, if we do not fall…
(a) we unite.
(b) we succeed.
(c) we are divided.
(d) we are united.
37 Pankaj Gandhi’s Academy/Logical Reasoning
44. Arun cannot decide what he should do. If he marries Aruna, his father, Alec, will not be
happy. If he marries Arunima, then Alec will be happy. If he marries Aruna, then he (Arun)
will be happy. If he does not marry Aruna, then he (Arun) will not be happy.
Ultimately, if Alec is happy, what has Arun done?
(a) Married Arunima.
(b) Not married Aruna.
(c) Both (a) and (b).
(d) Taken a vow of celibacy.
45. Dolce and Gabbana are designers who work together. If Dolce does not design denim, then
Gabbana also does not design denim. If Gabbana does design summer wear, then Dolce
ends up designing the fall collection. Leather design is Dolce‘s forte and if Dolce designs
leather, Gabbana does not design Rexene.
Which of the following is true if Gabbana designs Rexene?
(a) Dolce designs summerwear.
(b) Dolce does not design the fall collection.
(c) Dolce does not design leather.
(d) None of the above.
DIRECTIONS: Given below is a passage followed by several inferences. Examine the inferences
separately in the context of the passage, and determine whether they are true or false. Mark
(a) if the inference is definitely true, i.e. it properly follows from the statement of facts given.
(b) if the inference is probably true, though not definitely true, in the light of statement of facts given.
(c) if the inference is uncertain, i.e. data is insufficient to decide whether the inference is true or
false.
(d) if the inference is probably false, though not definitely false, in the light of statement of facts
given.
(e) if the inference is definitely false, i.e. it cannot possibly be from the statement of facts given.
Novae and Supernovae are stars whose brightness increases suddenly by 10 to 20 magnitudes or
more and then fades gradually into normal brightness. The distinction between the two types has not
been precisely explained. It would appear that they differ not in kind but in degree. The sudden
increase in brightness is attributed to an outright or a partial explosion. In novae, it seems that only
the outer shell explodes, whereas in supernovae, the entire star explodes. Novae occur more
frequently than supernovae. As Prof. C.F. Powell puts it, ―The whole structure of the star is blown to
pieces, it flares up in brilliance so that its intrinsic luminosity for the first 30 days after the explosion is
equal to 1000 million of our suns.‖
46. Novae and supernovae are always brighter than our sun.
Exercise 1(B)
1. Robin was just elected president of the intellectually challenged peoples‘ association (ICPA),
so he must be a boring person.
The statement above makes which of the following assumptions?
I. Only boring people can be members of the ICPA.
II. Only boring people can be president of the ICPA.
III. Only members of the ICPA can be boring.
(a) I only (b) II only (c) III only (d) I and III only (e) I, II and III
2. Scientists have uncovered evidence that even ancient people believed in the concept of time.
Sites uncovered during archeological expeditions in many parts of the world including places
like India have shown the existence of either hourglasses or huge sundials. The Inca temple in
Peru is the earliest known example of this belief in the ability to calculate time.
Which of the following is an assumption based on the argument above?
(a) The placement of time measuring instruments in the ancient civilizations indicates a feeling
of reverence towards the concept of time.
(b) Belief in the concept of time is the central tenet of most ancient civilizations.
(c) The other tools and artifacts found near the excavated sites show that there were expert
watch repairers available too.
(d) All ancient civilizations worked day to day under a strict regimen based on time.
(e) Only people who believe in the concept of time would have built sundials and manufactured
hourglasses.
3. If Ronin takes the GMAT, then Susan, Tabatha and Vancleef will also take the GMAT.
If the statement above is true, which of the following statements must also be true?
(a) If Ronin does not take the GMAT, then Susan does not take the GMAT.
(b) If Susan Tabatha and Vancleef take the GMAT, then Ronin will also take the GMAT.
(c) If Ronin and Susan take the GMAT, then Tabatha and Vancleef will not take the GMAT.
(d) If Vancleef does not take the GMAT, then Ronin will not take the GMAT.
(e) If Tabatha does not take the GMAT, them Sally will not take the GMAT.
4. The reason that the Prime Minister appointed John Galt as the chairman of the Planning
Commission, and the reason that the conservatives are happy about the decision is that they
believe that Galt‘s appointment, if approved by the Parliament, will ensure that the Commission
is dominated by the far Right for a long time to come. For the same reason, the liberals oppose
him.
The speaker in the above paragraph implies that
(a) liberals believe that Galt would make a good chairman of the Planning Commission.
(b) liberals believe that Galt would take conservative planning decisions.
(c) liberals would not oppose Galt‘s appointment if the conservatives did not support it.
(d) conservatives would not support Galt‘s appointment if the liberals did not oppose it.
(e) Galt should attempt to unite the opposing factions in the government.
5. In a survey of the reading habits of Magazine subscribers, it was found that an average of four
to five people actually read each copy of the most popular fashion magazine, Belle. On this
basis we can conclude that our fashion magazine, Nerve, which sells 6000 copies are actually
read by 24,000 to 30,000 people.
The estimate above assumes that…
(a) individual magazine readers generally read more than one magazine.
(b) fashion changes from one magazine to another.
(c) the ratio of readers to copies is the same for Nerve as it is for Belle.
(d) the number of readers of Belle is similar to that of Nerve.
(e) most readers like to share their copies of magazines with family and friends.
7. These are tests framed in 1996. Therefore they have not been framed by Pinka.
The above statement rests on which of the following assumptions:
(a) Pinka framed tests only after 1996.
(b) Pinka framed no tests in 1996.
(c) Only Pinka framed tests in 1997.
(d) Pinka did not work here in 1996.
(e) No tests were framed in 1996.
8. World No.1 Leander Paes and his partner Mahesh Bhupathi, brought glory for Indian tennis
and India by emerging victors in the Wimbledon men‘s doubles, their second straight Grand
Slam together.
The above statement assumes that:
(a) Leander and Mahesh have played some gay Grand Slams together.
(b) Winning Wimbledon brings glory to one's country.
(c) Leander Paes is World no.1
(d) People know that Paes-Bhupathi are the no.1 doubles pair in the world.
(e) None of the above.
9. Christopher Marlowe wrote Dr. Faustus, a play that is based on necromancy. Dr. Faustus‘s lust
for power and fame tempted him to sell his soul to the devil in exchange for the power to fulfill
all his wishes. This play became very famous during the Elizabethan era when it was staged.
Inspite of the play being didactic and the protagonist being a greedy, lustful and power-crazy
eccentric, the play evoked a curious sympathy towards Dr.Faustus. Even today, the play is an
all time favourite of all literature students. Hence, it can be concluded that for any literary
composition to become popular the protagonist need not always be the epitome of virtue.
What is the major assumption made by the author, in drawing the conclusion?
(a) Any literary composition should have a protagonist, whether good or bad.
(b) Popular opinion is based more on portrayal of characters rather than on plot or technique of
any literary composition.
(c) A successful creative piece is that which is universal in appeal and one that stands the test
of time.
(d) An ignoble character always tends to achieve more popularity because the audiences
satisfy their desire, to be foul, vicariously through these characters.
(e) Dr.Faustus was the first Elizabethan play, which had a protagonist, who was not an epitome
of virtue.
10. In recent years shrimp harvests of commercial fishermen in the South Atlantic have declined
dramatically in total weight .The decline is primarily due to competition from a growing number
of recreational fishermen, who are able to net young shrimps in the estuaries where they
mature.
Which of the following regulatory actions would be most likely to help increase the shrimp
harvests of commercial fishermen?
(a) Allowing only commercial fishermen to fish in estuaries.
(b) Limiting the total number of excursions per season for recreational fishermen.
(c) Putting an upper limit on the size of the shrimp that can be caught by recreational
fishermen.
(d) Requiring recreational fisherman to use large mesh nets in their fishing.
(e) Allowing recreational fisherman to move out of estuaries into the South Atlantic.
Pankaj Gandhi’s Academy/Logical Reasoning 40
11. Being in the sun for just five minutes can lead to darker skin and, in the long run, premature
ageing. Fortunately, Likeme Sunscreen with SPF provides UVA and UVB rays protection.
Likeme Sunscreen Lotion – you‘ll never worry about being in the sun again.
Which of the following undermines the above argument?
(a) Some users of Likeme Sunscreen Lotion have complained of leukodermal patches on their
body.
(b) UVA and UVB rays protection is not desirable since the rays contain elements which
improve the skin's resistance to harmful elements in the atmosphere.
(c) Likeme is a cheap substitute of a US-based cosmetic company that specialises in skin
products.
(d) Sunlight does not lead to sunburn.
(e) None of the above
12. The Universe Encyclopaedia Company Limited that used to sell only encyclopaedias had a
sales figure that was higher than any other company in the market because of its cheap and
competitive pricing of the product. The company has now introduced encyclopaedias in
comprehensive multimedia CD-ROMs from IBM. Since then it‘s sales have fallen.
Which one of the following could be a strengthening argument that would account for the fall in
sales?
(a) The encyclopaedias were expensive so the customers could not afford them.
(b) CD ROMs are available at a much higher price and have limited durability.
(c) The life long source of knowledge is more popular in the printed form.
(d) Most people do not have the infrastructure to use CD-ROMs.
(e) All of the above
13. Kargil is one of the worst less-than-war situations ever between the two countries. The U.S and
the world had justifiable concerns that any escalation in the war may lead to an environment
where the nuclear option is brandished.
Which of the following is an underlying assumption in the passage?
(a) India is a nuclear power.
(b) U.S is a nuclear power.
(c) War situations have an impending nuclear threat in the scenario in question.
(d) None of (a), (b) and (c)
(e) All of (a), (b) and (c)
14. Researchers in Finland compared men who kept their hair into middle age with those who
started to go bald before the age of 30. They found that losing your hair is also a sign that you
are at a significantly greater risk of insulin-resistance related illness like hypertension,
coronary-artery disease, obesity and diabetes. You can counteract some of that extra risk by
keeping your weight down, exercising, eating healthy foods and avoiding cigarettes.
Which of the following is an assumption?
(a) Men who lose hair once they are thirty are normal.
(b) If you eat foods full of carbohydrates like pastries and full cream milk you will lose hair.
(c) That you will not contract insulin related diseases like hypertension, coronary-artery
disease, obesity and diabetes if you keep your weight down.
(d) Some men can go bald.
(e) None of the above
15. Curd : If your curd is not fresh, it is better that you avoid eating it. The old culture can cause a
bout of diarrhoea.
Which one of the following has the same pattern of argument as the argument above?
(a) Salads : If you think it‘s the prawns you ate at the dhaba last night that has upset your
stomach, think again. It could actually be the green salad you ordered to accompany it.
(b) Pencils : Chewing pencils absentmindedly can cause tiny stress fractures in your teeth. So
do not chew pencils.
(c) Water : Water, water everywhere is not necessarily safe to drink. By drinking water from
anywhere you are allowing germs to cause hepatitis and dysentery.
16. A glass of red wine is good for your health. But generally you tend to roll wine over your front
teeth before you swallow it and that could make you look like a Dracula who has just had its
midnight fix of warm blood. So if you are having a cosy dinner for two, choose a Chardonnay
instead of a cabernet.
Which one of the following is an assumption?
(a) You don‘t want to look like a Dracula, when you are on a cosy dinner for two.
(b) You will definitely go on a date.
(c) You do not like the red tinge that the red wine leaves behind.
(d) Most people like to roll red wine over their front teeth.
(e) None of the above
17. A woman who claims a perfect marriage simply fell into her lap, is omitting a sizeable part of
the story. Solid, long-lasting love involves self-sacrifice, compromise and behind-the-scene-
blowups you aren‘t privy to. You only see the façade she shows in public. It‘s like comparing
how you look the first thing in the morning with an airbrushed photograph of a model with
flawless makeup in good lighting.
Which of the following is assumed in the above passage?
(a) When a woman claims a perfect marriage it necessarily involves love.
(b) A perfect marriage is like a well made up model.
(c) All people who are married should be in love.
(d) Do not compare a model with a woman who claims a perfect marriage.
(e) All of the above
18. Most people today have become extremely fitness conscious and careful about calories. The
calorie conscious modern generation is much slimmer compared to the previous generation.
Which one of the following is an inference?
(a) People today are fitter than the people of the previous generation are.
(b) If you are fitness conscious, you will be slim.
(c) This fitness consciousness is part of an increasing trend of health consciousness that was
absent before.
(d) All of (a), (b) and (c)
(e) None of (a), (b) and (c)
19. There are those of us who, determined to be happy, are discouraged repeatedly by social and
economic forces that cause us nothing but trouble. And there are those of us who are blessed
with health and wealth and still grumble and complain.
Which one of the following can you infer from the passage?
(a) There is disparity in distribution of natural resources that can make people happy.
(b) Some people are well endowed and yet cannot be made happy.
(c) There are different kinds of people in the world.
(d) All of (a), (b) and (c)
(e) None of (a), (b) and (c)
20. ―Odissi is beginning to serve a social cause ―, says Gangadhar Pradhan, a well known Odissi
guru. ―There was a need for a change, and the need justifies the effort‖, echoes Ileana
Citaristi, the Italian exponent of Odissi who has made Orissa her home. ― We ran the risk of
boring people. Odissi would have lost its appeal‖, explains Mohanty.
Which of the following statements is an inference?
(a) Odissi is becoming boring these days.
(b) If an art form does not serve a social cause, it loses its appeal.
(c) Now that it serves a social cause, Odissi is no more boring.
(d) All of a, b and c
(e) None of a, b, and c
Exercise 1(C)
Manmohan Singh owns a cosmetic store. In preparation for his annual Big Independence Day sale,
Manmohan Singh arranged with Bharat‘s print shop for the printing of 10,000 colour circulars.
However, before they could be delivered, a fire destroyed Bharat‘s print shop and with it, Manmohan
Singh‘s circulars. The value of business expected on the Independence Day sale being very high and
the time availabe very short, Manmohan Singh has to decide whether to have the work done over
again by another firm or to rely only on the advertisements he has released in the local newspaper.
Also, Manmohan Singh is concerned that the new printing firm is capable of delivering a printing job
of good quality because the Independence Day sale is one of his most important promotional efforts.
But he doesn‘t want to pay too much for the printing. Also, it is essential that Manmohan Singh get
the circulars by 14th August, which is one day before the Independence Day. Manmohan Singh called
Rupam Printing to get an estimate. Rupam Printing told Manmohan Singh they could do the job for
Rs.1500 plus another Rs. 350 for folding and stapling, a total cost of Rs.1850. In addition, they could
guarantee delivery by May 11. Manmohan Singh has heard, however, that the quality of work
delivered by Rupam Printing sometimes suffered because they always did things so quickly. Next
Manmohan Singh called Archie‘s Perfect Prints, a firm with a good reputation. Archie‘s quoted
Manmohan Singh a price of Rs.1350 for the printing plus another Rs.250 for folding and stapling, a
total cost of Rs.1600. But Archie‘s were not certain they could have the work done before May16.
Finally Manmohan Singh called Newlook Printing, a new firm that had only just opened the week
before. Newlook‘s manager, Shivraj Thukral looked anxious for new business and said he would do
all the work for Rs.1100. Manmohan Singh is worried, however, that though Shivraj Thukral appeared
to be dynamic and sincere, lack of experience on his part might delay the work and the work might
not be of top quality.
The following questions consist of items related to the preceding passage above. Consider each item
separately in terms of the passage and on the answer sheet blacken space.
(a) If the item is Major Objective in making the decision; that is, one of the outcomes or results
sought by the decision maker;
(b) If the item is Major Factor in making the decision that is a consideration, explicitly mentioned in
the passage, that is basic in determining the decision.
(c) If the item is Minor Factor in making the decision; that is a secondary consideration that affects
the criteria tangentially, relating to a Major Factor rather than to an Objective.
(d) If the item is Major Assumption in making the decision; that is, a supposition or projection
made by the decision maker before weighing the variables;
(e) If the item is an Unimportant issue in making the decision; that is, a factor that is insignificant
or not immediately relevant to the situation.
1. The cost of printing the circulars destroyed in the fire at Bharat‘s Print Shop.
2. The date by which a reprint of the circulars would be ready.
3. Whether to rely on advertisement alone or replace the circulars lost in the fire.
4. Reputation of Bharat‘s Print Shop as a printer of colour circulars.
5. Total price quotations given by Rupam Printing, Archie‘s Perfect prints, and Shivraj Thukral.
6. Shivraj Thukral‘s lack of experience.
7. Rupam Printing‘s speed of completing any job.
8. The quality of work that would be done by the various firms.
9. Reprinting the circulars of good quality, at a reasonable cost and in time to promote the
Independence Day sale.
10. The value of the business Manmohan Singh expects to do during the Independence Day sale.
Exercise 2 (A)
DIRECTIONS: In each of the following questions two or more statements are given under P and Q
respectively. These statements are followed by one inference each. Find out, on the basis of the
statements,
(a), if the conclusion in respect of `P' only is correct.
(b), if the conclusion in respect of `Q' only is correct.
(c), if the conclusions in respect of `P' as well as `Q' are correct.
(d), if both conclusions in respect of `P' and `Q' are incorrect.
2. P. Some philosophers are wise. All philosophers are human beings. Therefore
All human beings are wise.
Q. Some wise are philosophers. All human beings are philosophers. Therefore
All human beings are wise.
Q. All crows are not jackals. All foxes are not jackals. Therefore
Some crows are jackals.
DIRECTIONS: In each of the following arguments one premiss is missing. Pick out from the answer-
choices the premiss which will complete the argument without imparting any fallacy to it.
DIRECTIONS: In the following questions, each statement is followed by an inference. Assuming the
statement to be true point out if the inference is (a) True, (b) False or (c) Uncertain.
DIRECTIONS: In the following questions one statement is followed by two possible implications. Study
the and mark one of the following answer choices:
(a), Statement 1 is implied.
(b), Statement 2 is implied.
(c), Both the statements are implied
(d), Both the statements are not implied.
18. In the scheme of Nature, waste is useful for the renewal of life.
1. There is waste in the scheme of Nature.
2. The scheme of nature involves renewal of life.
DIRECTIONS: In the following questions a conclusion is followed by two statements which give us data
for the conclusion. Study that data and the conclusion and mark.
(a), If statement 1 alone can bring us to the conclusion.
(b), If statement 2 alone can bring us to the conclusion.
(c), If both the statements 1 and 2 taken together can bring us to the conclusion.
(d), If the statements taken separately or together cannot bring us to the conclusion.
23. Conclusion : The quality of work produced today cannot be compared with the earlier
works.
DIRECTIONS: Each question contains six statements followed by four sets of combinations of three.
Choose the set in which the statements are logically related.
DIRECTIONS: In each of the following questions one interrogative statement is followed by two
arguments, one beginning with `yes' and the other with `no'. Mark:
(a), if only argument 1 is forceful
(b), if only argument 2 is forceful
(c), if both 1 and 2 are forceful
(d), if neither 1 nor 2 is forceful.
DIRECTIONS: From the alternatives, choose the one which correctly classifies the four sentences
as:
F : Fact : if it relates to a known matter of direct observation, or an existing reality
or something known to be true.
DIRECTIONS: In the following questions, two statements are followed by two inferences. Write
(a), if only inference 1 follows
(b), if only inference 2 follows
(c), if both of them follow
(d), if neither of them follows.
DIRECTIONS: Given below is a passage followed by a few statements. You have to read the
passage and on the basis of the information available, mark your answer as
(a) if the argument is an upstream argument i.e. if the passage is a direct conclusion or inference of
the question statement
(b) if the argument is a downstream argument i.e. if the question statement is a direct conclusion or
inference of the passage.
(c) if the argument is a lateral argument i.e. if the statement supports the passage but is neither
upstream nor downstream.
(d) if the argument is irrelevant to the passage.
As managerial downsizing becomes an imperative, corporates are busy innovating on severance pay
packets. The single point strategy for severance pay design is to minimise the impact of income tax
on the recipient.
41. Paymasters are keen on retaining their reputation despite retrenchment of managerial staff.
42. Some companies pay a month‘s salary as royalty for every year of service.
43. Companies realise that they may need to hire back some of the managers.
DIRECTIONS: For questions 46-48: In each of the following sentences the main statement is
followed by four sentences each. Select the pair of sentences that relate logically with the given
statement.
46. After the war Cleo has to visit Julius. Either Cleo will roll out of the carpet or Cleo will be
bitten by an asp. Julius wants Cleo to roll out of the carpet or Antony will lose his heart.
A. Antony will lose his heart.
B. Cleo will roll out of the carpet.
C. Cleo will be bitten by an asp.
D. Cleo will not roll out of the carpet.
(a) AD (b) DC (c) CD (d) DA
47. The exhibition of renowned paintings opens next Saturday at the Boat Club. New painters
are also expected to participate in the exhibition with an on-the-spot painting contest. Mr.
Painter Blunderbuss will participate in the contest. Either Mr. Blunderbuss will paint or Mr.
Blunderbuss will feign. But he is a dilettante and does not want people to know that.
A. Mr. Blunderbuss will not paint.
B. Mr. Blunderbuss will not feign.
C. Mr. Blunderbuss will feign.
D. Mr. Blunderbuss will paint.
(a) AB (b) BD (c) CB (d) DB
48. Either the elephants will donate blood or the elephants will hide behind the ants. The
elephants will donate blood.
(a) the elephants will not hide behind the ants.
(b) the ants will not hide the elephants.
(c) the elephants will hide behind the ants.
(d) none of the above.
DIRECTIONS: Read the following lines and answer the questions that follow.
49. Indian industry has finally said the ‗A‘ word: AIDS. After years of trying to pretend that the
dreaded disease was not a problem at the workplace, the Indian corporate sector is now
looking at the Aids threat in the face.
Which of the following statements, if true, weakens the above argument?
(a) 10 per cent of the companies listed have held Aids awareness programmes.
(b) Most companies spend enormous sums of money on Aids-afflicted employees.
(c) Companies have scarcely advanced beyond awareness programmes.
(d) Aids awareness campaign is a major goal for many companies though participation level
is insignificant.
50. A major problem with investment magazines was that they concentrated only on the stock
market. Their sales have dipped and many publishers of investment magazines have closed
shop.
Which of the following statements, if true, strengthens the above argument?
(a) The downswing in the capital market made investors wary of investments in stock.
(b) Investment opportunities in gold and real estate have slipped past the hands of investors
who were hooked on to investment magazines.
(c) Realistic data regarding companies was absent in investment magazines, which created
undue hype about company issues.
(d) none of the above.
Exercise 2(B)
1. If these paintings were made after 1900, they were not painted by Whistler.
The statement above depends upon which of the following assumptions?
(a) All paintings done after 1900 were painted by Whistler.
(b) All paintings done in 1900 or earlier were painted by Whistler.
(c) Only paintings done by Whistler were painted in 1900 or earlier.
(d) No paintings done in 1900 or earlier were done by Whistler.
(e) No paintings done after 1900 were painted by Whistler.
2. Some of the most popular television commercials are those which employ the technique of
indirect selling i.e. not showing any explicit connection with the product. But as an advertising
technique, this may have drawbacks. Surveys have shown that while most viewers vividly
recall the advertisement, they frequently do not recall the brand associated with it. This casts a
shadow over the ability of indirect advertising to increase product sales and generate revenue.
Which of the following is assumed by the passage above?
(a) Indirect advertising tends to reduce the credibility of the product in the consumers‘ eyes.
(b) Though enjoyable, indirect commercials are often less memorable than serious, relevant
commercials.
(c) A commercial that fails to create brand or product recognition does not help in increased
sales.
(d) Indirect commercials may confuse as many viewers as they inform and entertain.
(e) The ultimate goal of advertising is to increase name recognition of the product being
promoted.
3. Based on the results of a study undertaken by the state health department, the government
has concluded that the system of providing health care to people in their homes, especially the
elderly, is much cheaper than providing the same care in the state nursing homes. But what
the study does not mention is that the cost savings are achieved by severely undervaluing the
work of the home nurses. This system, which is based on the theory of low wages, cannot be
the ideal way to provide health care services to the sick and the elderly.
The criticism above is strengthened if which of the following were true?
(a) Approximately 90 per cent of the cost of providing health care to the patients and the
elderly in their homes consist of the wages for the attendants.
(b) Many elderly patients would like to live with other old people so as to being among their
own and receiving health care there.
(c) Most elderly patients lack the financial resources to purchase at-home health care without
government assistance.
(d) The state plans to introduce and implement a licensing procedure for the nurses and home
attendants who provide health care services for the patients.
(e) In the past unions of health care workers have been able to procure big increases in wages
for them.
4. When the Mayor of Hope, Wisconsin, appointed a Hispanic as the principal of the local
Education Board, the appointment was considered political by many blacks and whites; and
when the mayor appointed a black businessman as the Director of Trade and Commerce
Department, many whites and Hispanics made the same charge. They are all correct. But what
is wrong with that? The appointments were political acts well within the mayor‘s rights as
defined by the City Charter.
Which of the following, if true, would most strengthen the argument above?
(a) The appointed principal of the Education Board and the Director of Trade and Commerce
were as well qualified as any other candidate for the position.
(b) Shortly after the two appointments, the mayor appointed a white man as the District
Attorney.
(c) Blacks who opposed the appointment of a Hispanic as the principal had favoured another
black for the same post.
5. In a marked reaction against the extreme puritanism and rigid verse and prose structures of
the Victorian age, the playwrights and writers of the Reformation went the opposite extreme
abjuring all that was ornate or rigid in tone and words, and declaring only amoral, light, free
versed texts to be real literature. Today there seems to be a reevaluation of this view, as
exemplified by the current slew of works with ornate, rigid styles and complicated plots in this
year‘s Booker Prize nominees.
It can be inferred from the passage above that the present movement among the literature
critics is toward
(a) a renewed appreciation of the use of complicated and ornate styles in literature.
(b) a rejection of the Reformation styles of writing prose and plays.
(c) a greater admiration of the light and licentious themes of the later age.
(d) the adaptation of the Victorian style in the works of the Reformation poets.
(e) a deeper understanding of the neo-Victorian values underlying the Reformist theory.
6. Some scientists believe that, in some species of birds, there are metal particles in their heads
that act as compasses and help them fly accurately over long distances during migration, as
they react to the earth‘s magnetic field. To test this, they removed these metal pieces from the
heads of a few birds surgically and released them along with untreated birds on their migratory
journey.
Which of the following results would most seriously weaken the theory being tested?
(a) The unaltered birds were irritated by the erratic flight patterns of the altered birds and drove
them off from the flock.
(b) The altered birds could not follow the usual route and got hopelessly puzzled.
(c) The altered birds formed their own group and followed the migratory route two weeks after
the original flock did.
(d) The altered birds were able to follow the migratory path by day but not by night.
(e) The altered birds changed the route and went to a new migratory destination successfully.
7. The ratio of post-graduates opting for teaching has declined over the past decade. The HAPTA
(Help All Poor Teaching Associates) has therefore decided to hike salaries and offer attractive
income tax relief packages to those opting to teach at Post-Graduate level in an attempt to lure
students to this career.
Which of the following is an assumption justifying the action of the HAPTA?
(a) Fewer students have been enrolling for Post-Graduate courses in the last few years.
(b) Most students are interested in professional courses like the MBA.
(c) Several students are opting for careers in the film industry.
(d) The HAPTA can implement any policy approved by the government.
(e) Increasing the benefits would help lure more people to opt for teaching.
10. The 38 corporations that filed United States income tax returns showing a net income of more
than $100 million accounted for 53% of the total taxable income from foreign sources reported
on all tax returns. Sixty percent of the total taxable income from foreign sources came from the
200 returns reporting income from 10 or more countries.
If the statements above are true, which of the following must also be true?
(a) Most of the total taxable income earned by corporations with net income above $100 million
was earned from foreign sources.
(b) Wealthy individuals with large personal incomes reported 47 percent of the total taxable
incomes from foreign sources.
(c) Income from foreign sources amounted to between 53 and 60 percent of all reported
taxable income.
(d) Some of the corporations with net income above $10 million reported income from 10 or
more countries.
(e) Most of the tax returns showing income from 10 or more countries reported net income of
more than $100 million.
11. One of the covenants of the Hippocratic oath is ―I will follow that method of treatment which
according to my ability and judgement I consider fit for the benefit of my patient and abstain
from whatever is harmful or mischievous‖. For centuries this sacred trust between doctors and
patients bestowed the medical profession with a nobility that has endured. Even today in India
there are thousands of competent doctors who are dedicated to the profession and render
extraordinary service.
Which one of the following is an assumption in the passage?
(a) In recent times the sanctity of the doctor-patient relationship is being besmirched by a few
commercialised hospitals.
(b) There has also been a profusion of commercialised institutions with hardly any
accountability.
(c) The Hippocratic oath is taken by doctors who are expected to adhere to it in word and spirit.
(d) All of (a), (b) and (c)
(e) None of (a), (b) and (c)
12. If a cabinet minister gets ―misdiagnosed‖ by a super-speciality hospital like the Indraprastha
Apollo, what happens to the lay person in the kitchen clinics? At least the minister‘s family had
a central team of doctors probing his death. What can the lay person, who has to go to the new
sprung unregistered private hospitals, do?
Which one of the following is an assumption on which the passage is based?
(I) A super-speciality hospital or a central team of doctors cannot diagnose the lay person.
(II) The private hospitals are not as efficient as the super-speciality hospitals.
(III)The minister would have been alive if he had consulted a kitchen clinic.
(IV)A kitchen clinic is not as efficient as a super-speciality hospital.
(a) I, II and III (b) Both II and IV (c) All of the above (d) Both II and III (e) Only I
13. Do you know the one about the grateful genie who promises his liberator one impossible wish?
―I‘ve always wished for a highway spanning the Atlantic Ocean from Marseilles to New York‖,
says the young man. ―Difficult‖, muses the genie.‖ All the raw material and engineering, all that
effort… too much to ask for.‖ ― Well then, tell me the deepest secrets of a woman‘s heart.‖
Right, says the genie brightly, did you want two lanes on that highway or four?
Which one of the following is an inference from the passage?
(a) The genie is not really grateful.
(b) The genie is a male chauvinist pig because he thinks that a woman‘s heart is too deep to
be fathomed.
(c) The genie thinks that it is easier to build the highway than tell the young man the secrets of
a woman‘s heart.
(d) All of (a), (b) and (c)
(e) None of (a), (b) and (c)
Pankaj Gandhi’s Academy/Logical Reasoning 52
14. Clothes are out. As we know, that is. The world‘s first commercially available electronic
clothing is about to go on sale in high streets across Europe. Clothes equipped with fully
integrated computer networks have been designed and developed.
Which one of the following statements is a strengthening argument?
(a) These clothes do not have to be ironed at all.
(b) Jackets will be equipped with a mobile phone, a portable audio device, a remote control
panel, a microphone and head phones.
(c) If these clothes have a lot of facilities they are sure to cost a lot so the common man will not
be able to afford them.
(d) The company that produces these clothes will be the first of its kind.
(e) None of the above
15. It‘s not for nothing that Mumbai is talked about as the most cosmopolitan of Indian cities. There
is no other city, which treats its women with more respect. It‘s not unusual to see women
travelling alone by the local train in the dead of the night.
Which of the following sentences is an inference?
(a) Women who live in metropolitan cities are safer than those who live in the smaller towns.
(b) In a cosmopolitan city, women can travel by the local train.
(c) If a woman can travel alone in the dead of the night, it means that the city treats its women
with respect.
(d) All of (a), (b) and (c)
(e) None of (a), (b) and (c)
16. You are wrong! Using a hands-free kit for your cell phone will not save you from the dreaded
radiation. According to a British consumer research magazine, using hands-free ear- pieces
could more than triple the brain‘s exposure to radiation, compared to a conventional mobile
phone.
Which one of the following is an assumption?
(a) A conventional mobile phone emanates harmful radioactive rays.
(b) The British consumer research magazine is a reliable and dependable source.
(c) You believe that a cell phone with a hands free ear-piece does not expose the brain to
radiation.
(d) Scientists believe that electromagnetic radiation from mobile phones warms brain tissue
(e) All of the above
17. Modern X-ray machines used to peer into checked-in-baggage are not necessarily film-safe
and can damage your film. So in order to protect your memories of the holiday always carry
film – exposed or unexposed – in your carry-on-luggage, which is subject to film-safe X-ray.
Which one of the following is an assumption on which the passage is based?
(a) Carry-on-luggage is not put through a x-ray check.
(b) You are returning from a holiday.
(c) Film is sensitive and can be affected by the kind of emissions involved in x-rays.
(d) Modern x-ray machines are important parts of baggage checks at the airports.
(e) None of the above
18. "Why can't I be a PM? I am more qualified than Sohrabji. I've gone to jail more times."
Which of the following is an assumption underlying the above statement?
(a)You have to go to jail to be a PM.
(b) The more times you have been to jail the more suitable you are for post of PM.
(c) Sohrabji is an eminent politician who has been to jail several times.
(d) Most politicians who have not been to jail have questionable qualifications.
(e) All of the above
19. Psychiatrists at Duke University recruited 87 people who‘d been diagnosed with depression,
while hospitalised for another illness. They then asked them about their spiritual beliefs.
During the following year, the researchers found that those with the strongest faith shed their
depression the fastest. Other studies have shown that believers are less likely to get
depressed.
53 Pankaj Gandhi’s Academy/Logical Reasoning
Which one of the following can be inferred from the passage?
(a) Depression can be effectively combated if one has a belief in oneself and a higher form of
existence.
(b) Belief in God is always beneficial.
(c) People who are ill and depressed at the same time are usually non-believers.
(d) Belief is a catalyst towards a healthy mental attitude.
(e) None of the above
20. No one put it more bluntly than Indira Gandhi. Delivering a lecture in Rome in 1981, she said
that an intercontinental missile costs the same as setting up 65,000 primary schools. This
candid juxtaposition explains why India is both a world topper in illiteracy and child labour.
Which one of the following can be inferred from the given statement?
(a) If India were not a nuclear power it could have had 65,000 primary schools.
(b) India has chosen to develop military might in preference to children‘s health and education.
(c) Indira Gandhi was in a position to influence literacy and labour.
(d) Both (b) and (c).
(e)All of (a), (b) and (c)
Exercise 2(C)
Meena Patel has saved Rs.1,00,000 and plans to open her own beauty parlour. She must decide
which of the three locations at which to rent :15 Veer Lok lane,20/A Somnath Road, or 35 Haribhai
Chowk. She wants to keep renovation costs to a minimum because some of the Rs.1,00,000 must be
used for down payments on equipment needed and to cover expenses during the start-up period.
Meena has calculated the renovation costs for the three sites:
Meena feels that she would be able to persuade the owner of 35 Haribhai Chowk to bear the
expenses of the renovations. Since Meena expects her business to grow, she feels she should rent
as large a space as possible. Finally, Meena wants to run a profitable business, so she needs to
keep the monthly rent as low as possible.
The following questions consist of items related to the preceding passage above. Consider each item
separately in terms of the passage and on the answer sheet blacken space.
(a) If the item is Major Objective in making the decision; that is, one of the outcomes or results
sought by the decision maker;
(b) If the item is Major Factor in making the decision that is a consideration, explicitly mentioned
in the passage, that is basic in determining the decision.
(c) If the item is Minor Factor in making the decision; that is a secondary consideration that
affects the criteria tangentially, relating to a Major Factor rather than to an Objective.
(d) If the item is Major Assumption in making the decision; that is, a supposition or projection
made by the decision maker before weighing the variables;
(e) If the item is an Unimportant issue in making the decision; that is, a factor that is insignificant
or not immediately relevant to the situation.
1. Keeping the cost of renovating a site as low as possible.
2. Sufficiency of Rs.1,00,000 for starting up the business.
3. Size of each site under consideration
4. Cost of a new storefront for the 20/A Somnath Road site
5. Expense that would be incurred to paint any one of the premises
6. Length of lease offered at each of the locations being considered
7. Necessity of minimizing the cost of renovating a site
8. Monthly rental charged at the sites located at 15 Veer Lok lane,20/A Somnath Road,and 35
Haribhai Chowk
9. A site with a reasonable rent that does not require extensive renovation
10. Possibility of persuading the owner to do the renovation required at 35 Haribhai Chowk .
Exercise 3 (A)
DIRECTIONS: In each of the following questions some statements are followed by two inferences. Point
out if:
(a), Conclusion 1 follows.
(b), Conclusion 2 follows.
(c), Conclusion 1 and 2 follow.
(d), Neither 1 nor 2 follows.
1. Statements : Time saving devices constitute a new work culture which removes sloth.
India needs it badly.
Conclusions : 1. Indians are slow and slothful.
2. India does not have time saving machines.
2. Statements : The Planning Commission is opposed to the proposal. It feels that the cost
of subsidising helicopter operations will be exorbitant.
Conclusions : 1. The Planning Commission plans expenditure for the government.
2. Helicopter operations must be subsidised
DIRECTIONS: In each of the following arguments one premiss is missing. Pick out from the answer-
choices the premiss which will complete the argument without imparting any fallacy to it.
3. She is proud because she is beautiful.
(a) All proud are beautiful.
(b) All beautiful are proud.
(c) Some proud are beautiful.
(d) Some beautiful are proud.
DIRECTIONS: Assuming the statement to be true point out if the inference is (a) True, (b) False or (c)
Uncertain.
5. Statement : No glass is a jar.
Inference : No jar is a glass.
6. Statement : Some summer tonics are good.
Inference : Some good things are not summer tonics.
7. Statement : No non-toxic thing is harmful.
Inference : All non-toxic things are beneficial.
8. Statement : Strong light is harmful to eyes.
Inference : All things harmful to eyes is strong light.
9. Statement : All that glitters is not gold.
Inference : Some things which glitter are not gold.
DIRECTIONS: In the following questions one statement is followed by two possible implications. Study
them and mark one of the following answer choices:
(a), Statement 1 is implied.
(b), Statement 2 is implied.
(c), Both the statements are implied
(d), Neither of the statements is implied.
Pankaj Gandhi’s Academy/Logical Reasoning 56
10. Woman is the embodiment of sacrifice.
1. Man cannot make sacrifices.
2. Woman generally makes sacrifices.
DIRECTIONS: In the following questions a conclusion is followed by two statements which give us data
for the conclusion. Study that data and the conclusion and mark.
(a), If statement 1 alone can bring us to the conclusion.
(b), If statement 2 alone can bring us to the conclusion.
(c), If both the statements 1 and 2 taken together can bring us to the conclusion.
(d), If the statements taken separately or together cannot bring us to the conclusion.
DIRECTIONS: Each question contains six statements followed by four sets of combinations of three.
Choose the set in which the statements are logically related.
DIRECTIONS: In each of the following questions one interrogative statement is followed by two
arguments, one beginning with `yes' and the other with `no'. Mark:
(a), if only argument 1 is forceful
(b), if only argument 2 is forceful
(c), if both 1 and 2 are forceful
(d), if neither 1 nor 2 is forceful.
23. Should the President of India always accept the advice of the Prime Minister?
1. Yes, because the Indian Prime Minister is directly elected by the people.
2. No, because if the president is to accept the advice of the Prime Minister always, the former
becomes unnecessary.
DIRECTIONS: for Qns 36 to 40. From the alternatives, choose the one which correctly classifies the
four sentences as a
27. 1. All living things should die that out of death come life.
2. As human beings we may be biased but as members of nations we tend to be better.
3. Tendulkar is the best batsman in the Indian cricket team.
4. But Azharuddin‘s batting average is the highest.
(a) FIJF (b) JIFF (c) FIJJ (d) JJJF
DIRECTIONS: In the following questions, two statements are followed by two inferences. Write
(a), if only inference 1 follows
(b), if only inference 2 follows
(c), if both of them follow
(d), if neither of them follows.
36. If these lectures do not conform to our religious books these are harmful; if these are profound
they are useless. If these are educational in nature, they are not seditious but are useless.
Which of the following is true if the lectures are not useless?
(a) They are seditious and conform to religious books.
(b) They are neither religious nor profound.
(c) They are neither educational nor profound.
(d) None of the above.
37. If the train is not late I will miss the train; if it is late I will miss my appointment. If I do not miss
my appointment, the train is not late. The train is late. Therefore…
(a) I will not miss the train or my appointment.
(b) I will miss my appointment.
(c) I will not miss the train.
(d) Cannot be determined.
38. If she is beautiful I will marry her. If she is not beautiful I may not like her. If she has a good figure,
I will give her the glad eye. If I do not look at her, she will give me a black eye. Which of the
following is true if I have perfectly normal eyes?
(a) She is not beautiful.
(b) She does not give me a black eye.
(c) I have given her the sad eye.
(d) Cannot be determined.
39. If he stands first he will continue his studies; if he does not he will discontinue his studies. If he
does not continue his studies, he will open a paan shop. If the paan shop does not run, he will
start studying again. He will discontinue his studies. So,
(a) he will not stand first.
(b) his paan shop does not run.
(c) he stands first.
(d) he starts studying again.
40. If it rains I will come; if it does not rain I will not come. If it is sunny, I will stay at home. If it is
cloudy I will come. If it is not sunny, I will go.
I will come. Therefore
(a) it is cloudy
(b) it rains.
(c) it is not sunny.
(d) All of the above.
DIRECTIONS: Given below is a passage followed by several inferences. Examine the inferences
separately in the context of the passage, and determine whether they are true or false. Mark
(a) if the inference is definitely true, i.e. it properly follows from the statement of facts given.
(b) if the inference is probably true, though not definitely true, in the light of statement of facts given.
(c) if the inference is uncertain, i.e. data is insufficient to decide whether the inference is true or
false.
(d) if the inference is probably false, though not definitely false, in the light of statement of facts
given.
(e) if the inference is definitely false, i.e. it cannot possibly be from the statement of facts given.
South Korea‘s highest court struck down a 14th century law that prevented people having the same
surname from marrying, ruling in July, ‘97 that it was unconstitutional and outdated. The decision
affected an estimated 60,000 couples who live together but whose clan names had prevented them
from marrying even though there was no evidence of blood ties. The law was written in 1308, when
in-breeding was a concern because people lived in isolated villages for generations. It has had a
major impact in a country where most of the 44 million people share a few dozen surnames. One out
41. About 20 percent of the South Korean population are named Kim.
42. A lot of the couples living together in South Korea will marry after the court order.
43. About one person out of every eight people in South Korea have the name Park.
44. The basic reason for the law was to prevent marriage between blood relations.
45. Before July, ‘97, couples with the same surnames sometimes married.
DIRECTIONS: In each of the following questions, some statements are followed by two inferences.
Mark
(a) conclusion 1 follows,
(b) conclusion 2 follows,
(c) both follow or
(d) neither follows.
46. Statements : Rich countries average more than one main telephone line for every two
people.
47. Statements : Florence has drawn up a handbook of good manners for tourists.
48. Statements : At least one child out of four in developing countries is toiling under
conditions resembling slavery.
DIRECTIONS:In the following questions a few statements are given on the basis of which two
inferences are drawn. As the answer, you have to mark
(a) if only conclusion 1 follows ;
(b) if only conclusion 2 follows ;
(c) if both the conclusions follow ;
(d) if neither of the conclusions follow.
49. All Harrys are Fritzes. All Fritzes are Sonnys. All Sonnys are Santys. All Santys are Sintos.
Therefore,
1. All Harrys are Sintos
2. All Sintos are Harrys.
50. All politicians are foxes. Some foxes are crabs. Some crabs are alligators. All alligators are
stupid. Therefore,
1. All politicians are stupid
2. Some stupid are politicians.
Exercise 3(B)
2. Students of St. Stephen‘s College must get a better standard of education than students of
Christ College because the average marks of students of St. Stephen‘s is higher than that of
students at Christ.
The claim above depends upon which of the following assumptions?
I. The average marks earned by students is a good measure of the quality of education that a
student receives.
II. Extracurricular activities at St. Stephen‘s are given more emphasis than at Christ.
III. The grading standards at the two colleges are roughly the same.
(a) I only (b) III only (c) I & III only (d) II & III only (e) I, II & III
3. Religious leaders in our country were once expected to express and articulate lasting moral
values. This is no longer the case. Today we see, increasingly, religious spokespersons
entering the arena of political matters and commenting on overtly political issues formerly
reserved only for the member of the secular community.
The logical structure of the passage above depends upon the author‘s assumption that the
expression of lasting moral values is
(a) a strictly religious function.
(b) no longer deeply valued.
(c) essentially nonpolitical.
(d) reserved for secular society.
(e) the most important role of religion.
4. Those in the business of movies say that the best examples of cinema are the ones that
appeal to the common man with common subjects and lots of special effects. The final purpose
of movies being to earn money, there has to be a compromise at some stage or the other,
leading to the commercial elements being incorporated into the movies. They cite the
examples of the top grossing movies like Jurrassic Park, Titanic and Star Wars to prove the
point. They forget that there have been a lot of movies without the help of these commercial
elements and special effects like Casablanca, The Sound of Music and A Clockwork Orange
and they have been equally successful at the box-office.
The author‘s point is made primarily by
(a) offering a counterexample to rebut his opponent‘s argument.
(b) calling into question the motives of his opponents.
(c) pointing out an inconsistency in his opponents‘ use of terms.
(d) drawing a distinction between valid and invalid methods of arguments.
(e) underscoring the subjectivity of his opponents‘ basic assumptions.
5. Scientists have found that rubber swimsuits actually have a high degree of friction against
water, thus working against the competitive swimmer. A highly elastic, but non-cohesive
material like lycra would instead help the swimmer to be faster without any danger of the suit
soaking up the water. The swimsuits of the future are sure to be made out of some such
material.
Which of the following, if true, would most weaken the argument above?
(a) Swimsuits made of lycra would be cheaper to manufacture than those of rubber.
6. A recent study conducted by an independent organisation reported that only 15 per cent of
college bound women would consider studying in an all-girl‘s college. It is clear that women‘s
colleges must become co-educational in order to survive.
Which of the following, if true, would most weaken the argument above?
(a) Graduates of women‘s colleges have better records of getting into medical and professional
schools than do students of co-educational colleges.
(b) Fewer than 10 percent of all college bound men would consider attending an all-male
college.
(c) The total capacity of all-women‘s colleges can accommodate only 2 per cent of the nation‘s
college bound female students.
(d) Many women‘s colleges report a decline in funds contributions in the last few years.
(e) The total number of college bound students is projected to shrink in the next few years.
7. There is a vague popular belief that lawyers are necessarily dishonest. I say vague, because
when we consider to what extent the clients inform their counsellors the most intimate details
of their personal and financial lives, it seems improbable that the impression is very distinct
and vivid. Yet, the impression is common, almost universal. Let no young person choosing law
as a profession for a moment yield to the popular belief. Resolve to be honest at all events;
and if in your own judgement, you cannot be an honest lawyer, then resolve to be honest
without being a lawyer.
Which of the following, if true, most weakens the speaker‘s claim that most people have no
specific reasons for distrusting lawyers?
(a) Most people would not confide in a person they would not trust.
(b) The inadvertent distortion of incidents in the course of the trial leads to doubts in the minds
of people, regarding the righteousness of the lawyers involved.
(c) Most people have got very little direct contact with lawyers.
(d) Most people do not seek the assistance of a lawyer unless they have been arrested.
(e) Few people have ever become the victim of a dishonest lawyer.
8. In the above argument, the advice offered by the speaker to young people considering a
career in law implies that
(a) being honest is more important than becoming a lawyer.
(b) being a lawyer is more important than being honest.
(c) it is virtually impossible to be a honest lawyer.
(d) most people find it very difficult to be honest.
(e) practising law makes it exceptionally difficult to be honest.
9. In national surveys conducted by the Society for Proliferation and Consumption of Alcohol
(SPCA) between 1990 and 1995, the percentage of respondents who reported that they visited
bars and pubs for a drink regularly rose from 29 to 35 per cent. However, statistics compiled by
Bar Owners In League (BOIL), the organisation made up of the management of the major bars
in the country showed a gradual decline in the number of people visiting these pubs.
Each of the following, if true, could help explain the apparent contradiction in the statements
above, EXCEPT
(a) There has been a sharp drop in the number of persons visiting pubs on an occasional
basis.
(b) Attendance statistics compiled by the BOIL are often highly inaccurate.
(c) As older pub-goers died or gave up drinking after being plagued by ill health, they were
replaced by an equal number of young drinkers.
(d) There was a marked rise in the number of people opting to drink in the privacy of their own
homes.
63 Pankaj Gandhi’s Academy/Logical Reasoning
(e) Those responding to the surveys were not representative of the population as a whole.
10. If Pakistan wins the World Cup, then the World Cup must be a fixed tournament.
The statement above depends upon which of the following assumptions?
(a) Only Pakistan can win a tournament if it is fixed.
(b) Pakistan can win the World Cup only if it is fixed.
(c) No team can win the World Cup unless it is fixed.
(d) Pakistan is a good team.
(e) Pakistan will not win the World Cup.
11. It was amazing to see the nonchalant attitude of some cricketers after the raids. The ‗god‘ who
cried on TV could only say he felt ―wonderful‖. These cricketers do not deserve the CBI kind of
polite interrogation. They should be subject to third degree questioning. That seems to be the
only option, since we have no Cronjes among our cricketers to confess.
Which one of these statements has the author assumed while making this statement?
(a) The Indian cricketers have been dishonest.
(b) These cricketers are not revealing essential facts about the areas they are questioned on.
(c) Third degree questioning makes people admit to their wrongs.
(d) Cronje is an honest hypocrite.
(e) All of the above
12. Call it pathetic fallacy if you will but among many popular theories - like the one that says God
is male – there exists another perception that the www is English. The truth is quite the
contrary. For an increasing number of people who either studied Newton‘s theories in Hindi or
didn‘t study them at all, the web today is a tool that understands their mother tongue.
Which one of the following statements will weaken the argument?
(a) Out of the 7.5 million people who view the page of the Web Duniya, a site that allows you to
send mails in regional language, only 2 million people have accounts.
(b) All one has to do is just type the message in English and the message is automatically
transliterated into the regional language chosen.
(c) This silent popularity of communication in Indian languages stems from the fact that most
Indians feel that English doesn‘t convey their feelings accurately enough.
(d) The software used has a code that will state which English letters combination should be
replaced by which letter of the regional language chosen.
(e) None of the above
13. Robots have the ability to exhibit only programmed behaviour. Their performance can range
from the simplest of activities to the most complex group of activities. They can not only build
other robots, but can also repair themselves. Physically they may resemble human beings but
mentally they do not. Even the most highly advanced robot does not have the capacity to be
creative, have emotions or think creatively.
Which one of the following can be inferred from the argument?
(a) Robots have reached their peak of development.
(b) The most complex group of activities necessitates independent thinking.
(c) Creativity cannot be programmed.
(d) Human beings do not lead a programmed life.
(e) All of the above
14. Entrepreneurs innovate. Innovation is the specific instrument of Entrepreneurship. It is the act
that endows resources with a capacity to create wealth. Innovation indeed, creates a resource.
There is no such thing as a ―resource‖ until man finds a use for something in nature and thus
endows it with an economic value.
Which one of the following statements will be a strengthening argument?
(a) Innovation does not have to be technical and need not necessarily contribute to the
productive level of the economy.
(b) Not more than a century ago, neither mineral oil seeping out of the ground, nor bauxite, the
ore of aluminium was a resource; today they are all valuable resources.
(c) Without innovation, the tremendous expansion of world trade in the last forty years could
not have possibly have taken place.
Pankaj Gandhi’s Academy/Logical Reasoning 64
(d) Most big companies of the world today have made it from almost nothing based on the
initiative and creativity of a few good men.
(e) None of the above
15. Scarcity is not created by war; it is a permanent characteristic of all human society and has
been faced by the whole of the human race. It springs from the fact that the material resources
of the world are limited and that our ability to make use of those resources is even more limited
by our ignorance.
Which one of the following statements will be a weakening argument?
(a) As our knowledge grows and we increase our skill, we can exploit more and more of the
opportunities that nature offers us.
(b) The Intel Brains and Co. have used the Magnetic Resource Re-sounder and have prepared
an extensive chart of all the material resources of the world.
(c) If Resource Management were taught even in the school level, there would be no scarcity.
(d) Scientific knowledge has made accessible many valuable minerals from depths below the
earth‘s surface that could not be reached by earlier generations.
(e) All of the above
16. The best method to stimulate your memory is to use it to the utmost. Challenge your memory
with memory testing games. Learn new skills. Absorb new information. Maintain a curious
frame of mind. Remember learning does not stop at school or college.
Which one of the following arguments will strengthen the argument?
(a) Memory can be improved only within a limited range.
(b) The more you try to learn, the more active the brain is. The more active the brain, the better
the memory power.
(c) In advancing years, the individual has already absorbed a lot of information, much of which
is probably irrelevant.
(d) both (b) and (c).
(e) None of the above
17. It‘s not that man cannot express emotions. Men have never been rewarded for expressing
emotions. You are rewarded for doing things, you are rewarded for coming first, and you are
rewarded for scoring a goal, for beating up somebody, for climbing a tree. Being a man is
about doing.
Which one of the following strengthens the argument?
(a) Being a man is not about telling people what you feel.
(b) Men have been taught from childhood to do, to act because action is the male‘s
preoccupation.
(c) Women have to do all the expression, because the woman unlike the man cannot be the
silent actor, since she has to emote and express all that she feels.
(d) All of the above.
(e) None of the above
18. Soon after the Russian Revolution, the Soviet Union declared that education and electricity –
the two Es – were the two eyes of the country. India is different. And that is why we turn a blind
eye to both education and electricity.
Which of the following is a weakening argument?
(a) Electricity is the driving force behind economic progress. There is hardly any sector – or
any person that can do without electricity.
(b) Since Indian independence, the money spent on education and electricity has been on the
increase.
(c) The chairman of a State Electricity Board in India confessed in an interview to Channel Six
yesterday that he did not wish to support proposals for private sector generation of
electricity.
(d) Many MNCs are now willing to set up centres in India, but the frequency of power failure
puts them off.
(e) All of the above
20. Variety is definitely in. A lifetime in one industry is out. Employees can, thus, lessen their
career risk by being open to an industry shift. Conversely, employees with experience across
industries stand less chance of finding themselves at a dead-end than those without.
Which one of the following is a weakening argument?
(a) If I am recruiting for an FMCG firm, I consciously widen the net beyond FMCG companies,
in the hope that the new manager will bring in a fresh perspective and skills to the
assignment.
(b) Usually, people who change learn with every new assignment and are not necessarily
unstable.
(c) A jack of all trades is a master of none because the more fields you try to put your foot into,
the worse your performance becomes.
(d) In the present day scenario, there is no meaning in sticking to just one job.
(e) All of the above
Exercise 3(C)
By any measurement, the Tamboli Wood Company was a small Company. Ganesh Tamboli came to
U.P. in 1900. He was thirty years old at that time and skilled in the use of lathes, sanders and wood
milling machines. After working in various employments for five years in a growing industrial city, he
set up a small shop turning out axe handles. The shapes of the handles and the quality of wood he
used won a modest market for Tamboli. Other wood products such as newel posts, hand rails and
miscellaneous home building supplies requiring fine turning accounted for 20% of his total sales of
Rs.130,000 in the year 1930.
Upon graduation from college in 1932 Mahesh Tamboli the elder son joined his father's business
while the younger son Hari gained employment in a plastic producing company. When Ganesh
Tamboli died in 1939, each of his sons was well established in his respective work. Sales of the
Company were Rs. 450000 in 1939.
In 1949 Mahesh was seriously considering selling the Tamboli Wood Company. The sales of the
Company had increased to Rs.6,00,000. The profits of the company produced a good living to
Mahesh but the work had become such a routine matter that Mahesh was dissatisfied. He began to
investing opportunities into which he could move and new production areas into which he could shift
the company.
It was then Hari suggested they establish a basic plastics company using the space and office
facilities of the Tamboli Wood Company. Each realized that, if successful, this would constitute a
complete shift for the company. Together they planned a new venture. The items they isolated as
being of prime importance were:
(1)markets for the basic plastic
(2) production facilities and process layout
(3) Man power skills and
(4) finance.
After studying the possible alternative, they decided on a course of action which would retain the axe
handle production, first on a full scale and then on a decreasing scale in order to provide a flow of
cash during the transition period. Space in the Tamboli building was made available to Hari, for
laboratory work and for experimental production runs.
When a marketable product had been developed, Mahesh arranged for it to be handed by a
manufacturer's representative. The first pieces of equipment were brought for the plastics production
process for Rs.200,000. Axe handles production was reduced. Incoming cash was severely
budgeted.
A critical point was reached when the axe handle sales were reduced to Rs. 300,00 the lowest they
could be reduced in terms of sales commitments made by the company. At this time the growing
amount of plastic sales was only Rs. 50,000 which was not sufficient to carry the company. A point of
decision had been reached - either to move entirely into plastic or to move back again into axe
handle production.
The decision of the brothers was to shift entirely into plastics. The decision entailed a severe
financial drain and in the first six months the company incurred a loss of Rs. 100,000. Each brother
turned in his savings into the company- Mahesh gave Rs.40,000 while Hari gave Rs. 30,000. Mahesh
devoted all his time to learning the new production process while Hari continued with his lab work. He
also tried to establish contacts with local plastic users. For a year the volume of sales through the
manufacturer's representative fluctuated between Rs. 5,000 to Rs. 15,000 a month. But in order to
meet peak demands further equipment worth Rs. 80,000 would have to be installed.
The following questions consist of items related to the preceding passage above. Consider each item
separately in terms of the passage and on the answer sheet blacken space.
(a) If the item is Major Objective in making the decision; that is, one of the outcomes or results
sought by the decision maker;
(b) If the item is Major Factor in making the decision that is a consideration, explicitly
mentioned in the passage, that is basic in determining the decision.
(c) If the item is Minor Factor in making the decision; that is a secondary consideration that
affects the criteria tangentially, relating to a Major Factor rather than to an Objective.
(d) If the item is Major Assumption in making the decision; that is, a supposition or projection
made by the decision maker before weighing the variables;
(e) If the item is an Unimportant issue in making the decision; that is, a factor that is insignificant
or not immediately relevant to the situation.
DIRECTIONS: In the following questions study that data and the conclusion and mark.
(a), If statement 1 alone can bring us to the conclusion.
(b), If statement 2 alone can bring us to the conclusion.
(c), If both the statements 1 and 2 taken together can bring us to the conclusion.
(d), If the statements taken separately or together cannot bring us to the conclusion.
9. Conclusion : The cheaper the inputs, the lower will be the price of food.
Data : 1. The price of food varies directly as the inputs.
2. The price of food must be lowered.
DIRECTIONS: Each question contains six statements followed by four sets of combinations of three.
Choose the set in which the statements are logically related.
DIRECTIONS: In each of the following questions one interrogative statements followed by two arguments,
one beginning with `yes' and the other with `no'. Mark:
(a), if only argument 1 is forceful
(b), if only argument 2 is forceful
(c), if both 1 and 2 are forceful
(d), if neither 1 nor 2 is forceful.
15. Should there be reservation of jobs for women in India?
1. Yes, because women have been living under social constraints.
2. No, because such a step would make them helpless dependents.
DIRECTIONS: In the following questions, two statements are followed by two inferences. Write
(a), if only inference 1 follows
(b), if only inference 2 follows
(c), if both of them follow
(d), if neither of them follows.
19. All passengers are messengers. All messengers are intruders. So
1. All intruders are passengers.
2. Some passengers are intruders.
20. All actors are stars. All stars planets. so
1. All actors are planets.
2. All planets are actors.
21. No student is prudent. All students are young. So
1. No prudent is young.
2. No young is prudent.
Pankaj Gandhi’s Academy/Logical Reasoning 70
22. All cows are buffaloes. All goats are buffaloes. So
1. All cows are goats.
2. All goats are cows.
24. All canals are rivers. The rivers irrigate the land of this state. So
1. All the canals irrigate the land of this state.
2. Some canals irrigate the land of this state.
25. All medicines are bitter. The doctor has advised him to take medicine. So
1. The doctor has advised him to take bitter things.
2. The doctor has advised him not to take sweet things.
26. All boys are toys. All toys are generally liked by the children. So
1. All boys are generally liked by the children.
2. Some children generally like boys.
28. All lords are peers. All peers are members of the upper house. So
1. All lords are members of the upper house.
2. Some members of the upper house are lords.
DIRECTIONS: In the following questions study the given information and mark one of the following
answer choices:
(a), Statement 1 is implied.
(b), Statement 2 is implied.
(c), Both the statements are implied
(d), Both the statements are not implied.
DIRECTIONS: In each of the following questions some statements are followed by two inferences. Point
out if:
(a), Conclusion 1 follows.
(b), Conclusion 2 follows.
(c), Conclusion 1 and 2 follow.
(d), Neither 1 nor 2 follows.
34. Statements : A welfare state must create conditions which ensure social justice. It
must eliminate the inequalities created by capitalism.
35. Statements : The population below the poverty line is computed on the basis of
minimum daily calorie requirement of food and actual consumption.
Forty-eight percent of India's population lives below the poverty line.
DIRECTIONS: Pick out from the answer-choices the premiss which will complete the argument without
imparting any fallacy to it.
37. Books are good friends because they keep our company.
(a) All good friends keep our company.
(b) All who keep company are good friends.
(c) Only those who keep company are good friends.
(d) Some who keep company are good friends.
38. Beggars are unpatriotic because they bring dishonour to the country.
(a) All unpatriotic bring dishonour to the country.
(b) All who bring dishonour to the country are unpatriotic.
(c) Some unpatriotic bring dishonour to the country.
(d) None of these.
By contemporary international standards, the Deskpro 1000, launched by Compaq India, is a stripped
down PC, lacking a CD-ROM drive as well as multimedia features. The company intended the 1000
to be an entry-level commercial desktop with current technology – but, crucially, at a price-point lower
than that of its existing models.
41. Compaq India‘s price-war strategy has paid off in the Indian market.
42. Price wars have often been used as a major weapon in the highly competitive PC market.
43. Compaq India had to use an effective strategy to make a niche for itself in the Indian market.
44. Compaq India‘s success depends on the Indian consumer‘s willingness to trade international
standards for price.
45. Compaq India‘s PC prices have a major drawback: they don‘t convince.
DIRECTIONS: For questions 46-50: Read the following passage and answer the questions that
follow. Mark
(a) if the statement is a logical conclusion to the passage.
(b) if the statement is a contradiction to the intent of the passage.
(c) if the statement is a far-fetched conclusion.
(d) if the statement is irrelevant to the passage.
If there is one word that sums up the economics of the BJP, it is frontspeak – the indistinct language
of compromise. Clarity, certainty and consistency is what business expected from the BJP. Instead, it
is now getting accustomed to a reforms rhetoric that is meant to cement a coalition rather to catalyse
change.
46. The direction of the BJP‘s economic policy seems vague and accommodating.
47. The BJP has carved out a well-defined economic policy with especial attention paid to export-
import.
48. The BJP‘s pusillanimous attitude to its coalition partners will bring about its ruin before the five-
year term is over.
49. Corporate performance has improved sharply and better output is expected over the next three
years.
50. The BJP‘s agenda of economic nationalism has borne fruit in its coalition politics.
Exercise 4(B)
1. With the computer market booming, the demand for electronic typewriters has steadily
declined over the last 15 years. So industry analysts were undoubtedly surprised to see a 30%
increase in total sales of electronic typewriters last year. In anticipation of an increase in its
sales, the Wordmaster Typewriter Company is planning to manufacture more electronic
typewriters for the coming year.
Which of the following statements, if true, would strongly suggest that the plan will fail?
(a) According to a consumer survey, majority of those surveyed showed a preference for
products of Wordmaster‘s major competitor.
(b) In order to produce more typewriters, Wordmaster will have to invest a sizeable amount of
money in new equipment and in a new factory.
(c) The typewriter industry as a whole recorded a jump in sales last year, but specifically in
electronic typewriters, which are the closest things in the market to a computer word
processor.
(d) Most consumers find computer word processors user-friendlier than electronic typewriters
and would prefer to buy these instead.
(e) All of the above
3. Referring to the passage above, which of the following, if true, most strongly supports the view
expressed in the passage?
(a) Acid rain caused by factories in Sweden has polluted lakes in places like Greenland.
(b) Soviet leaders had refused the western media access to the accident site in Chernobyl,
thus preventing complete information about its extent.
(c) Neighbouring states in the United States are frequently unable to agree on pollution control
norms to be implemented.
(d) Most countries have unilaterally taken steps to curb pollutants like non-biodegradable
plastic bags.
(e) Many housing societies have enforced practical and workable rules preventing the
residents from disposing waste in a haphazard manner.
4. Today the possibility of breeding an intelligent human race is no longer such an absurd theory.
Just as the sagas of the Tiahuanaco and the inscription on the pediment of the Gate of the Sun
talk about a space-ship which landed the Great Mother on earth so that she could bear
children, the old religious scripts too, never tire of saying that 'God' created men in his own
image. With the theory of a visit to our earth by unknown intelligences from the cosmos we
could postulate that today we are similarly constituted to those fabulous unidentified beings.
The above argument rests on the assumption that:
(a) The texts referred to are representative of all religious texts.
(b) There is extra-terrestrial life in the universe.
6. 80% of the population are his die-hard fans. They do not want to see a great actor stoop to
mere farce and commercials. Says the actor-director Naseer Nullah, "his fans are feeling let
down. They feel he owes them something."
The author makes her point by:
(a) Expressing an opinion and backing it with a supportive argument.
(b) Presenting an amalgam of fact and judgement.
(c) Examining a causal relationship.
(d) Stating a fact and substantiating it with expert opinion.
(e) Quoting statistics
7. Joyce, James and Woolf all wrote novels that focussed more on the thoughts and feelings of
characters than on external happenings and the story. It is their effort that the modern novel
owes its genesis to.
Which of the following statements follows a similar logical sequence?
(a) The bone marrow is unable to produce any RBCs. It seems clear that the patient will have to
undergo treatment for Thalessemia.
(b) Europe has always been the focus of the war culture. With role models like Napoleon,
Alexander and Hitler contemporary warfare has learned all its basic techniques from the
strategies of these men.
(c) The precedent set by the ruling in the Aruna Shanbaug case will ensure that the defendant
will not be given the life-imprisonment sentence for his crime.
(d) The DNA obtained from the Hendeca planet, if injected into a human ovum, might lead to
the creation of a monster. On the basis of theories advanced in various journals I
recommend that this kind of experimentation be avoided at all costs.
(e) The teaching of the Bible says that only Noah and his family, by dint of being good people,
escaped the deluge, and since then all events that happened before the deluge is called
antediluvian or very old.
8. Kerala is 100 percent literate, but experts feel that the stress on universalisation has affected
quality. Barely 30% of the students manage to pass. So the state government has decided to
change the system radically. Hence, teachers were told that learning should be a fun-filled
activity. Stodgy textbooks were replaced with colourful ones. Subjects were transformed into
activities where children were to play games to learn mathematics or history or any other
subject. The idea was to improve the quality of the primary education system in the state.
Which of the following if true would seriously weaken the argument?
(a) Many schools, which have followed the non-formal approach of education, have never
attained a very high passing percentage.
(b) Education is a very important part of a child‘s growth and hence has to be very systematic
and tough and not frivolous.
(c) It may happen that games overtake studies and rather than learn, students may get an
overdose of fun and frolic.
(d) Nothing in life can be achieved easily and hence to ensure a good education, games
should be completely prohibited.
(e) In today‘s competitive world, it is hazardous to waste time and energy on a new and
experimental approach to studies.
10. ―On the whole‖, Ms. Dennis remarked, ―engineering students are lazier than they used to be. I
know because fewer and fewer of my students regularly do the work they are assigned.‖
Which of the following is assumed in Ms. Dennis‘ reasoning?
(a) Plenty of people besides engineering students do not work as hard as they should.
(b) Ms. Dennis does not take into account the valid excuses the students may have for not
doing their work.
(c) Whether the students do the work they are assigned is an indication of laziness.
(d) Engineering students should work harder than the students in any other field.
(e) Ms. Dennis may not be a very effective teacher, and therefore the students may not be
doing their work.
11. The extent of the degradation is such that in a country where poverty and unemployment are
pressing problems more and more people see the attrition of the environment as the gravest
threat.
Which of the following statements most strengthens the above argument?
(a) As many as a third of people surveyed said that economic growth should be given
importance.
(b) Greater awareness has led to people seeking stricter laws stiffer penalties and higher taxes
to save the agricultural economy.
(c) A survey conducted by ORG-MARG in the battle between unchecked economic progress
and saving the environment shows that people are speaking up for preserving habitats.
(d) Car owners are willing to pay 10% more for household products and non-conventional
energy that are environment friendly.
(e) All of the above
12. When the aim is sure and swift any target becomes just another milestone. When people
harmonize their energies, you can be sure that all your efforts will converge towards a single
focussed goal.
Which of the following is a logical conclusion?
(a) Milestones are easily achievable.
(b) Unity is strength.
(c) Working together implies that everybody has a unilateral approach to the task at hand.
(d) Target achievement is simpler and more effective when many heads get together.
(e) None of the above
13. "The village panchayats" according to Professor Indirani Sridharan "represented the interests
of only one group of people, i.e., the rural rich. These members were selected rather than
elected by the masses because they were rich."
Which of the above statements follow a similar pattern of argument?
(a) The cricket team was selected by a board comprising entirely of members who represented
the state of Maharashtra. Therefore, 10 out of the chosen 15 team members to go to South
Africa played for Bombay
14. Kangal Das of the Buffalo Party says ―The Dinosaur people‘s party are such liars that even
their lies sound like truth…The court thinks whatever the centre says is right and we are
wrong.‖
What would strengthen the assumption if the above statement were considered true?
(a) Kangal Das belongs to the opposition party that has a history of flinging accusations at the
ruling party and is merely carrying out routine procedure.
(b) The court is always right so his accusations are baseless.
(c) Kangal Das has a personal grudge against the Dinosaur people‘s party due to which their
truth sounds like lies to him
(d) Kangal Das has an impeccable reputation and he has never made irresponsible statements
therefore he could not have misrepresented facts, a fact later accentuated by the courts
ruling in favour of the opposition.
(e) None of the above
15. We could have a perfect world if we controlled everyone's behaviour. But then, we would have
a completely perfect world with no people and no misbehaviour at all.
Which of the following is the most basic assumption that the above statement works on?
(a) That it is possible to eliminate people.
(b) That it is quite possible to control behaviour.
(c) That controlled behaviour is related to a perfect world.
(d) Behaviour is an inevitable outcome of being human
(e) All of the above
16. If the above is considered true which of the following statements is an inference that may be
drawn from the above statement?
(a) If people‘s behaviour were not controlled the world would not be a perfect place.
(b) Behaviour is bad and it must be controlled.
(c) We need to make judgements about behaviour.
(d) All of (a), (b) and (c)
(e) None of (a), (b) and (c)
17. What the English and the vernacular press both share, is a lack of discrimination. They dare
not trust their own judgements since they've no critical standards. For those who speak English
and are westernised and that includes many of our Indian critics, the sun rises and sets in the
West . If the West approves it, it must be good. In the regional language safety and security
often lie at the other extreme in parochialism and the bogey of safeguarding our culture.
Which of the following would the author be most likely to agree with?
(a) The press goes by all that is trumpeted by the West.
(b) The sun does not rise in the East.
(c) The only way to remedy the situation would be for Indian writers to hone their critical skills.
(d) The only way we can regain the stature and standards of the Bhakti poets is to be tough
with ourselves.
(e) None of the above
18. A leading business magazine says, "Before we check out the story we check out the reporter.
We ask him if the facts have been double checked. We make sure that the company
mentioned in the article has confirmed the news and find out if anything has been taken out of
context and is likely to be misinterpreted."
Which of the following may be inferred if the above data is considered true?
77 Pankaj Gandhi’s Academy/Logical Reasoning
(a) The reporters who are hired come with references, which have to be impeccable.
(b) The editor verifies the news and sees to it that nothing that may have dubious status is
printed in order to maintain the respectability and reputation of the paper
(c) Before printing, the authenticity of the story is ascertained and the reporter is questioned as
to its accuracy.
(d) All stories reviewed are printed after double-checking.
(e) All of the above
19. Says Bhaumik "soon the consumer will be able to choose between legal but expensive imports
and cheap but smuggled foreign goods.‖ Some retailers like Jimmy Dogwallah of a South Pune
supermarket stocking imported goods differ. Even with the duties, prices of legal imports wont
be that high. The gray market is bound to suffer. Retailers will opt for hassle free dealings,
good deals and incentives"
Which of the following would weaken the argument put forth by Jimmy Dogwallah?
(a) Retailers are looking for a larger consumer base as that translates into higher profits.
(b) The gray market is very well organized and has a large number of loyal clientele that is fully
satisfied with the prices of all products stocked.
(c) Retailers like Dogwallah cater to the needs of a chunk of consumers who belong to the
upper classes.
(d) The prices of legal imports are higher as compared to those of their Indian counterparts.
(e) None of the above
Exercise 4(C)
In January of 1978, William Sanford assumed the presidency of the Hogarth Information Systems
division of the Hogarth Corporation. The first issue the new president faced was whether the
Company was willing or able to make the substantial investment necessary to be able to keep up with
the other four major manufacturers of computer in the development of the next generation of
computer equipment. To this question, Sanford does not give a yes or no answer. "Let me put it this
way." he says. "I believe that computer business is beginning to be more mature, with predictable
trends. --- There will be no more great shakeouts. The people who are in the business now will
remain and be profitable.--- There will be no more revolutionary developments, only evolutionary
ones."
But many computer executives think the rapid changes in technology that have occurred over the
past 20 years are not over. "I believe the computer business is at the very beginning of its Youth,"
says Gorald G miller, president of one of Hogarth's largest competitors.
The shape of the future is crucial to Hogarth because the computer division's low profitability rank
among the five major companies in the field raises questions about its staying power. Hogarth's
computer business ranks last among the five in pretax return on assets, at 1.4 per cent, compared to
another firm's 8.3 percent and the industry leader's stand out 27.2 percent. Thus one of the reasons
some managers who have left Hogarth believe The company will ultimately sell out of the computer
business is that it simply cannot afford to stay the course.
Sanford, however, has stated that he has plans for expansion into ellied areas of the computer
industry which will provide a revenue base independent of competition from Hagarth's major rivals.
His projects under this new program include the acquisition of Dynoterm Corporation, an "intelligent"
terminal manufacturer; Synergistic Incorporated, a California manufacturer of integrated circuits; and
Gamble Associates, a software company which writes programs for customers under specific
contracts. Sanford believes that these three companies represent an integration of products that will
benefit Hogarth as well as its customers. The Dynoterm terminals will interface with Hogarth's main
frame computers, thus providing a flexibility not matched elsewhere in the industry. Synergistics'
integrated circuit modules will be used in the manufacture of all future Hogarth computers. Gamble
Associates will not program exclusively for Hogarth customers, but the record profits of the company
will contribute much needed cash flow in an industry noted for slow cash returns on sales, and the
captive subsidiary will provide credibility to those customers who fear that Hogarth will not be able to
supply quality programs as well as machinery.
There are however, several pitfalls in the way of Sanford's master plan. The company does not have
the cash or cash-raising recourses to purchase all three companies in one year. Therefore, it is
crucial that Sanford select the right subsidiary to purchase first.
In preparing the evaluation of the alternative purchases, Sanford first looked at the costs of
acquisition. All there companies could be purchased for cash as follows:
Hogarth could not finance the purchases of either Dynoterm or Snyerigistics from corporate cash
reserves but, with some belt-tightening, could raise the cash for the gamble purchase. On the other
hand, however, it would not be too difficult to arrange a stock swap with the two larger compani es,
since, Hogarth's stock had been holding steadily at 54 for several months and had not dropped below
50 for the past fiscal year.
The immediate advantage of the initial purchase of Dynoterm would be the expansion of the product
line. Hogarth has long been sensitive to the fact that many of its competitors offer a variety of
Synergistic manufactures the circuitry that goes into computers. Their competition is Indiana
Instruments, Vidorola, and Duotel. While smaller than their competition, Synergistic manufacturers a
quality product and has very stable 15 percent of the market. Synergistics' customers include
Hogarth, as well as the other major manufacturers, and several smaller mainframe computer
companies- almost all, in fact, except for the industry leader which is vertically integrated.
One of the advantages of owing Synergistics is the assurance of a steady supply of raw materials for
Hogarth's computers. As with the case of Dynoterm, the research and development department feels
strongly that the development of a separate division is un warranted in the face of Synergistics' price.
Although Indiana Instruments and Vidorola represent stable and reliable vendors of integrated circuit
chips, many Hogarth manufacturing executives feel safer with a captive supply of raw materials.
Synergistics' pretax profit was $ 10 million in 1978, with a price-profit ratio 4:5, somewhat better than
the 5 to 1 ratio of Dynoterm. In the case of Synergistics, however, approximately $ 3 million of the
total profits of $ 10 million is due to sales to Hogarth, this diluting the attractiveness of the profit
picture and price-profit ratio considerably, assuming a transfer of ownership is made. On the other
hand, the remaining profits can be expected to continue, and the production of integrated circuit
chips represents the type of product diversity Sanford feels Hogarth needs.
The evaluation of the gamble purchase is more subtle. Although the price tag of the company is
considerably less than that of the other two possibilities. Gamble, as a service company, possesses
no hard assets. Both Synergistic and Dynoterm have large plant facilities and inventories, while
Gamble's assets consist of nothing more than the skill and experience of its staff of programmers,
and the goodwill generated by the company over the ten years of its existence. Thus, while the price
of Gamble at $ 9 million is only three times the net profits of $ 3 million many Hogarth executives are
nervous about the fact that $ 9 million purchase would secure only intangible assets. Furthermore the
development of an internal programming staff that could provide all the functions of a gamble-type
subsidiary is not considered a difficult task given a $ 9 million budget. Once again, however, the
hypothetical internal staff would only program for Hogarth's line, while Gamble could be expected to
service the customers of all computer manufacturers, thus generating cash flow as well as providing
Hogarth will the full service diversified company image it desires. Also, the $ 3 million profit would be
totally captured to be paid for separately by the customers. The cash flow from programming would
be much more immediate than that generated by selling hardware, as the latter generally involves
60-month lease plans, tieing up the manufacturer's money for five years, while programming
revenues are collected immediately upon sale.
Furthermore, Sanford feels that because of the low price of Gamble, as well as the rapid return on
investment. Hogarth would soon be in a position to make the second purchase. The lack of tangible
assets, he feels, is not relevant consideration. For these reasons, he decided to make the Gamble
purchase first.
The following questions consist of items related to the preceding passage above. Consider each item
separately in terms of the passage and on the answer sheet blacken space.
(a) If the item is Major Objective in making the decision; that is, one of the outcomes or results
sought by the decision maker;
DIRECTIONS: In the following questions one statement is followed by two possible implications.
Study the and mark one of the following answer choices:
(a), Statement 1 is implied.
(b), Statement 2 is implied.
(c), Both the statements are implied
(d), Both the statements are not implied.
DIRECTIONS: In the following questions a conclusion is followed by two statements which give us
data for the conclusion. Study that data and the conclusion and mark.
(a), If statement 1 alone can bring us to the conclusion.
(b), If statement 2 alone can bring us to the conclusion.
(c), If both the statements 1 and 2 taken together can bring us to the conclusion.
(d), If the statements taken separately or together cannot bring us to the conclusion.
8. Conclusion : The government should reject the demand for a tribal homeland.
DIRECTIONS: Assuming the statement to be false, point out if the inference is (a) True, (b) False or
(c) Uncertain.
DIRECTIONS: Each question contains six statements followed by four sets of combinations of three.
Choose the set in which the statements are logically related.
DIRECTIONS: In each of the following questions one interrogative statement is followed by two
arguments, one beginning with `yes' and the other with `no'. Mark:
(a), if only argument 1 is forceful
(b), if only argument 2 is forceful
(c), if both 1 and 2 are forceful
(d), if neither 1 nor 2 is forceful.
24. Is it possible for India to maintain good relations with her neighbours?
1. Yes, because India is a bigger country and these neighbouring countries depend on her.
2. No, because India's neighbouring countries are instigated by the enemies of India to
create problems for her thus causing hindrance for good relations.
DIRECTIONS: From the alternatives, choose the one which correctly classifies the four sentences as
F : Fact :if it relates to a known matter of direct observation, or an existing
reality or something known to be true.
J : Judgement: if it is an opinion or an estimate or anticipation of common sense or intention,
I : Inference : its is a logical conclusion or deduction of something, based on the
knowledge of facts.
28. 1. We will have to divert our financial resources to the power sector.
2. Projects requiring the use of scarce materials have to be kept to the minimum.
3. The power sector of the country is poised for a giant leap forward.
4. At the same time, other energy sources will have to be exploited.
(a) JJII (b) JJJJ (c) JJJI (d) JJFJ
DIRECTIONS: In each of the following questions some statements are followed by two inferences.
Point out if:
(a), Conclusion 1 follows.
(b), Conclusion 2 follows.
(c), Conclusion 1 and 2 follow.
(d), Neither 1 nor 2 follows.
29. Statements : National integration cannot be achieved as there are contrasts between
the rich and the poor and different religious groups.
30. Statements : 1. The policy of liberalisation will make the rich richer and the poor
poorer. 2. The disparity between the rich and poor will widen.
DIRECTIONS: In the following questions, two statements are followed by two inferences. Write
(a), if only inference 1 follows
(b), if only inference 2 follows
(c), if both of them follow
(d), if neither of them follows.
31. All rings are bracelets. All rings are diamonds. So
1. Some bracelets are diamonds.
2. Some diamonds are bracelets.
32. Some locks are keys. All keys are hooks. So
1. Some locks are hooks.
2. Some hooks are locks.
33. All rabbits are cats. All cats are jackals. So
1. All rabbits are jackals.
2. Some jackals are rabbits.
34. All cupboards are purses. All purses are costly. So
1. All cupboards are costly.
2. Some costly things are cupboards.
35. All maps are caps. All caps are traps. So
1. All maps are traps.
2. All traps are maps.
42. If the price of onions increase, we will not have murg-do-pyaza. If the price of onions
decrease, we will have kanda bhajji. If we don‘t have kanda bhajji, then we will not have tak
either. If we have tak, we will have it with murg-do-pyaza.
Which of the following is true if we have murg-do-pyaza?
(a) We will have tak.
(b) We will not have onions.
(c) We will have kanda bhajji.
(d) The price of onions has not increased.
43. If, in a television programme, Javed Jafferi plays the role of Hip-Hop Hingorani, he does not
play the role of Future Futardo. If he is not playing the role of Suxy, he is playing the role of
his brother, Adrenaline Alberto, If he plays Ramnathan Romeo‘s role, he also gets into drag
and plays Jayanti Juliet‘s role. And he plays Jayanti Juliet‘s role only when he does not play
Hip-Hop Hingorani.
Therefore, if he plays the role of Future Futardo…
Pankaj Gandhi’s Academy/Logical Reasoning 86
(a) he plays the role of Adrenaline Alberto.
(b) he does not play Jayanti Juliet.
(c) he does not play the role of Hip-Hop Hingorani
(d) he plays Ramnathan Romeo.
44. If the clock is an Omega, it attracts the mice. If the mice are naughty, they will run up the
clock. If the clock strikes one, the other mice will get away with minor injuries. If the clock is
not an Omega, the mice will eat cheese instead.
Why have the mice not run up the clock?
(a) Because the clock was not an Omega.
(b) Because the mice are not naughty.
(c) Because the clock struck one of them.
(d) None of the above.
45. If Gregory Peck does not act in Roman Holiday, Audrey Hepburn will act in My Fair Lady. If
Gregory Peck acts in The Snows of Kilimanjaro, Ingrid Bergman acts alongside him. If Cary
Grant acts in Meet John Doe, then Gregory Peck will act in The Guns of Navarone, but not
Ingrid Bergman.
What would happen if Ingrid Bergman does not act ?
(a) Gregory Peck will not act in The Snows of Kilimanjaro.
(b) Gregory Peck will not do The Guns of Navarone.
(c) Cary Grant will not act in Meet John Doe.
(d) Gregory Peck will act in Roman Holiday.
DIRECTIONS: Given below is a passage followed by several inferences. Examine the inferences
separately in the context of the passage, and determine whether they are true or false. Mark
(a) if the inference is definitely true, i.e. it properly follows from the statement of facts given.
(b) if the inference is probably true, though not definitely true, in the light of statement of facts given.
(c) if the inference is uncertain, i.e. data is insufficient to decide whether the inference is true or
false.
(d) if the inference is probably false, though not definitely false, in the light of statement of facts
given.
(e) if the inference is definitely false, i.e. it cannot possible be from the statement of facts given.
Critics who level the charge of ‗government by the judiciary‘ overlook the important fact that in a majority of
the cases the Court is giving effect to the mandate of the Parliament expressed in the laws enacted by it
but which laws are not implemented at all by the Executive. These critics forget that it is the notorious
tardiness of the legislatures, and the inertia, almost bordering on callousness, of the executive branch
which provide the occasion for or rather compel judicial intervention. For example, when constitutional
obligations and statutory provisions relating to the employment of children in factories and hazardous
occupations are blatantly flouted on account of executive inaction, and the consequent gross violations of
the human rights of the children are brought to the notice of the court, the court cannot fold its hands and
refuse to act.
46. The courts do not have any right to direct the government as to its duties.
47. The judiciary has intervened in the child labour and employment of children in factories case
48. There are people who are against judicial activism in executive affairs.
49. There are no anti-child labour laws in the constitution of the country.
50. The judiciary has consciously decided to play an important part in ruling the country.
Exercise 5(B)
1. A Government survey released today shows that almost 80% of the people who use the
national airline are satisfied with the service they get. The market research people stood
outside the major airline terminus in the capital and asked people leaving the terminal, ―Do you
have any complaint about the flight you just got off?‖ Only 20% responded ―yes!‖
Which of the following, if true, would most undermine the conclusion of the argument above?
(a) Almost 60% of the terminal capacity was used by all the other private airline services, not
just the national one.
(b) One percent of the people approached by the interviewers refused to respond to the
question.
(c) The interviewers began their inquiry just after the passengers were discharged from a flight
that was 40 minutes late.
(d) The interviewers were able to speak to just 50% of the people leaving the terminal, but
those people were selected at random.
(e) For six months following the day of the inquiry, no official complaints were filed against the
national airline by any passenger.
2. Professor Moriarty did not commit the murder in the library with the bludgeon, therefore Ms.
Peel committed the murder in the lounge with the stiletto.
The argument above depends upon which of the following assumptions?
I. The murder was committed either with the bludgeon or with the stiletto.
II. The murder was committed either in the library or the lounge.
III. The murder was committed either by Professor Moriarty or Ms. Peel.
IV. The murder was either committed by Professor Moriarty in the library with the bludgeon or
by Ms. Peel in the lounge with the stiletto.
(a) I only (b) IV only (c) I & III only (d) I, II, & III only (e) I, II, III & IV
3. The laws governing the preservation of forest cover and trees unfairly impinge on the rights of
the owners to treat their property as they see fit. In some cases, the owners of old banyan and
acacia trees have complained that the government rules have forbidden them from clearing the
land or even to build on them aesthetically and develop them as real estate, thus making them
more valuable financially.
Which of the following statements, if true, seriously weakens the author‘s argument?
(a) Altering the landscape of a property usually does little to enhance its aesthetic or economic
value.
(b) In traditional legal doctrine, ownership of a property implies the right to alter it at will.
(c) Only trees over 100 years old are normally affected by landmark preservation laws.
(d) Landmark designations must be approved by a local regulatory body before taking effect.
(e) Trees, old or new, represent an ecological heritage which the whole community has
legitimate stake in preserving.
4. The cleaning and restoration of the frescoes in the Ajanta and Ellora caves were undertaken
by some of the world‘s best art restorers. But the results have provoked a storm of controversy.
Most modern viewers, it seems, had become accustomed to seeing the frescoes with their
colours dulled by yellowing glue and varnish, the contours obscured by centuries accumulation
of grime.
The passage implies that the Ajanta and Ellora frescoes
(a) have been the subject of controversy due to their artistic merit.
(b) have been obscured by dirt during the recent process of restoration.
(c) suffered until recently from callous neglect on part of the government.
(d) should not have been cleaned and restored without more careful planning.
(e) were originally much brighter and more vivid than most people realize.
6. Computer education fees have risen sharply over the past decade and will continue to rise in
the future. Yet, the college and educational institutions have the means to reverse this trend.
They get large amounts of money as endowments from private donors. They should use these
grants to reduce tuition costs.
Which of the following, if true, would most weaken the argument above?
(a) Most students are able to meet their college fee requirements, even if they have to take
loans to do so.
(b) The costs incurred by colleges in teaching students are greater than the fee students pay.
(c) Donors who make the endowments usually restrict the areas in which the funds are to be
utilized by the college.
(d) More college students work part time nowadays, which helps them meet the rising costs of
education.
(e) Non-tuition costs of college education have increased more rapidly than the tuition costs.
7. My neighbour, upon my asking him how the snowblower I lent him had been broken, replied: ―I
didn‘t borrow your snowblower; and if I did, I‘m sure it was not broken when I returned it; and if
it was broken when I returned it, it was broken when I borrowed it.‖
Which of the following is most similar to the argument above?
(a) Forensic expert: ―An examination of the body shows three bullet wounds to the head,
labelled A, B and C. Any one could have caused death. If it wasn‘t A, then it was either B or
C; and if it wasn‘t B, it was C.
(b) Child: ―I never tell a lie; and if I ever did tell a lie, I‘m not lying now. And if I am lying now,
you‘ll never find out.
(c) Doctor: ―The patient was admitted complaining of nausea, dizziness and stomach cramps,
the classic signs of food poisoning. During dinner, the shrimp cocktail, baked chicken or ice
cream he had must have caused it.
(d) Attorney: ―My client wasn‘t at the scene of the crime; and if he was, he didn‘t do it; and if he
did it, he was insane at the time.
(e) Stock analyst: ―The sudden drop in the market was caused by three factors—the world cup
debacle of the national team, the attack by neighbours in the northern part of the country
and the fall of the government. In the absence of any one of these factors, the drop would
have been less severe.
8. The existence of flying saucers and UFOs has been shown to be illusory. Sceptical
researchers have demonstrated that a number of photographs reportedly showing flying
saucers are either crude forgeries or misinterpreted images of earthy objects such as clouds,
birds, weather balloons, or small private planes.
If the photographs are accurately described in the passage, which of the following is the best
argument against the conclusion drawn?
(a) Not all UFOs can be conclusively proved to be manmade objects.
(b) The fact that a number of photographs are fake does not generally disprove the
phenomenon.
(c) Some of those who have witnessed flying saucers have no apparent motive for lying.
(d) Given the size and complexity of the universe, it seems unreasonable to assume that life
exists only on Earth.
(e) Researchers who are sceptical about flying saucers invariably bring their own biases and
preconceptions to work.
89 Pankaj Gandhi’s Academy/Logical Reasoning
9. The scientific temper should be inculcated in students from the very beginning. It will make
sure that they have a healthy scepticism towards traditional and religious practices and
teachings that they might otherwise accept without question.
10. Cardinal Richloo: The plays and short stories attributed to Balzac, a poorly educated plebian,
were in fact written by Louis IV, the Sun King, who had the intelligence, erudition and ability to
produce such excellent works of art.
Napolean Bonafide: Highly unlikely, in fact, impossible! If Louis IV had written great stories
such as Le Rue Noir and L’ombre de la Femme, he would have quickly become known as one
of the greatest French authors in history. But he has no such reputation.
Napolean Bonafide‘s argument assumes that it is improbable that
(a) Louis IV‘s authorship of the plays and stories would have been kept a secret.
(b) an uneducated person like Balzac could have written stories like Le Rue Noir and L’ombre
de la Femme.
(c) Louis IV had the artistic gifts needed to achieve literary greatness.
(d) education and talent in a creative person necessarily go hand in hand.
(e) a patrician in the times of Louis IV could not have had time or opportunity to achieve greatness
as an author.
11. Although there is a lull in the two-wheeler industry, as the economy improves, the industry will
improve. Hero Honda has a well-defined focus that will help it grow at a faster rate.
Which of the following arguments strengthens the above statements?
(a) The industry is a dynamic one that has always bounced back.
(b) Hero Honda is a company that has all the required resources to help it tide over the crisis.
(c) Most manufacturing companies have bounced back as past history has proved.
(d) An upward trend is predicted for companies in the export industry.
(e) None of the above
12. The company does not plan to push products despite the fact that a product firm is more easily
saleable than a service firm. We have chosen to be a service firm. A good rival could drive a
product out of the market
Which of the following statements follows a similar pattern of argument
(a) The company is aware of the cards it holds… But the big boost to revenues is expected
from network expansion. Despite a healthy CAGR, S&B bank remains heavily dependent on
its home state for its deposit base - only 126 of its 426 branches are outside the state yet it
is definitely the leader within the state.
(b) The company is essentially a sick unit that is being used by the owners to throw a rug over
nefarious activities .The investigation has resulted in a tremendous victory for the
government which is losing millions of rupees every year by such dispossession
(c) While its dependence on its parent has been reducing MET is gradually moving to create
mission-critical applications from BT. For instance it has created a billing and customer
care product that handles 75% of its external revenues.
(d) Post 2005, once patent laws come into force, Morepen will not be able to reverse engineer
molecules and sell in unregulated markets while preparing for a push into regulated
markets like the US when patents expire.
(e) All of the above
14. Since the universe is infinite and so is the outer space, our conventional units of measurement
are not suited to measure the astronomical distances between the earth and the sun, the moon
or other planets or distances between other heavenly bodies. Therefore different units of
measurement such as the light year and the astronomical unit have been evolved to measure
distances in the space
Which of the following is an inference that can be drawn from the above statement?
(a) Conventional units of measurement are finite while the universe is infinite.
(b) Our conventional units of measurement are no longer valid in today‘s global environment.
(c) The units of measurement used so far have been found faulty.
(d) As the conventional units of measurement have been found inadequate new units have
been evolved.
(e) None of the above
15. Says an ABCK member, referring to her pulling down of the Bechara Bhai government "Yamma
gave up power to fight corruption. Today she stands vindicated, the NPA has compromised on
defense. "
Which of the following undermines the author‘s stance in the above statement?
(a) Yamma has been proved to be an incarnation of the just and rightful leader of the masses.
(b) Yamma's fight against corruption still continues.
(c) Mistakes similar to those Yamma made were unearthed and therefore she stands
vindicated.
(d) Yamma's political life is tainted with corruption charges that are not very different from that
of those she is against.
(e) All of the above
16. The human brain is very fertile and is never at rest. It results in intellectual output in the form of
inventions, trademarks and the like. When it assumes a form of an entity apparently intangible
it takes the form of property. Which is aptly called intellectual property. In order to ensure
further progress of civilisation and give impetus to the creative people a sort of legal protection
is always necessary.
Which of the following is an assumption inherent in the above argument?
(a) Purposeful working of the human brain is constantly creating this intellectual property.
(b) The legal protection should provide the individual who created this intellectual property with
exclusive rights to use the invention for which patent rights are granted
(c) The human brain is transformed into a tangible entity.
(d) Intellectual property can ensure further progress of civilisation.
(e) None of the above
17. US officials like sneering at how South Asia is "not even on our radar screen." But the truth,
when seen from a South Asian perspective appears to be somewhat different. Seen from this
vantagepoint it appears that the US has always been involved in Kashmir and so far the
involvement has come mainly in the form of Pakistan's cause
Inferring from the passage which of the following is the author most likely to agree with?
(a) The US actually perceives South Asia as a threat and for this reason use their superior
attitude as a smokescreen.
(b) The US has a hand in backing insurgency in Kashmir.
(c) In spite of the supercilious attitude, the US has an interest in South Asian affairs.
91 Pankaj Gandhi’s Academy/Logical Reasoning
(d) America has always backed Pakistan in its acts against India.
(e) None of the above
18. As an artist Tolstoy is classical in the same way that Homer, Chaucer and Spenser are; his art
here still moves to the instinctive rhythms of 'joy after woe and woe after gladness'. Tolstoy
was increasingly preoccupied with questions of religion and belief and Tolstoyism dominated
his life. Which of the following may be inferred from the above statements?
(a) The classical art of Homer, Spenser and Shakespeare deeply influenced Tolstoy, who
became increasingly preoccupied with Tolstoyism.
(b) Chaucer and Spenser were reputed adherents of the school of thought that came to be
known as Tolstoyism.
(c) Tolstoy's art is classical in its similarity of rhythms and in his preoccupation with questions
of religion and belief.
(d) Tolstoy's art is classical because he was involved like Chaucer and Spenser in their later
life with questions of religion.
(e) None of the above
19. (i) Since oil has been struck in the foothills of mountain ranges like the Andes, the Alps, and
the Rockies, ONGC is confident of discovering reserves in areas like the foothills of the
Himalayas.
(ii) A combination of the recession and Maharashtra government directive in May1997 which
discouraged students from using workbooks at school, were responsible. Worse the price
of paper, which constituted 47% of costs in 1997 have started edging upwards after
remaining constant for 3 years.
(iii) NIIT's eCommerce solutions ensure that while you gain new customers, your existing
ones keep coming back. It‘s the best way to keep your business growing exponentially.
(iv) NIIT can ensure that you steal a march over competition with e-Business solutions that
are up and running within 90 days. Because on the WWW the only way to survive is by
racing ahead.
Which two of the above statements follow a similar pattern of argument?
(a) i & ii (b) iii & ii (c) iv & iii (d) i & iv (e) ii & iv
20. Remarkable is the person who adheres to the straight and the narrow path of integrity when
the riches that entice are so awesome and when meticulous honesty entails a life of hardship
and consumerist deprivation for one's family.
Which of the following may be inferred from the above statement?
(a) A person who swerves from the straight and narrow path of integrity will find wealth dogging
him.
(b) It is necessary to lie so that your family does not suffer from a life of hardship and
deprivation.
(c) It is difficult to be honest and have integrity in a world of riches and consumerism.
(d) There are some people who are remarkable because they are honest.
(e) All of the above
Exercise 5(C)
The Bay State Opera Company, located in one of the larger cities in the state, operates with an
annual budget of $750,000, of which $525,000 is taken through ticket sales, and the deficit is met
from various contributors, including the federal government's program to subsidize the arts, various
charitable and artistic foundations, and the local community. In April of 1977, the federal government
changed its funding program and curtailed all grants to the arts for fiscal year 1978. The money thus
saved by the government was to be diverted to direct welfare payments and to a rejuvenated space
program. The total cutback for the Bay State Opera Company's budget was estimated to be $75,000
at the most for 1978. The opera Company's performing season runs for 28 weeks from mid-February
through early September. There are four operas produced each season, with each opera performed
each evening Monday through Saturday plus a matinee on Wednesday and one on Saturday, for the
total of eight performances per week for seven weeks. The Opera Company's theater has a seating
capacity of 400. With an average ticket price of $7, the potential box office income is $627,200,
although the past few years' experience has been less than optimum, with an average attendance of
only 70 percent, yielding box office receipts of $439,040.
Paula Seibel, the manager/prima donna of the company, is faced with the problem of making up the
$75,000 deficiency in the budget caused by the withdrawal by the withdrawal of the federal funds.
She has established that there are no further charitable or patron sources available at this time and
that there is no reasonable way to reduce expenses that would not affect the quality of the
productions. Furthermore, she rejected the idea of raising ticket prices on the grounds that a price
increase would alienate the Friends of the Opera, a local group that has been stalwart in its support
of the company over the years. Also, her impression of the local market for opera is such that higher
ticket prices could actually result in lower box office receipts due to attendance fall off that would
more than compensate for the higher unit price. The only avenue thus remaining for Miss Seibel is to
increase general audience attendance through a program of presenting opera of a more broad-based
popular appeal, thus increasing the potential market and the resultant box office receipts. While
doing this, however, Miss Seibel recognizes the importance of maintaining Bay State's artistic
standards, which the singers and the Friends of the Opera considered extremely high, and, as could
be expected, not to be compromised at any cost. Furthermore in selecting the operas to be
performed for the 1977-1978 season, she realize it is important not only that they attract a larger
audience but that they provide an interesting variety for opera buffs, in terms of production scale,
mood, style, and setting, as well as period and languages. This implies drawing upon the entire
history of opera to provide a balance between lavish productions and small ones, classical and
modern pieces, and comedies, serious dramas and tragedies. Finally, Miss Seibel wishes to select a
program of operas that will have sufficient prestige value of attract national publicity in the opera
world and enhance the reputation of the company. This last consideration would have consequences
in later years as it would improve Bay State's chances of receiving federal funds (as well as other
foundation support) should the situation in the federal budget change.
In her plans for the new season Miss Seibel has to consider the existing expense budget and the
requirement to stay within it. Under no circumstances can the expanded repertory involve higher
costs. The expenses which are influenced by the selection of operas are primarily of a production
nature, such as scenery, lighting, props, costumes, and performers' and orchestra musicians'
salaries. The budget allocations for these items for the upcoming season are:
Scenery $12,000
Lighting 6,000
Properties 7,000
Costumes 9,000
Performers' salaries 200,000
Orchestra musicians' salaries 100,000
$334,000
In Miss Seibel's opinion, the production can be staged within the budgetary restrictions for scenery,
lighting, and props regardless of the type of opera, as these variables tended to be rather fixed. On
The salaries for the musicians in the orchestra are significant portion of the total budget, but Miss
Seibel could find no way to reduce this figure. All operas require music, and, although the
requirements for proficiency among orchestra musicians are not such that star salaries are required,
at a basic minimum a full 30-piece orchestra is needed to supply musical support to the singers, and
there is no way this could be financed for less than $100,000 annually. These factors left as the
major cost consideration in the selection of the four operas the number of performers and their
salaries. Various operas, of course, require different numbers of singers with varying levels of
proficiency, and, for some of the more demanding roles, the company would have to pay the highest
salaries to ensure that the best performers available would fill these roles. The company is not
repertory company as such and does not maintain a professional staff in residence, but rather hires
special singers as needed for each production. Since, it is a professional organization, Bay State is
required to hire and pay union performers who belong to the Actors' Equity Association, an umbrella
organization of all performers in the dramatic arts field, including opera singers. The salary ranges
specified by the Equity Association for a company such as Bay State range from about $250 per
week to $400 per week, depending upon the nature of the role, the singer's experience, and other
factors. By special arrangement with the union, however, Bay State is required to pay scale wages
only for the period of the actual performances, that is, eight shows per week for the seven-week
duration of each opera. The performers are not to be compensated for blocking or rehearsal time,
including final dress rehearsal, even though Bay State sells tickets for the last two stages. Taking all
these factors into consideration, Miss Seibel then began to review the selection of operas to be
performed.
Data Evaluation questions
The following questions consist of items related to the preceding passage above. Consider each item
separately in terms of the passage and on the answer sheet blacken space.
(a) If the item is Major Objective in making the decision; that is, one of the outcomes or results
sought by the decision maker;
(b) If the item is Major Factor in making the decision that is a consideration, explicitly mentioned
in the passage, that is basic in determining the decision.
(c) If the item is Minor Factor in making the decision; that is a secondary consideration that
affects the criteria tangentially, relating to a Major Factor rather than to an Objective.
(d) If the item is Major Assumption in making the decision; that is, a supposition or projection
made by the decision maker before weighing the variables;
(e) If the item is an Unimportant issue in making the decision; that is, a factor that is insignificant
or not immediately relevant to the situation.
1. Popularity of opera.
2. Increase in attendance at Bay State for the 1977-1978 season
3. Possibility of increasing income by increasing the number of performances.
4. Cost of performers' salaries for each production.
5. Impossibility of obtaining federal aid in 1977-1978.
6. Lighting costs for each opera.
7. Selection of an appealing season of operas.
8. Decrease in audience as a result of increased ticket prices.
9. Adherence to budgeted expenditures.
10. Cast size.
DIRECTIONS: In the following questions a conclusion is followed by two statements giving data.
Mark,
(a), If statement 1 alone can bring us to the conclusion.
(b), If statement 2 alone can bring us to the conclusion.
(c), If both the statements 1 and 2 taken together can bring us to the conclusion.
(d), If the statements taken separately or together cannot bring us to the conclusion.
3. Conclusion : Political parties will awaken political consciousness among the people.
Data : 1. Political parties contest elections.
2. Elections arouse political consciousness.
DIRECTIONS: Each question contains six statements followed by four sets of combinations of three.
Choose the set in which the statements are logically related.
DIRECTIONS: From the alternatives, choose the one which correctly classifies the four sentences as
F : Fact : if it relates to a known matter of direct observation, or an existing reality or
something known to be true.
J : Judgement : if it is an opinion or an estimate or anticipation of common sense or
intention,
I : Inference : it is a logical conclusion or deduction of something, based on the
knowledge of facts.
14. 1. Each one of us should play his part in setting things right.
2. The longing for peace is the deepest feeling.
3. If I decide to take the examination, my father will be very happy.
4. May the best man win.
(a) JFIJ (b) JIIF (c) JJIF (d) JJIJ
15. 1. Friendship between Pakistan and India should not be too difficult.
2. We are prepared to resist aggression.
3. If we keep our nuclear options open, Pakistan will remain wary of us.
4. We should, therefore take care that our nuclear research is not stopped.
(a) JFFJ (b) JIJF (c) IFIJ (d) JFIJ
18. Statements : Today we make health an end in itself. But it should be the means only.
We pay more attention to imaginary illness than to actual illness.
19. Statements : The cause of war is greed. The centralised method of production is to
satisfy this greed. It helps them to control the lives of other people.
20. Statements : There are glaring inequalities between man and man. Laws can ensure
an ordered society but only hard work will help us to achieve the social
objective of economic goals.
DIRECTIONS: Two statements are given. These are followed by two possible inferences. Mark
(a), if only inference 1 follows
(b), if only inference 2 follows
(c), if both the inferences follow
(d), if neither of the inferences follows.
23. All hosts are guests. No guest has left this place. So
1. No host has left this place.
2. Some hosts have not left this place.
24. All bookworms are first divisioners. First divisioners have been awarded prizes. So
1. All bookworms have been awarded prizes.
2. Some bookworms have been awarded prizes.
25. All servants are faithful. Some faithful people have to suffer in life. So
1. Some servants have to suffer in life.
2. All servants have to suffer in life.
DIRECTIONS: In each of the following arguments one premiss is missing. Pick out from the answer-
choices the premiss which will complete the argument without imparting any fallacy to it.
26. John is shrewd because he is a politician.
(a) Only politicians are shrewd.
(b) All politicians are shrewd.
(c) All shrewd are politicians.
(d) No politician is shrewd.
97 Pankaj Gandhi’s Academy/Logical Reasoning
27. He is lucky because he has won the lottery.
(a) All who win the lottery are lucky.
(b) Only those who win the lottery are lucky.
(c) All lucky win the lottery.
(d) Some lucky have won the lottery.
DIRECTIONS: In each of the following questions one interrogative statements followed by two
arguments, one beginning with `yes' and the other with `no'. Mark:
(a), if only argument 1 is forceful
(b), if only argument 2 is forceful
(c), if both 1 and 2 are forceful
(d), if neither 1 nor 2 is forceful.
34. Is it beneficial for the national unity of India to separate religion from politics?
1. Yes, because India is a secular state and it is in the interest of national unity to remain
secular.
2. No, because religion imparts a moral touch to our politics.
DIRECTIONS: For questions 41-43: Which of the above answer options follow from the given
statements .
41. Kamal, the philosopher, says he stinks, therefore he is. But he also requires to think, without
which he cannot survive. Either Kamal will think or he will stink. Those close to Kamal say he
is better at stinking than at thinking. Therefore,
A. Kamal, the philosopher, will think.
B. Kamal, the philosopher, will not stink.
C. Kamal, the philosopher, will stink.
D. Kamal, the philosopher, will not think.
(a) AB (b) DB (c) BA (d) CD
42. When the computer is connected, it makes Naru cry. When the computer does not work,
Naru sneezes. Either the computer is connected or the work is delayed. When Naru does not
wait he is early, but if Naru waits the work is delayed.
A. the computer is not connected.
B. the work is delayed.
C. the work is not delayed.
D. the computer is connected.
(a) BA (b) DC (c) AB (d) none of these
44. Flying in the face of the contemporary wisdom which dictates that only stretch targets and a
gigantic leap towards a yet-to-be-envisioned future can lead companies to growth, the step-by-
step argument posits that betting the company on a huge gamble is more likely to fail than
succeed.
Which of the following statements, if true, strengthens the above argument?
(a) Only strategic innovations, involving new products or services can do the job.
(b) Dramatic results act as a buoyant for company morale.
(c) Revolutionary changes that undo the gains of the past are unnecessary.
(d) Not all companies can reciprocate the swiftness of such dramatic steps.
45. Economic nationalism, it would seem, still survives. It is alive and kicking in India.
Which of the following statements, if true, weakens the argument above?
(a) Exporters have been held back for centuries and face the brunt of a licence raj.
(b) Import licences are not issued easily and the import of capital goods is severely
restricted
(c) The government has announced a huge reduction in import tariff.
(d) none of the above.
DIRECTIONS: Given below is a passage followed by a few statements. You have to read the
passage and on the basis of the information available, mark your answer as
(a) if the argument is an upstream argument i.e. if the passage is a direct conclusion or
inference of the question statement
(b) if the argument is a downstream argument i.e. if the question statement is a direct
conclusion or inference of the passage.
(c) if the argument is a lateral argument i.e. if the statement supports the passage but
is neither upstream nor downstream.
(d) if the argument is irrelevant to the passage.
Seldom has the battle for the ballot left the bureaucracy unscathed. Barring 1996, when the advent of
the UF government did not, every new Parliament has, inevitably, ushered in a change in the
bureaucracy.
46. The party in power has to rely on the loyalty of the bureaucracy.
47. The bureaucracy has made itself amenable to the government‘s policy decisions.
Exercise 6(B)
1. The percentage of family income spent on purchase of books has remained almost constant
for the last 20 years—around 5 to 6 percent. When new genres of books became popular, with
new styles, authors and more marketing, they did not expand this percentage. Therefore, book
publishers have observed the CD-ROM literature boom with concern, fearing that every dollar
spent on renting virtual CD books could mean a dollar spent less on books.
Which of the following, if true, would most forcefully undermine the argument above?
(a) The cost of renting a CD-ROM book is generally lesser than the price of a published book.
(b) Most book publishers receive a portion of the income from the sale of CD rights to their
movies.
(c) Fears of some book publishers that CD-ROMs would completely supercede movies have
not come to pass.
(d) Since the start of the Information Technology and CD-ROM boom, money spent on forms of
relaxation other than CDs and books have dropped.
(e) Some books, which were unsuccessful when published, have later become successful
when released in CD form.
2. In an industry where change is the only constant, survival depends on the quickest reaction
time. No one else keeps pace with change for you like PMI, with our "on-line with the world"
approach to software training and development.
The above claim would be seriously weakened if:
(a) Most people are not interested in keeping up with the latest in software development.
(b) There has been no major innovation in software for the last year.
(c) PMI maintains links only with European nations.
(d) More people are inclined towards hardware than towards software development.
(e) PMI has been unable to make use of the latest Microsoft software package.
3. According to LMNO, in 1997, 69.2% of the accused in cases of extortion and robbery were
middle-class males between the ages of 16-30 years. They concluded that most of the heinous
crimes that year were committed by young novices.
The above conclusion assumes that:
(a) Extortion and robbery are heinous crimes.
(b) Young middle-class males are novices.
(c) Conclusions about heinous crimes can be drawn from data concerning extortion and
robbery.
(d) Young novices are between the ages of 16-30.
(e) The cases examined are representative of crime in general.
4. Based on the same data, the above conclusion is seriously undermined if:
(a) The survey examined only 100 criminal cases.
(b) Statistics of kidnapping, murder and rape cases show that the main culprits are middle-
aged men.
(c) The survey only looked at crime in middle class areas.
(d) Most criminals are women.
(e) The definition of ‗heinous‘ crimes is not common throughout the world.
5. The nun with the child claims that Mother Teresa has performed several miracles before and
after her death and deserves to be canonised as a saint. ―This child,‖ she says, ― was born
premature and was nearly dead when he arrived. I prayed to Mother. I knew that she would
help him. It is a miracle.‖
Which of the following would most strengthen the nun‘s claim that Mother Teresa should be
canonised?
(a) No person in this millennium has attained sainthood.
(b) The Vatican has proclaimed that it would forego the requisite five-year waiting period before the
process of canonisation can begin.
6. Based on the same data, which of the following would weaken the nun‘s claim considerably?
(a) There is no doctor willing to testify to the claims the nun makes about the child‘s condition when
he was born.
(b) 30 years after his death, an Italian priest, Padre Pio is still in the process of being declared
a saint.
(c) The child though he has survived is severely handicapped.
(d) Mother Teresa has taken charity from people with dubious backgrounds.
(e) Mother Teresa has been accused of forcible conversions of poor people to Christianity.
7. Bill Gates did not finish his college education. But he was a tremendous success in life .He
explored completely new possibilities in the field of software and went on to become the owner
of the Microsoft Corporation and the richest man in the world. Now Bill Gates is flooded with
questions, on the Internet, from worried and anxious parents who want his reassurance that
though their kids do not study they would be successful in life. But Bill Gates rejects the claim
that he did not finish his education. He insists that he completed his high school which he
feels, is the most important part of a child‘s education.
Which of the following if true would strengthen Gates‘ position?
(a) Education does not teach a child to face the battles of life. It is facing real life, which
teaches a child how to succeed.
(b) Almost eighty percent of the successful millionaires of the world are college dropouts.
(c) Parents of today always tend to cut the Gordian knot and hence rather than finding ways to
encourage their children to study they seek reassurances.
(d) Education in any form is vital in the shaping of a person‘s life and hence can never be
overlooked.
(e) Expensive and competitive college education creates managers of other‘s ideas and not
creators.
8. If Rani acts in Baba‘s music video, then Govinda sings the duet. Either Govinda sings duets in
music videos or he dances at discos.
Which of the following is true if Govinda does not sing?
(a) Rani does not act in Baba‘s video and Govinda does not dance at the disco.
(b) Govinda does not dance at the disco and Rani acts in Baba‗s video
(c) Govinda dances at the disco and Rani does not act in Baba‘s music video.
(d) Rani acts in Baba‘s video and Govinda dances at the disco
(e) Rani and Govinda make a music video together.
9. If Archie wears a blue T-shirt and Ronnie wears yellow, then Betty will wear green. Midge will
wear pink if Big Moose wears beige and Dilton wears black. In case Jughead chooses brown,
then Reggie does not wear red.
What happens if Midge does not wear pink?
(a) Midge wears peach.
(b) Dilton does not wear beige.
(c) Big Moose wears red & Archie wears blue.
(d) Dilton does not wear black.
(e) Ronnie wears yellow & Betty does not wear green.
10. Based on the same, If Ronnie wears yellow but Betty does not wear green, which of the
following must be true?
(a) Ronnie wears a pale shade of pink.
(b) Jughead chooses brown.
(c) Archie does not wear blue.
(d) Archie wears blue.
(e) Betty wears red & Reggie wears green.
Pankaj Gandhi’s Academy/Logical Reasoning 102
11. The recent exposure has left a wake of cynicism regarding the conduct of politicians and
bureaucrats. The rot that seems so deep and so pervasive is matched only by people's
helplessness. Experts and armchair analysts proffer a deluge of ideas suggestion and advice,
but the sad part is nothing ever gets done. Which of the following may be inferred from the
statement above?
(a) The recent exposure has been one that has cast an ill light on the ambassadors.
(b) The government of India has such a long history of running a corrupt and inefficient system
that it is impossible to rectify the system.
(c) In spite of the deluge of ideas proffered by experts and analysts, these ideas have not been
implemented.
(d) Nothing ever gets done because of people's helplessness.
(e)None of the above
12. Total or partial obscuration of the sun's light when viewed from the earth is called solar eclipse.
During a solar eclipse the moon takes up such a position between the earth and the sun that it
partially or completely obscures the light of the sun. It occurs on New Moon day when the
moon is in conjunction with the sun but, not at every new moon. On the other hand the total or
partial obscuration of the moon's light when viewed from the sun is called lunar eclipse.
Which of the following statements does the above passage imply?
(a) During the solar eclipse the earth overshadows the moon which in turn overshadows the
sun.
(b) Solar eclipse occurs when the sun is in line with the earth.
(c) Not every new moon is a sign of the occurrence of solar eclipse during which sunlight is
partially or completely obscured.
(d) Every new moon day is a harbinger of the lunar eclipse.
(e) All of the above
13. We urban Indians grew up on a diet of Enid Blyton's description of the English countryside, the
food the English ate, the clothes they wore and the adventures they had. Meanwhile we
missed out on the actual life that was occurring in our country. The best part of life passed us
by. It was the children in the rural countryside who actually had a life close to nature running
around trees and stealing mangoes.
Which of the following would strengthen the assumption that it is necessary to have a feel of
the life in your own country?
(a) Most children find themselves at a total loss when they grow up because they have neither
had a typically English childhood nor a native upbringing.
(b) Indian children have missed out on the real joys of growing up in the open outdoors.
(c) Experiencing life in one's own country makes children aware of real-life adventures.
(d) A feel of life in one's own country enables one to have a better perception of the flora and
fauna around.
(e) None of the above
14. At a time when terms like post nationalism are in vogue, good old patriotism may seem an
obsolete concept. Thankfully the young don‘t think so. Pride in India, belief in its inherent
strength and the resolve to rise to its defence, should the need arise, is evident. India's future
is therefore in safe hands.
Which of the following statements strengthens the above argument?
(a) The perception is that young think patriotism is an obsolete concept but actually they are
patriotic to the core.
(b) Most college students interviewed felt that if there were a need they would join the army to
defend the country.
(c) The dominance of patriotic themes in pop songs like 'Made in India' and 'I'm an Indian'
testifies to the sentiment amongst the youth.
(d) Ethnic designs that are now a fashion statement reflect the move towards nationalism.
(e) All of the above
16. The dream to create a cornucopia of spare parts for the human body is on its way to becoming
a reality. Human embryonic stem cells - the cells at the beginning of the biological unfolding of
a human embryo - have been isolated for the first time by US scientists who claim that they
would soon be able to mimic parts of nature's programme in the laboratory, making heart,
blood and nerve cells to replace or fortify diseased tissues.
The above passage rests on the assumption that
(a) Human body is like a machine
(b) Henceforth patients would not have to wait in line for transplantable organs
(c) Scientists can assemble a human body out of the spare parts that they will be able to create
(d) Natural processes can be simulated in laboratory conditions.
(e) All of the above
17. Some age-related changes in the brain are more pronounced in men than in women, according
to a study in the American Medical Association's 'Archives of Neurology'. Researchers used
magnetic resonance imaging to examine the brains of 330 elderly volunteers who were living
independently in the community. The popular belief, however, is that senility sets in faster in
women than in men.
Which of the following will weaken the popular belief stated above?
(a) 330 is too small a number for the research to be valid.
(b) Women have more responsibilities than men do, at the workplace as well as at home, due
to which they age faster.
(c) Since the women among the elderly volunteers were living independently, they did not age
faster.
(d) American Medical Association is reputed to have come out with research results that have
never been proved wrong.
(e) None of the above
18. Santa went to see a movie that attracted most of the cowbelt audience. Santa enjoyed the
movie.
Which of the above statements would strengthen the claim ―Santa is a cowherd‖?
(a) All the people who wanted to see the movie were people who tended cattle.
(b) Only people who are cowherds enjoyed the movie.
(c) Santa as well as the people who went to see the movie all wore cowbelts and they all
enjoyed the movie
(d) All the people who wore cowbelts enjoyed the movie
(e) All of the above
20. The greatest asset of any company in its reputation is the confidence of its customers and
shareholders. Nothing is more difficult to achieve or easier to lose.
Which of the following statements most strengthens the above statement?
(a) The software companies surveyed paid great attention to their clients and their needs.
(b) On an average, most companies spend 80% of their allotted publicity budget on customer
relations operations to see that their clients are satisfied and the most trustworthy people
who are people with the best inter personal skills are hired to man these departments.
(c) A number of representative public limited companies surveyed claimed that giving investors
extra stock options that made them happy had nourished their reputation.
(d) All of (a), (b) and (c)
(e) None of (a), (b) and (c)
Exercise 6(C)
Pollant productions was a motion picture company that had built a solid reputation for producing
grade-B movies that were not known for their contribution to cinematic art but generated consistent
profits. One of its early films, a low-budget story about two motorcyclists on a cross-country trip had
become a cult hit and grossed $50 million. Normally, however, Pollant's films grossed between $2
and $3 million, with the total production costs running from $ 1 1/2 to $2 million. Thus providing a
very respectable return on investment to the backers of 40 to 50 percent.
Pollant's secret (which, in fact, the company was not shy about proclaiming) was to control costs very
carefully so that a film could be "brought to the can" for a minimum of cash outlay, and to spot rising
fads and capitalize on them as quickly as possible. It was for this reason that the management of the
company decided to embark on the production of a series of surfer movies. The production crew was
given the assignment of putting together a skeleton proposal for the first film in the series, tentatively
to be titled "Ride the Wild Surf."
After several weeks, the staff had selected a working script and secured the agreement in principle of
two television stars for the leading roles. It only remained to select the shooting site, assemble the
players, props and mechanicals, and begin.
The production crew narrowed the possible shooting sites to three locations: Los angles, central
Colorado, and New Jersey. Each location had its advantages and disadvantages.
Los Angeles was certainly the most convenient. Pollant would not have to pay the travel expenses of
the cast and crew, which would be significant because the union contract called for two days off for
every five days worked, with the cost of travelling home for the weekends borne by the company. The
weekend travel, plus the motel and meals, could run as high as 30 percent of the salaries of the
speaking cast. Also, Los Angles was Authentic; it was here that the sport of surfing became popular,
and everyone in the United States identified surfing with L.A. finally, the weather in Los Angles was
perfect final footage could be shot at the beach virtually every day of the year.
On the other hand, the local population was blast about appearing in movies and insisted on being
paid union scale for nonspeaking and extra roles. Furthermore the labor laws of the State of
California supported them. Since many of the scenes required crowed beaches, a few cloudy days
yhat happened to fall when such scenes were scheduled could affect the budget seriously. Colorado
and New Jersey had unlimited numbers of free extras. Pollant had made other movies outside of L.A.
and found the locals willing, cooperative, and patient, even when a scene required several takes. In
terms of travel, Colorado was obviously cheaper than New Jersey, and there were several large
lakes in the state whose far shore was past the horizon, thus passing for an ocean. However,
shooting plausible surfing scenes in Colorado necessitated a wave machine to create artificial surf.
Pollant could lease a machine, but the company would be force to transport it from California. The
machine also created two new problems. In the first place, it was unreliable and frequently broke
down in the middle of a sequence of important shots. It could take from ten minutes to several days to
fix, depending upon the seriousness of the breakdown and whether the production staff's handyman
could repair it or whether it was necessary to call the manufacturer in L.A. to fly a repairman out to
Colorado. The second drawback to the wave machine was that no amount of artistry with the camera
could hide the fact that the waves were not true surf. They could be made large enough, and indeed
could be ridden, but something was missing; the creative people could not bring themselves to
believe in the artificial waves. Most of them had surfed for sport-and some had even competed; the
sudden eruption in the middle of a placid Colorado lake did not inspire them to their highest artistic
efforts. In addition, while midwestern audiences might be fooled by carefully controlled photography,
anyone who had really surfed, or, for that matter, had been to an ocean beach, could tell that the
waves were not authentic.
New Jersey did not have this drawback, as the surf was genuine, but New Jersey was limited by its
short summer season and, of course, the travel expense. Also, the leading man, about whom pollant
was very enthusiastic, refused to travel to the East Coast. He maintained that personal problems
Henry Babodge, the associate producer for the project was given the assignment of preparing an
analysis of the alternative locations. He decided first to break the issues down into two major
categories: tangible and subjective. The tangible issues were those that could be quantified so that a
decision could be made on the basis of finances alone, such as travel costs, wave machine rental
and repairs, extras, salaries, and similar items. The subjective issues included using the second-
choice leading male actor in subsequent films and using artificial waves in a Colorado lake. These
were issues that Babidge considered only resolvable by top management. While Pollant was famous
in the industry for low-budget successes, nevertheless it was important that the low budget not be
evident to the audience. Since "Ride the Wild Surf" was to be the first in a series rather than a stand-
alone project, a somewhat longer view had to be taken. It was even acceptable. Babidge reasoned,
for "Ride" to break even or show a loss. if it laid the groundwork for a half-dozen lower-cost new
releases. If the imagination of the public could be captured and audiences become entwined in the
lives and loves of the central characters, the full series could be a financial blockbuster.
The following questions consist of items related to the preceding passage above. Consider each item
separately in terms of the passage and on the answer sheet blacken space.
(a) If the item is Major Objective in making the decision; that is, one of the outcomes or results
sought by the decision maker;
(b) If the item is Major Factor in making the decision that is a consideration, explicitly mentioned
in the passage, that is basic in determining the decision.
(c) If the item is Minor Factor in making the decision; that is a secondary consideration that
affects the criteria tangentially, relating to a Major Factor rather than to an Objective.
(d) If the item is Major Assumption in making the decision; that is, a supposition or projection
made by the decision maker before weighing the variables;
(e) If the item is an Unimportant issue in making the decision; that is, a factor that is insignificant
or not immediately relevant to the situation.
DIRECTIONS: Each question contains six statements followed by four sets of combinations of three.
Choose the set in which the statements are logically related.
4. 1. All M is P
2. No S is M
3. No S is P
4. Some S is not P
5. No P is M
6. Some M is S.
(a) 564 (b) 123 (c) 231 (d) 254
DIRECTIONS: In each of the following questions one interrogative statement is followed by two
arguments, one beginning with `yes' and the other with `no'. Mark:
(a), if only argument 1 is forceful
(b), if only argument 2 is forceful
(c), if both 1 and 2 are forceful
(d), if neither 1 nor 2 is forceful.
DIRECTIONS: In each of the following questions some statements are followed by two inferences.
Point out if:
(a), Conclusion 1 follows.
(b), Conclusion 2 follows.
(c), Conclusion 1 and 2 follow.
(d), Neither 1 nor 2 follows.
11. Statements : A writer works under the compulsive need to create a product of beauty. A
work of art is itself a reward.
Conclusions : 1. A writer does not expect any reward for his works.
2. A work of art is a beauty in itself.
12. Statements : We do not educate children for the purpose of educating them. We want to
prepare them for life.
13. Statements : Accumulation of knowledge is the result of the collection of little bits.
Those who accumulate nothing are failures in life.
14. Statements : Neither poverty nor ascetic's life should be praised. Mind and body should
be disciplined but not tortured.
15. Statements : The traditional role of the universities was to serve scholarship. But in a
democratic society it takes into account the needs of the society.
17. All coins are rupees. Some rupees are paper currency. So
1. All coins are paper currency.
2. Some coins are paper currency.
19. All plants are shady. All shady things are good. So
1. Some good things are plants.
2. Some plants are good things.
20. Some pins are tins. Some tins are not shins. So
1. Some shins are not pins.
2. Some pins are not shins.
22. If you are a player you will not enjoy the game. You are a spectator. So
1. You will enjoy the game.
2. You will not enjoy the game.
23. If you get up early you can finish your work. You cannot finish your work. So
1. You get up early.
2. You do not get up early.
24. If you are an officer you can attest this paper. You are an officer. So
1. You can attest this paper.
2. You cannot attest this paper.
DIRECTIONS: In each of the following arguments one premiss is missing. Pick out from the answer-
choices the premiss which will complete the argument without imparting any fallacy to it.
DIRECTIONS: In the following questions one statement is followed by one inference. Assuming the
statement to be false, point out if the inference is (a) True, (b) False or (c) Uncertain.
33. The Homo Erectus evolved into the Homo Sapiens, and he is not the end of evolution.
1. Man is the product of evolution.
2. Evolution is a continuous process.
DIRECTIONS: In the following questions a conclusion is followed by two statements which give us
data for the conclusion. Study that data and the conclusion and mark.
(a), If statement 1 alone can bring us to the conclusion.
(b), If statement 2 alone can bring us to the conclusion.
(c), If both the statements 1 and 2 taken together can bring us to the conclusion.
(d), If the statements taken separately or together cannot bring us to the conclusion.
37. Conclusion : The task of the civil servant is to transform policies into action.
Data : 1. Many centuries ago Varna system transformed itself into caste
system in India.
2. Caste system has corrupted our politics.
DIRECTIONS: Given below is a passage followed by several inferences. Examine the inferences
separately in the context of the passage, and determine whether they are true or false. Mark
(a) if the inference is definitely true, i.e. it properly follows from the statement of facts given.
(b) if the inference is probably true, though not definitely true, in the light of statement of facts given.
(c) if the inference is uncertain, i.e. data is insufficient to decide whether the inference is true or
false.
(d) if the inference is probably false, though not definitely false, in the light of statement of facts
given.
(e) if the inference is definitely false, i.e. it cannot possible be from the statement of facts given.
A word about preventive medicine, or helping yourself to health. A great deal of nonsense has been
talked about healthy life; at one time we were told to take 18 chews of each bite, to do deep
breathing, to take plenty of exercise, to get lots of fresh air, to eat regularly (eat peculiar diets). But
modern researchers have cast doubts on these fancies. Moderate exercise is necessary for health
but athletes who do vigorous exercise are not noted for their longevity. Fresh air is stimulating, but is
no better than the normal air in most rooms. But general quality of air affected by pollution is indeed a
concern.
Exercise 7(B)
1. A consultant hired to increase efficiency in a rubber sole manufacturing company told the
manager: Increase the size of the raw rubber sheet rolls purchased as raw material. Buy ten
ton rolls instead of the two ton sizes currently in use. This way, with more rubber sheeting to a
roll, the operators would not need to stop often for the time consuming work of changing rolls.
This would reduce labour costs substantially.
The efficiency expert apparently assumes that
(a) rubber sheets wound on large rolls are not as strong as sheets wound on smaller rolls.
(b) sole manufacturing machines do not break down and do not require maintenance.
(c) workers in the factory are paid by the hour rather than on a piecework basis.
(d) machine operators are not allowed to leave their machines during the work period.
(e) speeding up production will improve the quality of soles produced in the company.
2. ―Overall‖, said the professor, ―the science graduates now attending the University are more
interested in art than the science students of ten years ago. The proof is that more and more of
my students who are in the science stream attend art class on a regular basis.‖
The professor‘s conclusion depends upon which of the following assumptions?
(a) The professor‘s science students show greater interest in art because the professor is a
better teacher than she was ten years ago.
(b) Science students are more concerned about getting good scores in art because it increases
their chances of getting a University scholarship now.
(c) Whether or not a student attends art class on a regular basis is a good measure of whether
a student is interested in art.
(d) Professors should not expect from science students the same kind of commitment to art that
they give to science.
(e) An interest in art is something every student must learn to inculcate in himss early in his
academic career.
3. Based on the same data, what do you think is a possible weakness in the professor‘s
argument?
(a) The students who enroll in her classes are not typical of students in general.
(b) Many other students who are not science students are also interested in art.
(c) The professor does not take into account the special demand placed upon science
students.
(d) The professor does not explain why students are more artistically inclined than ten years
ago.
(e) Some science students should be given special coaching to ensure that they make the
grade in art.
4. People in favour of banning lotteries state that lotteries are immoral because they cause the
poor to spend their hard earned money frivolously. While it is true that poor people tend to
purchase proportionately larger number of tickets than do the middle-class and the wealthy, it
is also true, by extrapolation that they win the jackpots relatively more than the wealthier
people. So the system is not regressive.
The argument above can be most weakened by pointing out that
(a) a person who wins a lottery can no longer be termed poor.
(b) only a few of the poor who buy lottery tickets win large lottery prizes.
(c) many of the middle class and the rich do not buy lottery tickets.
(d) most countries do not have national lotteries.
(e) the revenues generated by lotteries serve to reduce taxes for everyone.
6. In the latter part of the century, especially between 1950 and 1970, large scale illegal trawling
by the private boats and hunting killed over 50,000 Olive Ridley sea turtles in the state of
Orissa in India, in order that they be served as a delicacy in restaurants. This left the total
Olive Ridley turtle population in the Orissa coastal areas teetering precariously at just about
40,000. In 1975, the Indian government started implementing the laws regarding the hunting of
the turtles and arrested and convicted many people for indulging in the act. Still, by 1990, the
population of the Olive Ridley was seen to have fallen to around 30,000.
. Which of the following, if true, would best help explain the apparent paradox presented above?
(a) The poachers arrested in Orissa were rarely sentenced to long prison terms.
(b) Because of highly publicised campaigns against the slaughter of turtles, demand for turtle
meat fell drastically between 1875 and 1990.
(c) The turtle population in the neighbouring country of Sumatra rose slightly between 1975
and 1990.
(d) Before 1975, the anti-poaching laws passed by the parliament were rarely enforced.
(e) In Orissa, thousands of kilometres of coastline, the breeding ground of the turtles, were
used for constructing holiday resorts between 1975 and 1985.
7. The use of butadiene or tar to metal roads should be stopped immediately because it can be
used for the better purpose of manufacturing petrol. This is more important for a country such
as ours in that we do not have heavy petroleum resources and a crisis such as a war can cut
off our import supplies. By reducing the use of tar in something that has viable alternatives like
slag, we can make a major step towards energy independence and enhance our country‘s
security.
Which of the following, if true, would most weaken the argument above?
(a) Only a small fraction of the job of metalling roads is done by the tarring method.
(b) New methods of metalling roads can reduce the amount of tar needed and thus save petrol.
(c) The development of the solar powered car will reduce our dependence on petrol by the
middle of the next century.
(d) In times of war, warring nations would be tempted to seize forcibly as many barrels of tar as
possible from each other.
(e) Good metalled roads play an important role in the progress of a nation, and also help in
smooth troop movement in wartime.
8. Lance Klusener‘s nickname ―Zulu‖, refers to his fluency in that language, which reflects the fact
that he grew up in rural South Africa. As a result, almost uniquely among modern international
cricketers, he never played representative cricket as a junior. Once his abilities were noticed,
during a provincial practice session in 1993, it took him a mere four years to become an
established member of the national team.
The passage above is structured as:
(a) Cause-Effect Relationship
(b) Statement of Fact
(c) Point – Illustration
(d) Description
(e) Hypothesis and argument.
10. The impact of fraud on business is not limited to the amount defrauded. A major fraud can put
an entire business at risk and damage its reputation. It could create an unhealthy work
environment, where honest employees feel that one way to be rewarded is to defraud the
company. So, companies will now have to invent a comprehensive fraud-prevention plan to
safeguard themselves.
Which of the following statements, if true, weakens the above argument?
(a) Once a fraud is detected, the investigation does not follow in more than 30% of the cases,
which encourages the fraudsters.
(b) Companies, at the time of inception, implement foolproof security measures & good internal
controls, as stipulated in law and still are frequently defrauded..
(c) The companies that have previously implemented emergency fraud-response plans
realized that it had an adverse effect on the employees‘ morale.
(d) If the investigation is carried out, the fraudsters are severely punished according to
company rules and face legal action too.
(e) The quantum of losses due to fraud is not very large and hence does not justify the time
and money spent by companies to safeguard themselves.
11. Technology is no longer a set of back-office systems but a critical part of a company's ability to
deliver goods and services.
Which of the following statements may be inferred from the above statement?
(a) Companies undertake home delivery for customer satisfaction, which in turn spells higher
profits for the company
(b) Technology was once a part of back office systems but not anymore because technology is
a crucial part of the company's survival
(c) Without technology business is impossible.
(d) Both (a) and (b)
(e) None of (a), (b) and (c)
12. The net as a medium for shopping and buying opens new alternatives for consumers offering
them a different set of trade-offs between cost selection and convenience than any other
channel. Retail merchants are rushing to exploit its characteristics in their competition for
customers.
Which of the following statements most strengthens the above argument?
(a) The number of businesses involved in E-commerce has grown substantially aided by
hosting and other services.
(b) The net offers some compelling advantages to the buyer over traditional purchasing
methods.
(c) It is not understood if state regulation or industry self regulation should be implemented.
(d) Integrating web based E-com systems with existing corporate applications will be a major
challenge for companies implementing E-commerce.
(e) None of the above.
14. Chandrababu Naidu is striving to turn Kuppam, his home constituency into a model of sorts.
The Andhra CM has already built a 100-bed hospital, a sprinkler irrigation system with Israeli
help, spruced up the roads, installed sodium vapour lamps on every street and even built a
new guesthouse. There is more coming. A food processing plant and a private medical college
are among the projects recently approved. The town‘s proximity to Bangalore is touted to draw
private investment.
Which among the following can be an inference of the above passage?
(a) Chandrababu Naidu is a dedicated politician and leads by example.
(b) His home constituency was underdeveloped.
(c) Chandrababu Naidu‘s development projects are backed by trusting investors.
(d) All of (a), (b) and (c)
(e) None of (a), (b) and (c)
15. 2000 AD. It‘s not just another new year; it‘s a time of self reckoning, time to reset our much
abused bio – clock and appreciate the gift of life; by going back to basics, back to Nature. Back
to the good old days of the humble bicycle. Biking is a simple exercise for body and mind,
helping us connect with Nature which no one can deny is therapeutic. Maybe it is time to reset
that bio clock. And go biking in the freshness of an early morning.
This passage rests on the assumption that
(a) Bicycles are environment friendly.
(b) Biking is good for physical and mental health.
(c) Bicycles have been in disuse.
(d) Early morning is the best time to go biking.
(e) All of the above.
16. Television isn‘t always the idiot box. ‗India‘s Most Wanted‘ on Zee helped nab 24 criminals.
Now Sony‘s ‗Missing‘, a series about missing people has also opened its account. It has
helped reunite 11 year old Kartik Tamrai Selvan who had been missing since January 20 with
his parents. Kartik ran away from his Chembur home and ended up in Mathura where he was
looked after by a Muslim family. When Kartik‘s case was featured in an episode aired on
August 2, his guardians saw it and promtply rang up Mumbai to have him sent back. ―‘Missing‘
has used the power of positive television in helping a family,‖ says a channel spokesperson.
Which of the following can be inferred from the given passage?
(a) The T.V. promotes national integration because it has helped a Hindu boy from the south
grow up in a Muslim family in the north.
(b) Lost and Found columns are now being televised.
(c) Some T.V. channels are now favouring novel uses of the T.V..
(d) Sony is trying to compete with Zee.
(e) None of the above.
17. He is the man Nagpur wants as advisor on economic affairs in the PMO. And he‘s got some
ideas on how to turn India into an economic superpower. At a recent RSS meet Bharat
Jhunjhunwala was overheard proclaiming the merits of cow‘s urine. He told the visibly
impressed audience that cow‘s urine, if suitably bottled and marketed, would rake in huge
amounts of foreign currency for the country.
Which of the following can be inferred from the above passage?
(a) Bharat Jhunjhunwala believes that if he can make the merits of cow's urine known he can
impress the audience and therefore become the advisor on economic affairs in the PMO.
(b) Bharat Jhunjhunwala believes that cow's urine has qualities that will attract an international
market and that this would turn India into a super power.
117 Pankaj Gandhi’s Academy/Logical Reasoning
(c) Cow's urine has qualities, which will attract international market.
(d) One of Bharat Jhunjhunwala‘s plans is to turn India into a super power by bottling and
marketing cow‘s urine.
(e) All of the above
18. Thousands of children died in the past six months, of a mysterious illness related to what our
officials call ‗undernourishment‘. The correct word is ‗starvation‘, but since it has been officially
abolished in India, officials dare not use it. So when children were dying of hunger, the government
sent them medicines.
Which one of the following is a strengthening argument?
(a) We have such an excess of food grains in the government warehouses at the moment – 50
million tonnes instead of the 17 million we require as buffer – so starvation deaths anywhere in
India are an outrage.
(b) Our leaders would rather let rats eat those million tonnes of food grains than save the lives of
children.
(c) Over the years, our officials have evolved a wondrous system whereby state governments have
become experts at pretending that starvation does not exist. They pretend that there is an
epidemic and not a famine.
(d) The first reaction of state governments when starvation deaths occur is to deny them and
prove themselves right with long lists of illnesses that have caused the deaths to back up
their argument.
(e) None of the above
19. Traders and urban consumers may enjoy the crop boom, but farmers have seldom if ever had
it so bad. Two days ago, K.N. Nataraja threw his tomatoes into a ditch rather than sell them for
practically nothing. He cursed being born into a farming family as he bagged up another load
in the hope of getting some return. For most farmers, farming is like running a government –
the deficit just builds up year after year. The difference is that the finance minister is never
held personally responsible for repaying the national debt. K.N.Nataraja had drunk pesticide
rather than face the disgrace of not being able to pay back his loans.
From the given passage we can infer that
(a) K.N. Nataraja is unlucky because he has to pay his debts while the finance minister is never
held responsible for the national debt.
(b) The price of tomatoes has reached a rock-bottom level due to stiff competition from other
farmers, who have made use of the crop boom.
(c) The price of tomatoes plummeted due to the crop boom.
(d) Cash crops spell ruin for farmers, born into a farming family.
(e) All of the above
20. At the heart of every Bio-Fresh refrigerator is Biotechnology, which uses Bio-ceramic material
in the manufacturing of the vegetable compartment. Ceramic has the unique property of
emitting InfraRed rays spontaneously. These rays help maintain the natural moisture and
nutrition content in vegetables, fruits and other natural products. Therefore, the Bio Fresh
Vegetable box, in combination with other features like the Bio Deodorizer, Dynamic Super X-
Flow Cooling System and Humidity Controller, helps maintain the freshness and nutrition of the
food for long.
The passage assumes that
(a) Fresh vegetables and fruits are indispensable for good health and every good refrigerator
should keep food fresh.
(b) Infra Red rays preserve the freshness and nutrition value of food for a longer time, and it is
always better to buy a fridge with the bio-fresh vegetable box.
(c) A refrigerator prolongs the freshness of vegetables, fruits and other natural products, and
so eatables are safer when stored in a fridge.
(d) People want vegetables, fruits and other natural products to stay fresh and that they are
interested in a fridge that will keep food fresh for longer.
(e) None of the above
Exercise7(C)
FSL was a food manufacturing company established in 1905. Until 1970, its major products consisted
of tomato specialties such as pickles, and barbecue sauces. Its consumer products business
accounted for 40% of sales; the balance consisted of sales to restaurants, hospitals, and to the
armed forces. The company had advertised for restaurant, hospital market but never for household
consumers.
In 1970, the company introduced a new line of special products meant for the consumer market. The
line was composed of a number of dishes. Each package contained all of the necessary ingredients
(except meat) including seasoned tomato sauce, cheese, and noodles.
The idea for the line of these products had been conceived by Jagdish, son of the company's
president. Jagadish's enthusiasm for the product was quickly picked up by other executives. The
financial expert wanted expansion which would enable the company to solve a number of financial
problems associated with its inability to attract outside capital.
Many meetings were held throughout the summer. The original thinking of the committee was that the
product line should be introduced at the beginning of the food merchandising season, which started
on about October 1. This deadline, however, subsequently proved to be unrealistic. Production of the
first items in the line did not get underway until Sept., 30 and packaging difficulties prohibited
introducing the product before mid-December.
In July the problems involved in the product introduction were not foremost in the planners' thoughts.
Many hours were spent on discussing the name of the product. Finally, the name vegetable scotch
was adopted as a compromise, but without enthusiasm from the President's son who believed that a
name did not describe the gourmet image that he thought the name should express. With the
exception of the name this man directed most of the decisions related to the marketing program.
From the beginning he argued that there were already plenty of "middle class" products on the
grocers' shelves. What was needed, he believed, was a prestige-even a "gourmet" -line. The
popularity of expensive restaurants in many cities convinced young manager of the opportunity to
market these food specialties.
Early in the planning it was decided not to limit distribution to the regional markets in which this
company had previously established its reputation. National distribution would be undertaken from
the beginning. It was planned that the preparation would be marketed in all major food chains except
those handling only private or controlled brands. Sales chain and headquarters would be made by
food brokers handling such products rather than brokers used to handling other goods.
For the first time in its experience, FSL planned to undertake an extensive consumer advertising
program. A small Delhi advertising agency with slight experience in handling food products was
appointed. However, by the time the agency had been selected and oriented to the marketing
program, the time remaining before the scheduled introduction did not allow for the preparation of
advertisements or sponsored programs on T.V. In order to break into the consumer market at the
time of product introduction i.e. on October 1, a consumer advertising program using newspapers,
television commercials and radio was prepared. Except for the product introduction period, however,
relatively little thought was given in planning sessions to the total amount money required to support
the new product with consumer advertising.
The new product was finally launched in mid- December. However, by February, two major
competitors began marketing similar products. Shortly thereafter a market research survey was
sponsored by the company to determine whether the product made a favorable impression on
housewives. The result of the survey were negative. Only twenty two per cent of the housewives
interviewed could recall the name and of those only twelve per cent had tried the products.
Consumer evaluation of the product was for the first time, only four per cent stated that they would
buy it again.
Another indication that worried the company's management was that few major food chains showed
interest. By mid-year product sales were so poor that management established a special committee
to determine without delay what immediate steps might be taken to reverse the poor sales record.
The following questions consist of items related to the preceding passage above. Consider each item
separately in terms of the passage and on the answer sheet blacken space.
(a) If the item is Major Objective in making the decision; that is, one of the outcomes or results
sought by the decision maker;
(b) If the item is Major Factor in making the decision that is a consideration, explicitly mentioned in
the passage, that is basic in determining the decision.
(c) If the item is Minor Factor in making the decision; that is a secondary consideration that affects
the criteria tangentially, relating to a Major Factor rather than to an Objective.
(d) If the item is Major Assumption in making the decision; that is, a supposition or projection
made by the decision maker before weighing the variables;
(e) If the item is an Unimportant issue in making the decision; that is, a factor that is insignificant
or not immediately relevant to the situation.
Exercise 8 (A)
DIRECTIONS: One interrogative statement is followed by two arguments, one beginning with `yes'
and the other with `no'. Mark:
(a), if only argument 1 is forceful
(b), if only argument 2 is forceful
(c), if both 1 and 2 are forceful
(d), if neither 1 nor 2 is forceful.
DIRECTIONS: Pick out from the choices the premiss which will complete the argument without
imparting fallacy to it.
DIRECTIONS: In the following questions, two statements are followed by two inferences. Write
(a), if only inference 1 follows
(b), if only inference 2 follows
(c), if both of them follow
(d), if neither of them follows.
13. All Europeans are black. All Indians are black. So.
1. All Europeans are Indians.
2. All Indians are Europeans.
16. If you leave just now you will catch the train. You leave just now. So
1. You will catch the train.
2. You will not catch the train.
17. If you worship God you will be happy. You are happy. So
1. You worship God.
2. You do not worship God.
19. If he drives slowly he will not reach in time. He will reach in time. So
1. He does not drive slowly.
2. He drives slowly.
20. If you touch the thorn it will prick you. The thorn will not prick.
1. You touch the thorn.
2. You do not touch the thorn.
24. 1. Whatever is valuable in the old, you preserve and whatever is modern you absorb.
2. Our professions are one thing, our practices are quite different.
3. Your conscience is your most valuable ally.
4. There is no such thing as a benevolent ghost.
(a) FIJF (b) IFJF (c) IFJJ (d) FJJF
25. 1. A fallen and prostrate India cannot be of any help to the world.
2. Death is not the end of everything.
3. An encyclopaedia is the best guide to general knowledge.
4. Getting one moment in time will make us immortal.
(a) JIFF (b) IJJF (c) IFJI (d) FJJI
DIRECTIONS: Assuming the statement to be false, point out if the inference is (a) True, (b) False or
(c) Uncertain.
31. Conclusion : Customs officials have arrested persons trying to smuggle gold.
33. Conclusion : The government affects all the major spheres of life of citizens.
DIRECTIONS: Six statements followed by four sets of combinations of three. Choose the set which is
most logically compact.
DIRECTIONS: Read the following passage carefully and answer the questions that follow. Mark
(a) if the statement is a logical conclusion to the passage.
(b) if the statement is a contradiction to the intent of the passage.
(c) if the statement is a far-fetched conclusion.
(d) if the statement is irrelevant to the passage.
Tata group holding company Tata Sons announced last month its decision to set up a trust to fund
political parties. This is the first time such a corpus is being publicly set up for the purpose, although most
companies have long been bankrolling, albeit covertly, politicians‘ election expenditure.
41. Tata Sons‘ move has led to a debate on the integrity of companies, which aid election expenses.
42. Corporate houses have for long tried to avoid funding elections in order to avoid the long arm of
law.
43. Corporate houses do not need to fund the election expense of political parties, which are funded
by individuals and the state.
44. Most politically motivated companies are eager to contribute to the Tata Fund as they will be able
to arm-twist the party that comes to power.
45. The setting up of funds publicly for political parties goes a long way in providing transparency to
the process of funding elections.
DIRECTIONS: Given below is a passage followed by a few statements. You have to read the
passage and on the basis of the information available, mark your answer as
(a) if the argument is an upstream argument i.e. if the passage is a direct conclusion or inference of
the question statement
(b) if the argument is a downstream argument i.e. if the question statement is a direct conclusion or
inference of the passage.
(c) if the argument is a lateral argument i.e. if the statement supports the passage but is neither
upstream nor downstream.
(d) if the argument is irrelevant to the passage.
Arun Bharat Ram, as the promoter, has no option but to increase his stake in his company SRF Ltd.
46. Arun Bharat Ram may lose the only company that he has and manages.
47. SRF is in the clutches of a takeover threat.
48. A lowering of the promoter‘s stake could make a company vulnerable to takeover attempts.
49. A takeover threat can be tackled by the management at the highest rung.
50. Arun Bharat Ram has exhausted all his options due to family problems in a family-owned
business.
125 Pankaj Gandhi’s Academy/Logical Reasoning
Critical Reasoning
Exercise 8(B)
1. The city police department reported that approximately 80 percent of all petty crimes go
unreported. So we can conclude that about 80 percent of all traffic violations also go
unreported.
The argument above assumes that
I. Statistics about petty crimes are applicable to traffic violations as well.
II. Traffic violations represent a more serious danger to the individual than do petty crimes.
III. The average dollar value of loss due to a petty crime is equal to the average dollar value of
a traffic violation caused loss.
(a) I only (b) II only (c) III only (d) I & II only (e) I & III only
2. Studies show that certain charitable organisations often use the foot-in-the-door-technique for
donations. To gain compliance, they ask their potential donors to first grant a small request.
Later, they make a larger request, the one that they really want.
Which of the following statements is an underlying assumption for the above argument?
(a) Once people agree to a small request, they agree to a larger request to be consistent with
their enhanced self- image of helping others.
(b) Potential donors are likely to help other organisations too and so they will not be able to
help much.
(c) Many people will help charitable organisations because of tax-exemption benefits.
(d) If the larger request is made along with a small request, 60% of the people will grant either
one of them.
(e) Most of the upper-middle class families donate money to a charitable organisation at least
once in a year.
3. When the Regency Restaurant serves Biryani and Manchurian on the same day, it also serves
Enchilados. Enchilados are never served on Thursday. Manuel will eat at the Regency only if
Biryani is being served.
If the statements above are true, which of the following must also be true?
(a) Manuel does not eat at the Regency on Thursday.
(b) Manuel does not eat both Biryani and Manchurian on the same day at the Regency.
(c) The Regency does not serve Manchurian on Thursday.
(d) Thursday is the only day when the Regency does not serve Biryani.
(e) Biryani and Manchurian are not both served together at the Regency on Thursday.
4. For this study, five qualities of a good management college were chosen—standard of faculty,
placements, reputation, alumnae and extra-curricular activities. For each college, scores from
1(lowest) to 10(highest) were assigned for each of the five qualities. The five scores for each
college were then averaged, yielding a total desirability score for each college. We hope that
this study will help you choose your college for further education.
The passage above makes which of the following assumptions?
I. It is possible to assign an accurate numerical score to each of a college‘s facilities.
II. Each of the five qualities of a college is equally important.
III. Most people enjoy some degree of personal choice in where they would study.
(a) I only (b) II only (c) III only (d) I & II only (e) II & III only
5. It has been seen that in the SAT examination, Indian students tend to score higher in
Reasoning areas than do American students at the same level. Some educational reformers in
the USA have attributed these results to the fact that the Indian educational system is very
rigid in structure and pays a lot of attention to rigorous study, homework, and memorisation,
much more than the American system.
Which of the following, if true, would seriously weaken the conclusion drawn by the American
educational experts?
(a) The Indian elementary school system is far less rigid and structured than the American
system.
6. There is a saying in military circles that ―Generals always prepare for the previous war.‖ In
much the same way, government officials spent time and energy solving problems that were
already resolved—one way or another—years before. By the time an issue reaches the
consciousness of enough citizens to become a high priority item in the minds of the officials,
the problem is usually past the point at which government officials can do anything about it.
All the following are assumed in the argument EXCEPT
(a) Most public problems tend to evolve towards a point at which the government can do little
to control them.
(b) Political and military leaders are both prone to react tardily to changes in their fields.
(c) Issues attain importance for public officials when a large number of citizens are concerned
about them.
(d) Planning policies solely based on past experiences is likely to be ineffective.
(e) Government officials can generally do little to influence directly the course of public policy.
7. If Graf wins Wimbledon this year, she will try to break Navratilova's record next year.
Therefore, if Graf does not win:
(a) Graf will retire.
(b) Navratilova will try to break Graf's record.
(c) Graf will not break Navratilova's record.
(d) Graf will not try to break Navratilova‘s record.
(e) None of the above.
8. When Uncle Bill took over as Prime Minister, Uncle Tony said he would sign the
SeeTeeBeeTee. However, with the inclusion of Uncles Boris and Slobodan, in the ministerial
coterie the equation changed. Now Uncle Tony insists that he will sign only if the other two
have signed first. Uncle Slobodan refuses to have anything to do with Uncle Tony.
Therefore, if Uncle Tony does not sign,
(a) Uncle Boris has not signed.
(b) Uncle Slobodan has not signed.
(c) Uncle Bill has not become Prime Minister.
(d) Both (a) and (b).
(e) None of the above.
9. Neither by the templates of globalisation nor by the principle of one-world, one-market can
transnational/multinationals triumph in India. It is a radically different market. To win over the
country‘s one billion customers transnationals must understand how global and how Indian
they must be. Both the extremes of clean slate localisation and following a rigid globally
erected template do not work for the highly demanding Indian market.
So, the winning gameplan for transnationals in India would probably be-
(a) Entering the Indian market through joint ventures with those companies who know the pulse
of the Indian market
(b) By employing managers who have grown up in India and those who understand the local
market.
(c) By slashing the prices of their products, which would ensure that a larger section of the
population could afford it.
(d) By resorting to a combination of strategies that attract the Indian children.
(e) To simply forget the global pedigree and simply behave like a new company in India.
11. Literacy and education levels are no guarantee for curbing sexual abuse and exploitation,
even when it comes to children. That has been proved by a state government-sponsored study
which reveals that a flourishing flesh trade involving boys and girls between ages eight and
eighteen and operates in the cities of Thiruvananthapuram, Kochi and Kozhikode.
Which of the following will strengthen the argument postulated above?
(a) Madras has a seamy history of child abuse.
(b) During a recent raid in a brothel consisting of children, the police was shocked to find
several graduates there.
(c) Elite households have more sordid stories swept under the carpet than those of the
underprivileged.
(d) Lonely NRI wives make up a huge clientele for the young boys.
(e) None of the above
12. Remember when you were a little kid. And dad's travelling bag, to you, was a treasure-trove of
little gifts, tucked under wraps of clothing. Dinky cars. Dolls that blinked under their long, hard
eyelashes. Perfumed pencils. Remember how, you would have his suitcase all opened-up,
before he even had time to loosen his tie. Men never grow up really. Digital products are a gift
of the modern world. Reach into the open bag. Revisit your childhood.
The assumption underlying the passage is
(a) Women grow up.
(b) Cars, dolls and pencils make ideal gifts, but so do digital products.
(c) All of us relish our childhood, and would like to revisit our childhood.
(d) Digital products excite interest.
(e) All of the above
13. Sonia Gandhi's pathological shyness is by now legendary. At the President's banquet for
Clinton, even though some extraordinarily interesting and influential folk were in attendance,
Mrs. Gandhi preferred to spend almost the entire evening in the profitless company of Mrs.
Murli Deora. That evening there were many others who Sonia would have benefited by
meeting both in party and personal terms. Many world leaders are self-effacing to the point of
being timid, like Richard Nixon, Dr.Manmohan Singh and Narasimha Rao.
Which of the following can be inferred from the above passage?
(a) Many world leaders like Richard Nixon, Manmohan Singh, and Narasimha Rao spend their
time in profitless company, when they can meet interesting and influential people.
(b) The author is of the opinion that Sonia Gandhi is a world leader.
(c) The author is of the opinion that Sonia Gandhi should interact with the crowd.
(d) Mrs.Gandhi was not on speaking terms with the others at the banquet that evening, and
found the profitless company of Mrs.Deora interesting.
(e) None of the above
15. It is widely believed that education cultivates and refines people. However, witnessing the
behaviour of educated people proves otherwise. The educated section of the society has
proved to be incapable of making crucial decisions or comes up with erroneous decisions due
to which the situation gets out of hand.
Which of the following can refute the point made in the passage?
(a) Educated people are like data storing machines and do not have the ability to process the
information they possess.
(b) Educated people have their minds primed with extensive knowledge, which aids decision-
making.
(c) Education involves cramming maximum amount of knowledge in a minimum period.
(d) In the course of education, one gets incomplete information due to which decisions are
unsound.
(e) None of the above
16. The term "humanism" has two very different meanings. It is now used to describe the belief
that human beings are self-sufficient and have no need for God. During the Renaissance,
however, humanism simply referred to the adaptation of classical learning to the needs of the
present world. Renaissance Humanism introduced a new and more positive view of humankind
and also a spirit of critical enquiry that would, eventually, lead to a profound questioning of
received ideas.
Which one of the following can you infer from the given argument?
(a) In its modern sense, humanism is incompatible with religious belief.
(b) Renaissance Humanism was at loggerheads with faith in God.
(c) Renaissance Humanism led to the revival of received learning.
(d) Latin and Greek culture are alive in terms of their present-day relevance.
(e) All of the above
17-18
After Zee news, it has been Zee Music‘s turn to go digital. Though this might lead to sharper
pictures and better audio quality, what is not so clear is Zee Music‘s inability to establish itself
as the ‗Indian Hindi Family Music Channel‘ as it so dearly wishes.
I. Other TV channels that have gone digital have been able to forge their way ahead of rival
channels.
II. Since channel ―etc‖ is in doldrums, Zee Music stands a good chance of establishing itself
as the ‗Indian Hindi Family Music Channel‘.
III. MTV, quite a well-established music channel has lost its discerning music buffs to channel
V.
IV. Zee Music has always had a low number of viewers and its going digital will not enable it to
compete with the reigning music channel – channel V.
18. Which of the above weakens the argument in the above passage?
(a) I and II (b) only II (c) only I (d) II and III (e) I and III
129 Pankaj Gandhi’s Academy/Logical Reasoning
19. L.K. Advani‘s security has been upgraded. Till now only the routes taken by the presidential
and prime ministerial motorcades were cleared of traffic and sanitised. Since last Wednesday,
the home minister, Mr. L.K. Advani, has been given a similar privilege. What some wags want
to know is what happens next when prime minister Vajpayee undergoes a knee surgery and is
expected to be indisposed for a month.
Which of the following can be inferred from the above passage?
(a) L.K. Advani will take over as PM during Vajpayee‘s period of recuperation.
(b) A home minister is not usually entitled to the privileges enjoyed by the PM and the
president.
(c) L.K. Advani is expected to act as proxy PM when required.
(d) L.K. Advani will also get his knee operated.
(e) None of the above
20. The burning of oil, coal and other inflammable energy sources produces carbon dioxide. The
consequent increase in the level of carbon dioxide in the atmosphere is believed to be
responsible for half of the greenhouse effect. To stop further increase in the temperature
industrial growth must be slowed down or production processes must be changed.
Which of the following would weaken the above conclusion?
(a) Global warming would be a welcome thing in the cold countries of the world.
(b) In this fast moving era, industrial growth has to be accelerated and not slowed down.
(c) Exhausts from automobiles result in the shooting up of carbon dioxide levels in the
atmosphere.
(d) Use of alternative energy sources is not viable, because all energy sources contribute to
the increasing green house effect.
(e) All of the above
Exercise 8(C)
Mr. C. E. Lakhani was the Managing Director and a large share holder of the Olympia Bag
Manufacturing Company (P) Ltd., Mr. Lakhani had inherited the business from his father. However, it
was under his leadership that the company had first introduced to the country, fiber glass moulded
luggage presently sold under the brand name "Exclusive". Exclusive sales presently accounted for
55% of the Rs. 14,00,000/- Sales of the Company as against only 20% three years ago.
The Exclusive brand of fibre glass luggage was light in weight but had great strength. Because of
high unit cost the clientele of the Exclusive brand was largely confined to air travelers and first class
travelers of the railways. While sixty percent of the buyers purchased the baggage on their own, the
remaining forty percent had it gifted to them on birthdays, marriage etc.
Mr. Lakhani had always wanted to be a leader. He was wondering how he should evaluate the
proposal put forward by the agents of Aero-Spatiale the makers of Air Bus. Mr. Ram Chander from
Aero-Spatiale had called in the morning and had made an offer to help The Olympia Bag
Manufacturing Company (P) Ltd. to make special suit cases for Air Buses and remain the brand
leader.
Mr. Ram Chander explained that Indian Airlines had placed a firm order with Aero-Spatiale for the
three Air Buses with an option to purchase 2 more Air Buses Any time 2 years after the delivery of
the 3rd Air-Bus. He explained that unlike other air crafts that were in use on the domestic flight in
India, an Air Bus could carry 320 passengers compared with 125 passengers on a Boeing 737. When
a filled Air-Bus landed at an Airport and if all the passengers carried a suit case which was stored in
the cargo hatch, it might take anywhere from 2 hours to 4 hours for all the passengers to get back
their luggage. As against this in a Boeing 737 flight, all the passengers could get their luggage in 30
to 40 minutes' time.
However, the Air bus was built with a provision that the passengers could stow a special size
suitcase on the rack over them. If this was the only luggage that the passengers had, they could walk
away in less than 5 minutes time. Mr. Ram Chander wanted the company to manufacture this suit
case and offered to sell the drawings for the suit-case at a nominal cost of Rs. 8,000/-.
Mr. Lakhani questioned Mr. Ram on the size of the suitcase. While volumes of the proposed suit-
case and the most popular Exclusive suitcase were nearly the same, the new suit-case would be 36"
wider and 15" shorter in length and breadth. This meant that a coat of a normal adult could not be
folded into the suit case without an extra crease coming in. Even the pants would be creased. In
effect, the market for the suit case would be limited to those traveling on Air-Buses only.
Mr. Ram Chander said that I. A. planned to fly the 3 Air Buses on the Golden Triangle i.e. Delhi -
Bombay-Calcutta routes where the graphic was the heaviest. On being questioned, he said that if IA
opted to purchase 2 more Air-Buses it was likely to fly one on North East Sector and other on
Southern Section. Daily passengers carried on the Golden triangle was 2500 while on the North East
and Southern Sector it carried 600 passengers altogether. Each of the Air-Buses would make 3
flights on the Golden Triangle and only 2 flights each on N. E. and Southern Sector. This was
economically viable as an Air Bus Broke even at about 30% capacity utilization.
Mr. Ram Chander said that in France it was their experience that if no passenger carried the special
suit cases, delay could be between 2 hours to 4 hours. If 20% of the passengers carried it, delay
would be about 1 1/2 hours, whereas if 40% of passengers had the special suit-case delay would be
only 40 minutes. Beyond 40% the delay would not be reduced for the passengers not carrying the
special suit case.
The cost of making a set of dies for the new suit-case would be Rs. 50,000/-. The dies would last
10000 pieces. The raw materials for the suit case would cost the same as for the normal and the
selling price also could be the same. There would be no technical problems in producing the new
The following questions consist of items related to the preceding passage above. Consider each item
separately in terms of the passage and on the answer sheet blacken space.
(a) If the item is Major Objective in making the decision; that is, one of the outcomes or results
sought by the decision maker;
(b) If the item is Major Factor in making the decision that is a consideration, explicitly mentioned
in the passage, that is basic in determining the decision.
(c) If the item is Minor Factor in making the decision; that is a secondary consideration that
affects the criteria tangentially, relating to a Major Factor rather than to an Objective.
(d) If the item is Major Assumption in making the decision; that is, a supposition or projection
made by the decision maker before weighing the variables;
(e) If the item is an Unimportant issue in making the decision; that is, a factor that is insignificant
or not immediately relevant to the situation.
Exercise 9 (A)
DIRECTIONS: From the alternatives, choose the one which correctly classifies the four sentences as
F : Fact : if it relates to a known matter of direct observation, or an existing reality or
something known to be true.
J : Judgement : if it is an opinion or an estimate or anticipation of common sense or
intention,
I : Inference : its is a logical conclusion or deduction of something, based on the
knowledge of facts.
DIRECTIONS: On the basis of more than two statements two inference are drawn. You are to mark
(a), if only conclusion 1 follows
(b), if only conclusion 2 follows
(c), if both the conclusions follow
(d), if neither of the conclusion follows.
6. All hands are feet. All feet are shoes. All shoes are socks. All socks are sockets. Therefore
1. All hands are sockets.
2. All sockets are hands.
7. All lovers are dogs. Some dogs are hogs. Some hogs are logs. All logs are wood. Therefore
1. All lovers are wood.
2. Some woods are lovers.
9. All rivers are waterfalls. All waterfalls are seas. All seas are navigable. Therefore
1. All rivers are navigable.
2. All navigable places are rivers.
10. All cups are glasses. All glasses are mirrors. All mirrors are faithful. Therefore
1. All cups are faithful.
2. Some faithful things are cups.
DIRECTIONS:. Assuming the statement to be false, point out if the inference is (a) True, (b) False or
(c) Uncertain.
DIRECTIONS: In each of the following arguments one premiss is missing. Pick out from the answer-
choices the premiss which will complete the argument without imparting any fallacy to it.
18. Doctors are respected because they save the lives of the people.
(a) All respected save the lives of the people.
(b) All who save the lives of the people are respected.
(c) Only those who save the lives of the people are respected.
(d) None of these.
DIRECTIONS: In the following questions one statement is followed by two possible implications.
Study the and mark one of the following answer choices:
(a), Statement 1 is implied.
(b), Statement 2 is implied.
(c), Both the statements are implied
(d), Both the statements are not implied.
DIRECTIONS: In the following questions a conclusion is followed by two statements which give us
data for the conclusion. Study that data and the conclusion and mark.
(a), If statement 1 alone can bring us to the conclusion.
(b), If statement 2 alone can bring us to the conclusion.
(c), If both the statements 1 and 2 taken together can bring us to the conclusion.
(d), If the statements taken separately or together cannot bring us to the conclusion.
26. Conclusion : The government must find a solution to the border dispute.
27. Conclusion : The teachings of Lord Buddha are confined to India only.
28. Conclusion : A grown up daughter is a friend who can see into the mother's mind.
29. Conclusion : The causes of accidents are being investigated these days.
DIRECTIONS: Each question contains six statements followed by four sets of combinations of three.
Choose the set in which the statements are logically related.
DIRECTIONS: In each of the following questions one interrogative statements followed by two
arguments, one beginning with `yes' and the other with `no'. Mark:
(a), if only argument 1 is forceful
(b), if only argument 2 is forceful
(c), if both 1 and 2 are forceful
(d), if neither 1 nor 2 is forceful.
38. Are objective type-multiple choice tests a true test of a candidate's ability?
1. Yes, because these need in-depth study of the subject.
2. No, because guessing also plays an important role in these tests.
39. Should the degrees of the teachers and doctors be valid for only some period and for
retaining these should they again take examination?
1. Yes, because these professionals must be aware of the latest developments in their
fields and periodic examinations is one method of keeping their knowledge up-to-date.
2. No, because no such discrimination should be made.
41. The day Monica accepts a gift from him, she will fall in love with Bill. If Bill does not give
Monica a gift, he will give it to Paula. If Hillary comes to learn of Bill‘s generosity, Bill will not
give Paula any gift. Unfortunately, Hillary comes to know about Bill‘s largesse from the New
York Times. What happens then?
(a) Bill gives Monica a gift.
(b) Bill and Monica fall in love with each other.
(c) Hillary takes drastic measures against Bill.
(d) Bill does not give any gifts to anyone.
42. Once the Women‘s Reservation bill is passed, Sharat Jadav will be very angry but Sushma
Garaj will be ecstatic. If it is not passed, the Parliament will have lesser number of women
and Sushma Garaj will not get a seat in it. If Sharat Jadav gets a seat in Parliament, the
Women‘s Reservation bill has not been passed. Which of the following is true if Sharat Jadav
does not get a seat in Parliament?
(a) The Women‘s Reservation bill has been passed.
(b) Sushma Garaj gets a seat in Parliament.
(c) The Parliament has more number of women.
(d) None of the above.
43. If Naseeruddin Shah does not act in a Govind Nihalani movie, then Om Puri will. If Om Puri
refuses to do an Aziz Mirza movie, Shah Rukh Khan will step in to do it. If Om Puri does not
act in the Govind Nihalani movie, Aamir Khan will be the replacement. If Aamir Khan does
not act in the Govind Nihalani movie, neither will Shah Rukh or Naseeruddin Shah.
Which of the following is true if Shah Rukh Khan acts in a Govind Nihalani Movie?
(a) Aamir Khan will act in it too.
(b) Naseeruddin Shah acts in it too.
(c) Om Puri does not act in the movie.
(d) All of the above.
44. If the Titanic sinks, people will lose their lives. If it does not sink, it will not become legendary.
If it does not become legendary, there will be no hit movie made on it. If it does become
legendary, James Cameron gets a job. Which of the following happens if the Titanic
becomes legendary?
(a) It sinks.
(b) People lose their lives.
137 Pankaj Gandhi’s Academy/Logical Reasoning
(c) James Cameron gets a job.
(d) All of the above.
45. If Havovi teaches Inductive Reasoning, Shantanu teaches Deductive Reasoning. If Shantanu
does not teach Verbal Ability, Havovi does so. If Havovi does not teach Reading
Comprehension, Shantanu does not teach it either but teaches Deductive Reasoning
instead. Which of the following is true if Shantanu decides not to teach Deductive
Reasoning?
(a) Havovi teaches Reading Comprehension but not Inductive Reasoning.
(b) Havovi teaches Inductive Reasoning but not Reading Comprehension.
(c) Shantanu does not teach Verbal Ability but Havovi does so.
(d) Havovi teaches Reading Comprehension and Inductive Reasoning.
DIRECTIONS: Given below is a passage followed by several inferences. Examine the inferences
separately in the context of the passage, and determine whether they are true or false. Mark
(a) if the inference is definitely true, i.e. it properly follows from the statement of facts given.
(b) if the inference is probably true, though not definitely true, in the light of statement of facts
given.
(c) if the inference is uncertain, i.e. data is insufficient to decide whether the inference is true or
false.
(d) if the inference is probably false, though not definitely false, in the light of statement of facts
given.
(e) if the inference is definitely false, i.e. it cannot possible be from the statement of facts given.
As many as 42 per cent of the rural households in Maharashtra have an annual income below
Rs.4,000. This was revealed in a survey. Another finding is that 77 per cent of people lived below the
poverty line in the state. The state government is now planning to undertake a fresh study to see the
change that has come about since the present government came to power. But this time the figure for
determining the poverty line has been raised to Rs.20,000 from Rs.11,000. Officials say that people
do not often reveal their true income in order to get the benefits of the anti-poverty programme.
47. The state government does not care for the poor people of the state.
48. The government has ulterior motives in raising the poverty line.
49. At least one survey has definitely been taken to gauge the wealth of Maharashtrians.
50. Many Maharashtrians lie to get the benefits of the anti-poverty programme.
Exercise 9(B)
1. The increasing reliance on computers represents a serious threat to the privacy of the
individual. Recently we have seen numerous examples of teenage and other ‗hackers‘
breaking the security codes of banks and social security agencies and getting sensitive
financial information about people.
The argument above depends upon which of the following assumptions?
I. People who obtain sensitive financial information about others will not share it.
II. It is not possible to develop a security system for a computer that cannot be broken.
III. Computers are not more efficient than other methods of record keeping.
(a) I only (b) II only (c) III only (d) I & III only (e) II & III only
2. There is no real need for teenage violence counselling. We should, instead, channelise the
money spent on these programs into marriage counselling and other programs designed to
hold the family together. If we succeed in lowering the rate of family dissolution, we will
automatically lower the incidence of teenage violence and abuse.
The author makes which of the following assumptions?
I. Marriage counselling and other programs are effective in keeping the family united.
II. Divorces and other family dissolution exacerbate the problem of teen violence and abuse
III. People presently working as teenage violence counsellors can be reemployed as marriage
counsellors
(a) I only (b) II only (c) I & II (d) II & III (e) I, II & III
3. In order to increase the number of autorickshaws available during the night between 1 p.m.
and 6 a.m., the auto commission, which sets fares has proposed a ten percent surcharge
during those hours. This is unlikely to have any effect on the availability of autos. The fleet
owners who lease autos to the drivers on a shift basis will simply raise the per-shift charge for
those hours, thus eliminating any incentive for the individual driver to work during night hours.
Which of the following, if true, most weakens the argument above?
(a) Fleet owners usually have unleased cars during the night hours covered by the
commission‘s proposal.
(b) Most people who need a taxi during the night would not mind paying the ten percent
surcharge.
(c) The majority of licensed autorickshaws are owned by individual private owners who do not
lease them from fleet owners.
(d) The previous year, the commission approved a five percent hike in auto fares.
(e) Passengers would not treat the surcharge as part of the driver‘s tip.
4. A landlord was recently found guilty of hiring thugs to harass legal tenants to force them from a
decaying building so that the apartments could be renovated and sold as high-priced co-
operatives. The judge sentenced the landlord to convey ownership of the property to a non-
profit organisation that would convert the building into housing for the homeless. Yet, the
sentence will not deter other landlords in the future from trying similar strong-arm tactics.
The argument above is most strengthened if which of the following is true?
(a) The current tenants will be allowed to live in their apartments even after the transfer of
ownership.
(b) The value of the unrenovated building to the landlord was virtually nothing.
(c) The landlord could have continued to make profits on the rental units in the building without
even renovating the property.
(d) Tenants have clearly defined rights under the landlord-tenant law including the right to quiet
enjoyment of the premises.
(e) A building begins to decay only when the rental income from the property is insufficient to
justify further investment in the property.
6. Based on the same question, if O‘hara received more votes than Powell, but Qureshi did not
win the prize, which of the following must be true?
(a) Rushdie won the prize.
(b) Theroux won the prize.
(c) Rushdie received more votes than Seth.
(d) McEwan did not receive more votes than Naipaul.
(e) Naipaul did not receive more votes than Rushdie.
7. Sauces and noodles manufactured in places like China and Taiwan gained popularity in India
in the 1980s primarily because they cost less than their local counterparts. Recent changes in
the foreign exchange regulations and currency status have increased the prices of the
imported items relative to the Indian products. However the sales of foreign sauces and
noodles have not declined in recent years.
Which of the following, if true, would help to explain why the sales of imported noodles and
sauces remain high?
(a) Trade ministries in the foreign nations have pursued policies that prevented prices of
noodles and sauce products from rising even faster.
(b) The cost of manufacturing noodles abroad is rising faster than it is in India.
(c) A coming recession in the Indian economy is expected to depress sales of imported
products in the near future.
(d) Indian consumers now perceive the quality of imports as being high enough to justify the
increased prices.
(e) Efforts to convince the Indians to buy India based products for patriotic reasons have yet to
bear fruit.
8. Young people who imagine the life of an actor to be one of glamour, money and fame, soon
discover not only the difficulties of the craft but the long odds against achieving any measure
of recognition or financial security. On being asked, ―Aren‘t most critics failed actors?‖ a
renowned thespian is said to have remarked, ―Yes, but so are most actors.‖
The statement by the thespian conveys which of the following ideas?
(a) A critic‘s job can be just as creative and challenging as acting.
(b) Few actors are fortunate enough to attain real success in their profession.
(c) For an actor success is measured more by influence exerted than by material gains
achieved.
(d) Many actors find that a stint as a critic makes for a greater understanding of the craft.
(e) There are no clear-cut standards of success for actors but there are for critics.
10. Last year, the number of cases of workplace sexual harassment reported by women in offices
increased by 25 percent. Ironically, these figures have been cited with approval by women‘s
rights advocates.
Which of the following, if true, would explain the seemingly paradoxical approval of the
women‘s rights advocates?
(a) Easier access to legal redress and encouragement to women has caused the number of
unreported cases of sexual harassment to reduce dramatically.
(b) The rate of convictions in cases of office sexual harassment has increased significantly in the
last few years.
(c) Sexual harassment cases have been on the high priority list of most of the women‘s rights
advocates and social workers.
(d) The law has been suitably amended so that cases of sexual harassment of male employees
by female superiors are also taken up.
(e) Local judges have begun to take cognisance of the extent of the importance of this problem
in the offices in the city.
11. The use of body spray has been proved to have a detrimental effect on the ozone layer.
However, the number of body spray users has not abated which is a cause for concern for
many scientists.
Which of the following, if true, would provide support for the unabated use of body spray?
(a) Many people are unaware of the harmful effects of body spray on the ozone layer.
(b) Ozone-friendly body sprays are making inroads into the market.
(c) Body spray is not the only product that has a deleterious effect on ozone layer.
(d) None of (a), (b) and (c)
(e) All of (a), (b) and (c)
12. The Indian Olympic contingent of 70 is back with a solitary bronze medal. That a nation of one
billion should be found lacking in sporting talent is a matter of humiliation. It‘s about time the
nation encourages interest in sports, especially at the school level.
Which of the following, if true, would counter the above argument?
(a) A nation preoccupied with survival cannot be expected to spend time and energy on an
accessory like sport.
(b) Very few school children are willing to play in the hot sun.
(c) India should invest more in education rather than in sports.
(d) In India, people interested in sports are hard to find.
(e) All of the above
13. The states of Kerala and Tamilnadu provide free secondary education to their citizens. The
nation‘s literacy rate would considerably shoot up if the other states follow suit.
Which of the following, if true, would weaken the above argument?
(a) States are not obliged to provide free secondary education.
(b) Nothing in life comes without a price. So education must be paid for.
(c) Parents in Kerala and Tamilnadu are willing to make good use of free secondary education,
which is not the case in other states.
(d) Most adults in Kerala are educated. So literacy campaigns are more effective there.
(e) None of the above
14. Webchat is widely considered a pleasant way of making friends, exchanging information or as
a pastime. The flip side is the loss of privacy and of course, being taken for a ride, which is
often overlooked.
We can infer that
(a) Webchat is user-friendly.
(b) One has to be wary during a chat.
(c) There are other foolproof methods of entertainment like reading books, watching TV or
going out for a walk.
(d) Every useful gadget carries a negative concomitant.
141 Pankaj Gandhi’s Academy/Logical Reasoning
(e) All of the above
15. Herbal remedies have always been enormously popular in India, the seat of Ayurveda – the
oldest medical system in existence today. Ayurvedic medicines consist of herbs, which are
mild, non-invasive and yet effective, for instance, in areas like prevention of disease which
allopathic medicines are incapable of. Allopathic medicines come hand-in-hand with
troublesome side effects.
Which of the following would seriously weaken the above argument?
(a) Herbs can be effective only under the prescription of a trained Ayurvedic doctor.
(b) Herbal medicines like Garlic, ginger and biloba inhibit blood clotting which could prove fatal
in several cases, especially with diabetic patients.
(c) Most herbal medicines are part of our daily food and are therefore easy to take than gulping
down a bitter allopathic medicine.
(d) Since herbs have stood the test of time they are synonymous with ‗safe‘. There can be no
side effects.
(e) None of the above
16. It is no secret that a number of software technology companies commenced operations with
product development dreaming to be the leading player in the area. However, soon realising
that financial feasibility is crucial for the existence of the business, a majority of them shifted
focus to services.
Which of the following would strengthen the shift in focus?
(a) The success of a product depends largely on marketing which needs enormous financial
resource.
(b) Software technology companies need a strong financial back-up in case of a stock market
plunge.
(c) As the software field is visited by frequent changes in terms of advanced technology, the
software companies need money to make the necessary changes and move ahead.
(d) All of (a), (b) and (c)
(e) None of (a), (b) and (c)
17. For decades, Bollywood actresses have been content to play the deified goddess-glamdoll.
After years of being mass-produced from the same cookie-cutter, heroines are slowly shedding
Bollywood‘s strait-jacketed stereotypes. A new generation of educated, savvy and sassy
women are demanding substantial roles and are often getting them.
Which of the following would support the change in the roles played by Bollywood actresses?
(a) After decades, sense has dawned on Bollywood actresses.
(b) Bollywood actresses have been inspired by Hollywood actresses.
(c) Encouraged by urban audiences and refocused markets, heroines are willing to take risks
and tread new grounds.
(d) Bollywood actresses are reading profound books on movies, which gives them a better idea
of what a movie is all about.
(e) None of the above.
18. Many dislike a ―bad loser‖ i.e., someone who does not take defects lightly as people say he
ought to, because ‗sportsman spirit‘ demands it. I disagree. You should be a bad loser
because you will remember failure as a strong challenge.
Which of the following would seriously weaken the viewpoint of the author of the above
passage?
(a) Bad losers fuss and enact dramatics that are disproportionate with the actual situation.
(b) Bad losers put themselves under tremendous pressure that can impede progress.
(c) Bad losers have no motivation underlying their efforts.
(d) Bad losers have a desperate urge to thwart another person‘s success.
(e) All of the above
20. Dotcoms are changing the way India does business. A sizeable number of business ventures
are already chalking up immense profits. But they have not changed the lot of the agriculturists
who feed its huge population.
Which of the following would undermine the above argument?
(a) Scientists have deployed the net to draw a farmer‘s timetable, that will help the farmers
battle the unpredictable weather gods.
(b) Dotcoms serve as gateways to a wide market for the farmers.
(c) One-fourth of the profits made by the dotcoms goes into agricultural development.
(d) All of (a), (b) and (c)
(e) None of (a), (b) and (c)
Exercise 9(C)
In 1974, C. Banerjee , L. Mitra , J. Ghosh, C. Majumdar and Lakshmi Gangani formed the Women's
Collective Bakery Cooperative. The purpose of the cooperative was to supply the local area with
natural baked products, primarily whole wheat, cracked wheat, oat and rye breads, using no
preservative or artificial ingredients. The area served was a fifty- mile radius of Patiala, Punjab. The
venture was an immediate success and within a year the company was covering its costs and playing
a modest salary and profit shares to the principles. In 1975 the group formalized its organization and
assigned the following responsibilities: President and General Manager - C. Banerjee, Transportation
and distribution - C. Majumdar, Manufacturing - E. Gangani, Quality Control - J. Ghosh, Sales and
marketing - L. Mitra. In particular, Banerjee 's job included the accounting and controls negotiating
with the bank for investment and working capital and performing secretarial and switchboard duties;
Majumdar drove the delivery truck; Gangani did the actual baking; Ghosh purchased the ingredients
from local farmers and served as the cooperative's taster; and Mitra sold and collected the bills. By
the end of 1976 the reputation of the cooperative had spread by word of mouth as far as Calcutta to
the east and New Delhi. At the monthly operating committee meeting. Banerjee reported receiving
dozens of unsolicited orders from health food stores in those cities. She mentioned that the turned
down the orders because those stores were clearly outside WCB's distribution area. ‖Some of those
stores are offering substantial initial orders at a higher markup than we are getting from our local
customers", she said. ‖We can increase our business substantially if we gear up for the additional
production."
"We didn't go into this venture to make a lot of money," replied Mitra . We did it to supply good,
wholesome food to the locals and to become financially self-sufficient. I personally have no interest in
selling bread to city folk." At this Bannerjee asked every body to consider the issue and discuss it at
the next meeting.
At the operating committee meeting next month, Banerjee stated that she had received even more
requests this month. Majumdar, said "As for as transportation and distribution is concerned, to deliver
to Delhi and Calcutta we will have to consider two radical moves. One, we have to hire drivers. This
means that we will become bosses, which is not, I think, what we want to do philosophically.
Practically speaking, hiring people involves supervision, paperwork, payroll, income tax withholding
and reporting, and a bunch of other headaches. This is a women's cooperative and that implies two
more considerations: do we hire only women and do we hire at all? I mean as opposed to bringing in
new partners, which is what at the word "cooperative" really means. The second issue, obviously, is
the truck, or trucks, I should say, We still have to buy more trucks. Our truck is big enough to handle
the extra volume, but it can't be in two places at once. Where will we get the money for the new
vehicles? And think about this: the bread will not make it to New York without refrigeration. You know
how rye bread gets solid as a rock after 24 hours. The extra 8 hours to New York will cut the shelf life
by too much. And a refrigerated truck really costs money."
J.Ghosh, the quality control manager said, "Well, we don't really have to refrigerate the long
deliveries. A smidgen of sodium monochlorohydrate will extend the shelf life by a full day, and the
city folk won't even know the difference. We just have to be careful to keep our shipping straight and
make sure we ship the pure bread locally and the 'salted' bread long distance."
"Out of the question" shouted Mitra. "That's the worst suggestion I've ever heard. We are already
beginning to corrupt our values by considering expanding just to make more money. Look at what
happens; we start to put preservative in the bread. I say ,no way."
" As far as production is concerned, I can bake bread all day long ― said Gangani. ―And if I can't do it
all alone, my girls can come in after school and help out. We can bake all the bread you want."
"Look," interrupted Ghosh, "We can hire drivers like everybody else and we don't need refrigerated
trucks. And even if we do need the refrigerated trucks, we can borrow the money from the bank
based on the new orders with the higher markup. Also, even though we make more money that
"The money isn't the only issue," said Mitra . "First, we'd become a business instead of a cooperative;
second, we'd have to hire people and boss them around; third, we'd have to deal with strangers and
city people; fourth and worst, we'd have to put preservative in the bread. I'm against the suggestion; I
don't think we should even consider it."
Bannerjee put a sheet of paper containing the expansion analysis paper in front of the others. "Right
now we each take home Rs. 12,000. If we expand the market, based on what I think we can get and
how many we can sell, we can increase our take-home profit to Rs. 33,000 or Rs. 30,000.‖
EXPANSION ANALYSIS
Current Proposal 1 (preservative) Proposal 2(refrigerated Truck)
Revenues 100,000 300,000 300,000
Flour & other ingredients 16,000 33,000 32,000
Gas & tolls 23,000 75,000 75,000
Depreciation on truck(s) 1,000 4,000 18,000
Driver's salaries ----- 24,000 24,000
Total Costs 40,000 136,000 149,000
Total profit 60,000 164,800 151,000
Each share (1/5) 12,000 32,800 30,200
―The difference in profit here between the Rs. 164,000 and the Rs. 151,000 is the extra cost of the
depreciation on the refrigerated trucks. If we preserve the bread, we can use ordinary trucks. I threw
in Rs. 1,000 to the cost of the ingredients of Proposal 1 to cover the cost of the monosodium. At Rs.
33,000 each we are no longer merely self-sufficient; we are downright rich‖.
The following questions consist of items related to the preceding passage above. Consider each item
separately in terms of the passage and on the answer sheet blacken space.
(a) If the item is Major Objective in making the decision; that is, one of the outcomes or results
sought by the decision maker;
(b) If the item is Major Factor in making the decision that is a consideration, explicitly mentioned in
the passage, that is basic in determining the decision.
(c) If the item is Minor Factor in making the decision; that is a secondary consideration that affects
the criteria tangentially, relating to a Major Factor rather than to an Objective.
(d) If the item is Major Assumption in making the decision; that is, a supposition or projection made
by the decision maker before weighing the variables;
(e) If the item is an Unimportant issue in making the decision; that is, a factor that is insignificant or
not immediately relevant to the situation.
DIRECTIONS: In each of the following questions some statements are followed by two inferences.
Point out if:
(a), Conclusion 1 follows.
(b), Conclusion 2 follows.
(c), Conclusion 1 and 2 follow.
(d), Neither 1 nor 2 follows.
1. Statements : Both plants and animals show remarkable changes in form and structure, in
becoming adapted to different environments. This divergence changes the
form and functions of some parts of the plant or the animal.
2. Statements : The emancipation of woman has caused the greatest revolution. The
greatest fight had been the fight for the independence of woman. Woman
has succeeded because of her proverbial patience.
3. Statements : One phase of business cycle is the expansion phase. This phase is two fold-
recovery and prosperity. Then it reaches the highest point-this is the end of
this phase.
4. Statements : The youth are after the arduous quest of grasping new ideas and adopting
new values. We will have to adopt a positive view of their problems.
5. Statements : Students nowadays are not really being educated. They are simply asked to
memorise facts. They do not develop critical thinking.
DIRECTIONS: On the basis of more than two statements two inference are drawn. You are to mark
(a), if only conclusion 1 follows
(b), if only conclusion 2 follows
(c), if both the conclusions follow
(d), if neither of the conclusion follows.
6. All bombs are explosives. All explosives are dangerous. All dangerous things are harmful.
Therefore
1. All bombs are harmful.
2. All harmful things are bomb.
8. All mangoes are sweet. All mangoes are yellow. All yellow things are ripe. No ripe thing is
eatable. Therefore
1. All mangoes are not ripe.
2. All mangoes are not eatable.
9. All players are children. All children are boys. All boys are not girls. No girl is a father.
Therefore
1. No player is father.
2. No father is player.
10. All women are cows. All cows are buffaloes. All buffaloes are goats. All goats are pigs.
Therefore
1. All women are pigs.
2. All pigs are women.
DIRECTIONS: Assuming the statement to be true point out if the inference is (a) True, (b) False or
(c) Uncertain.
DIRECTIONS: In each of the following arguments one premiss is missing. Pick out from the answer-
choices the premiss which will complete the argument without imparting any fallacy to it.
16. He sings well because he has a sweet voice.
(a) All with sweet voice can sing well.
(b) Only people with sweet voice can sing well.
(c) Some who have sweet voice can sing well.
(d) Some who can sing well have sweet voice.
17. India is an agricultural country because India is a poor country.
(a) Only poor countries are agricultural countries.
(b) All poor countries are agricultural countries.
(c) Some poor countries are agricultural countries.
(d) Some agricultural countries are poor countries.
18. He is imaginative so he can write poems.
(a) Only imaginative can write poems.
(b) All who can write poems are imaginative.
(c) None who can write poems is imaginative.
(d) None of the above.
DIRECTIONS: In the following questions one statement is followed by two possible implications.
Study them and mark one of the following answer choices:
(a), Statement 1 is implied.
(b), Statement 2 is implied.
(c), Both the statements are implied
(d), Both the statements are not implied.
21. He that hurts another hurts himself.
1. A man should not hurt anyone.
2. All men are alike.
22. Ignorance of law is no excuse.
1. One should not be ignorant of law.
2. Man tries to find an excuse.
23. By giving up smoking the incidence of lung cancer can be reduced.
1. The incidence of lung cancer is very high.
2. Smoking causes lung cancer.
24. Wood is the most valuable building material because it is durable and strong.
1. In addition to wood there are other building materials.
2. Wood is durable.
25. Dental infection is still the most common human disease.
1. Dental infection is a disease.
2. Dental infection is an old disease.
DIRECTIONS: In the following questions a conclusion is followed by two statements which give us
data for the conclusion. Study that data and the conclusion and mark.
(a), If statement 1 alone can bring us to the conclusion.
(b), If statement 2 alone can bring us to the conclusion.
(c), If both the statements 1 and 2 taken together can bring us to the conclusion.
(d), If the statements taken separately or together cannot bring us to the conclusion.
26. Conclusion : A European peace order is no longer a vision.
Data : 1. People suffer when expenses increase due to high prices and
income remains the same.
2. Prices have rise abnormally.
DIRECTIONS: From the alternatives, choose the one which correctly classifies the four sentences as
F : Fact : if it relates to a known matter of direct observation, or an existing reality
or something known to be true.
J : Judgement : if it is an opinion or an estimate or anticipation of common sense or
intention,
I : Inference : it is a logical conclusion or deduction of something, based on the
knowledge of facts.
32. 1. You must know not only your rights but also your obligations.
2. Time never stands still.
3. Greta Garbo was a successful actress.
4. It takes a thief to catch a thief.
(a) JIFF (b) JFFI (c) JIFF (c) JFII
33. 1. God made the country and man made the town.
2. Caste plays an important role in Indian political life.
3. If we subdue our ego, we will all be happy.
4. Artificial insemination is a boon to childless couples.
(a) JJIF (b) IJJF (c) FIFJ (d) FFIF
DIRECTIONS: Each question contains six statements followed by four sets of combinations of three.
Choose the set in which the statements are logically related.
DIRECTIONS: Given below is a passage followed by a few statements. You have to read the
passage and on the basis of the information available, mark your answer as
(a) if the argument is an upstream argument i.e. if the passage is a direct conclusion or inference of
the question statement
(b) if the argument is a downstream argument i.e. if the question statement is a direct conclusion
or inference of the passage.
(c) if the argument is a lateral argument i.e. if the statement supports the passage but is neither
upstream nor downstream.
(d) if the argument is irrelevant to the passage.
Prasar Bharati‘s board has exempted private producers of news and current affairs programmes on
Doordarshan from submitting tapes for a preview, and, consequently, from pre-censorship.
41. Doordarshan has, for long, been a slave to government regulations and its eagle eye.
42. Doordarshan is bold in its role as an eye-opener for the common people.
43. Prasar Bharati has felt the need to be more mature in its attitude towards news dissemination.
44. Private channels are competing with each other to beam better programmes.
45. The new strident tone of newscasters has raised hackles in the corridors of power.
47. The house is infested with rats. The pied piper has to be called to eradicate them. The piper
charges an exorbitant rate per rat. But eradication of the rats will also lead to eradication of
the cats. The children love the cats. Either the pied piper is paid an exorbitant amount of
money or the pied piper makes the children happy.
A. the pied piper makes the children cry.
B. the pied piper is paid an exorbitant amount of money.
C. the pied piper does not make the children happy.
D. the pied piper is not paid an exorbitant amount of money.
(a) AD (b) CA (c) BC (d) CB
48. Sonia wants to win in the elections at any cost. Mr. Buddha Milgaya has promised to help her
if she marries him. Either Sonia marries Mr. Buddha Milgaya or Sonia will be defeated in the
elections. Mr. Buddha Milgaya is allergic to winning elections.
A. Sonia will marry Mr. Buddha Milgaya.
B. Sonia will not win in the elections.
C. Sonia will not marry Mr. Buddha Milgaya.
D. Sonia will win in the election.
(a) DC (b) AB (c) CB (d) None
49. In the 35 sectors where Foreign Direct Investment (FDI) upto 51 per cent of equity is
automatically allowed, the BJP is unlikely to discourage investment.
Which of the following statements, if true, weakens the above argument?
(a) In industries requiring government sanction, the BJP could stall approvals.
(b) No company will be asked to leave India but will be made to feel unwelcome.
(c) The BJP will ban 100 per cent foreign owned subsidiaries in non-infrastructure
industries.
(d) The ideological hawks in the party might cause practical problems for foreign companies
investing in these sectors.
50. There will always be situations when a manager must reverse himself, but cannot hide behind
subtle tactics.
Which of the following statements, if true, strengthens the above argument?
(a) The manager‘s decision penalises parts of the company in the short run.
(b) The manager has the right to occasionally invoke his right to be arbitrary.
(c) Junior employees trust the manager.
(d) Both a and c
Exercise 10(B)
2. To which of the following would the author compare the ―new born baby‖ mentioned in the
passage?
(a) A technological breakthrough based on pure research.
(b) Government funding in support of scientific endeavours.
(c) Public support for spending on pure research.
(d) The flow of public benefits from new technological developments.
(e) A scientific discovery with no immediately apparent application
3. All those in last years' Mathematics faculty have gained admission into the IIMs. Changu and
Mangu also paid their admission fee for the IIM(L) yesterday.
The above statement implies that:
(a) Changu and Mangu are not part of the Mathematics faculty.
(b) IIM(L) is one of the IIMs.
(c) Paying your fee at IIM(L) implies that you have gained admission to the IIMs.
(d) Only the Mathematics faculty makes it to the IIMs.
(e) Changu and Mangu have taken admission into the IIM(L)..
4. If Nana Patekar acts in "Muharaam", Amitabh Bachchan will act in the film too. If Tabu acts in
the film, it will be a hit. If Amitabh Bachchan acts in the film Tabu will not act.
If "Muharaam" is not a hit
(a) Nana Patekar does not act.
(b) Amitabh Bachchan acts.
(c) Tabu does not act.
(d) Both (a) and (c)
(e) Both (b) and (c)
5. Violence and lawlessness have spread all over the country. They are like a river, which breaks
its banks and overflows in every direction. Its water is of little use. It can only cause harm to
the people. Such is the present state of affairs.
The statement by the speaker conveys the idea that: -
(a) Violence can be curbed if proper measures are taken.
(b) An overflowing river is comparable to violent people.
(c) Violence and lawlessness implies that the government has failed to maintain law & order in
the nation.
(d) Peace is essential for accomplishing anything.
(e) The Indian Judiciary needs to rethink and make necessary changes in the laws to avoid the
loopholes in it.
7. When they came into India, all the 3 shoe giants Reebok, Nike and Adidas started with the
assumption that their exclusive brands demanded exclusive stores. Their reasoning: their
brands would grab sales through destination shopping, where customers would travel to a
store specifically to buy a pair of their shoes. But the premise was faulty, as low sales showed.
Which of the following would rectify the faulty premise and resulting low sales?
(a) They would have to sell their products in a multibrand environment, so that the Indian
customer who is not committed to one brand can try them out.
(b) They would have to set up distribution and sales offices and begin a survey on the Indian
market scene.
(c) They would have to concentrate more on their commercials, making it less global and more
India specific.
(d) They went wrong in not taking into account rural India, which accounts for around 20% of
their customers who have never heard of these brands.
(e) Since they are competing with time tested Indian brands they would have to make their
stores and products exclusive.
8. Selection of news leads to suppression of news. Therefore, careless and uncritical reading of a
newspaper may lead us astray.
Which of the following statements has logic similar to the one given in the question?
(a) Selection of a friend, if wrongly done, leads us to disaster. Therefore, friends have to be
carefully selected.
(b) High aspiration is generally wild hope, which is a recipe for disaster. Hence, it should be
abandoned.
(c) Commercial advertisements praise the qualities and ignore the defects of a product. So the
unwary customer might be tricked into buying it.
(d) Driving fast can lead to accidents and death. Hence driving has to be done very carefully
and slowly except in highways.
(e) Scanty rainfall causes drought. Scanty rainfall is caused because a large number of trees
are cut down every year. Use of wood has to be reduced.
9. Tony Greig: Chris Cairns is the best New Zealand all-rounder. No wonder cricket fans call him
‗Kiwi No. 1‘.
Geoff Boycott : You are wrong. Gavin Larsen‘s performance with the ball and the bat, in the
World Cup was better than that of any other player in the team.
Geoff Boycott apparently assumes that Tony Greig said:
(a) Only a player from the New Zealand team is eligible to be the world‘s best all-rounder.
(b) Chris Cairns has the No.1 Ceat rating currently.
(c) Chris Cairns is the best all-rounder in the World Cup for the New Zealand team.
(d) A player is called an all-rounder when he is consistent in batting, bowling and his fielding.
(e) Gavin Larsen‘s fielding is relatively sloppy.
11. The English language has a lot of pejorative terms describing women, such as ‗harridan‘,
‗termagant‘, ‘virago‘, ‗witch‘, ‗governess‘ and so forth, for which there are no male equivalents.
Which of the following can be inferred from the above passage?
(a) Women actually epitomise the qualities described by these pejorative terms.
(b) No English man ever possessed the negative qualities denoted by these terms and hence
the necessity for male equivalents of these derogatory terms is ruled out.
(c) These derogatory terms describing women point to the underlying male chauvinism of the
English society
(d) All of (a), (b) and (c)
(e) None of (a), (b) and (c)
12. Medical research conducted by doctors highlights the fact that improperly designed chairs are
one of the major causes of back pain. Pain for which, very often, the only recourse is bed-rest
and pain killers. For your organisation, this means lower employee productivity, higher
absenteeism, mounting medical pills and other hidden costs.
Which of the following is a strengthening argument?
(a) Companies are now asking their employees to stand and work in split shifts of four hours.
(b) Companies are changing furniture and switching to ergonomically friendly chairs for many
employees.
(c) In fact more than three-fourths of all people suffer from back pain and most back pain can
be attributed to improperly designed chairs.
(d) Research has proved that people who sit in badly designed chairs are more prone to back
ache than workers who stand for long hours.
(e) None of the above
13. Blazing Blazers lend a depth, character, elegance and distinctive tone to both the wearer and
the shade he chooses.
Which of the following may be an assumption underlying the above argument?
(a) All people with depth of character wear Blazing blazers
(b) All Blazing Blazers can lend a depth and elegance to the wearer
(c) All who wear Blazing Blazers acquire depth of character.
(d) All of (a), (b) and (c)
(e) None of (a), (b) and (c)
14. The cliché that no book could be a better companion to Sicily than 'The Leopard', like most
clichés, is true. This novel must be one of the most under-read masterpieces of the last
century.
Which of the following statements would strengthen the above claims?
(a) Most books about wildlife say a great deal about universal human truths and hence are
useful on trips abroad.
(b) Clichés are statements which seem trite but on second reading carry the unerring force of
truth.
(c)' The Leopard' has had the highest sales in the last 20 years at the Sicilian airport and
railway station.
15. The reforms that took place in 1991 were in one sense panic reactions rather than considered
actions. The immediate provocation was that we were about to claim default on our
international obligations with reserves down to about one month's imports. Our first response
was to put a freeze on imports.
Which of the following would be a logical conclusion to the above paragraph?
(a) When the immediate crisis was averted we needed to dismantle the complex system of
industrial licensing.
(b) Once the panic subsided we realised that proactive steps like building an infrastructural
base were more beneficial than stop gap reactions.
(c) The reforms were not thought out and yet we have emerged comfortably at the end of one
decade.
(d) Economic reforms even at that stage were our only way out.
(e) All of the above
16. There are apprehensions in the wake of the software slump, that the rejuvenated publishing
industry will go the same way, only sooner. The bookstores have been swamped by new
writers and scores of new readers so that suddenly sales are up by almost a 100%. Despite
the fear, booksellers are making hay while the sun shines.
Which of the following, if true would reinforce the fear voiced by the author in the above
paragraph?
(a) Already, bookshops in the suburbs are showing downward trends after the initial
celebratory buying spate.
(b) Publishers confess that although sales are high for some kinds of fiction the overall sale in
books has declined rather than increased.
(c) Libraries in the town are closing down because no one goes there to read any more.
(d) Bookstores no longer have a place on the NYSE.
(e) None of the above
17. In the present scenario the importance of cultural roots cannot be overemphasised. As people
increasingly turn to their families and communities for emotional support it is the set of cultural
mores and norms which serves as the framework for mental health.
Which of the following if true would weaken the above argument?
(a) Culture cannot be preserved by the state and will die if the family as the foundational unit of
society, breaks up.
(b) Increasingly the family in India is being replaced by a set of structures including
international law and the geographical set up as a unit.
(c) Most people spend a majority of their time in the work arena and form support groups there
based on common interests rather than on family ties.
(d) It is necessary for people of the same country to bond with each other.
(e) All of the above
18. Several of their sons died in a war that we claim to have won. Now these old and completely
frustrated parents are asking for justice. How can we look them in the eye once we have
shaken hands with the very person responsible for their loss?
The above argument assumes that
(a) We are going to shake hands with a murderer.
(b) Parents who have lost their children often seek ways to avenge their loss.
(c) Justice can be done only when we can face these parents with a clear conscience.
(d) All of (a), (b) and (c)
(e) None of (a), (b) and (c)
20. Stokes did not challenge the caste system head-on. But his grand daughter believes that his
decision to convert marked the beginning of the end of caste-ridden prejudices in the area.
Which of the following, if true, would most strengthen the above argument?
(a) Stokes‘ grand daughter has had extensive experience with people who are religious
converts.
(b) By not posing a direct threat to caste politics, Stokes struck at the very root of the bias in
society.
(c) Conversion poses a sure threat to people with rigid caste beliefs.
(d) Stokes believed in the essential power of non-violence.
(e) All of the above
Exercise 10(C)
Since Mr. Krishan, a young management graduate had been recently appointed as the chief
executive officer of Coirfoam, a small company producing foam rubber mattresses. Coirfoam was in
financial trouble, and its owners wanted to sell it. The company had been established some twenty
years ago, but its market share had steadily declined over the last five years. Since Mr. Krishan had
no previous experience in the mattress business, he requested his friend, a marketing consultant, to
tell what he could do about it.
His friend analyzed the company's resources. Its best resources was its product and brand name.
However, synthetics are much cheaper than foam rubber mattresses. Latex mattresses are known for
their orthopaedic and antiallergic qualities, among others. The Coirfoam brand name had very nearly
become a generic term for all types of rubber mattresses. Coirfoam, however was the only latex
mattress produced locally.Apart from a superior product, the company had few resources. Its
equipment, though satisfactory, was old. It operated in leased premises on a year-to-year basis,
although the landlord was willing to conclude a long-term agreement on favourable terms. On the
other hand, the company's labour force was experienced and dedicated and its production manager
had more than ten years' experience in latex manufacturing.
The Chairman of the company, was seventy years old and wanted to retire. Apart from the sales
manager, no one else shared responsibility for marketing or administration. He told Krishan's friend
that if the company were sold, he had no intention of remaining in service; he had eighteen years of
experience in the mattress industry. If the sales manager left the company Krishan might not find a
suitable replacement. This was another issue that Krishan's friend had to study.
Coirfoam's financial position was precarious. The company was heavily in debt and its line of credit
fully extended. There was some question as to whether the company would be able to purchase
enough latex to keep production going, but the manager assured that the company had a bank letter
of credit to purchase an additional three month's supply.In spite of the Chairman's optimism, the fact
was that his company had steadily lost market share. Once the dominant mattress manufacturers,
with fifty percent of the local market, its market share had declined to less that ten percent. The
chairman attributed this decline to popularity gained by spring mattress manufacturers, who had only
begun production five years ago. Spring mattresses now accounted for seventy per cent of the total
market, another company ten percent, with the remaining twenty percent shared by a number of small
plants producing synthetic rubber mattresses. Spring mattresses had some attributes similar to those
of foam rubber, such as orthopaedic qualities. They were less costly to manufacturer but sold to the
consumer at about the same price as Coirfoam mattresses.
Because of Coirfoam's financial difficulties, it ceased advertising in newspapers and on radio for over
the past five years. As a result, retailers were reluctant to handle their product. In contrast to it two
spring mattress manufacturers had advertised heavily in the mass media. One of these
manufacturer's products was sold exclusively by the largest furniture chain in the country. During this
study of the mattress market, a number of retailers had expressed the opinion to Krishan's friend that
a whole generation of young people were largely unaware of Coirfoam product because of the lack of
advertising. One retailer was quoted as saying: "It is true that older people remember Coirfoam, but
these mattresses last for almost twenty years. The big market is not the replacement market, but
sales generated by family formation. Thousands of young couples get married every year, and every
marriage means another mattress sale. But these young people only see advertisements of spring
mattresses. It is obviously easier for my salesman to sell a mattress which his customers have seen
in countless advertisements that one which is relatively unknown."
Krishna's friend was aware of the fact that if Coirfoam was ever to regain some of its lost market
share, it would have to launch a major advertising programme to educate young adults about the
important attributes found in its products. A major question that needed an immediate answers was:
"To what extent are people aware of Coirfoam mattresses and their attributes?" Other questions
involved the attitudes of people toward foam rubber mattresses in general and how these attitudes
On the basis of the preliminary research Krishan was optimistic that he could turn the company
around. In support of his belief, he cited the recognition of the company among a significant portion
of the population, and the fact that they would buy a Coirfoam for their children. He believed that
once retailers became aware that new management had taken over the company, they would be
willing to stock the product. Krishan was aware that the research findings were not always in
agreement with his conclusions. However, the finding that young people were relatively unaware of
Coirfoam did not seem to worry him. He felt that a well-designed advertising program would convince
many people to buy a foam rubber mattress, rather than any competing type. Moreover, the
introduction of new management team would instill confidence among Coirfoam bankers. Credit lines
would be increased, thereby improving the company's financial position. However, before making a
final decision as to whether to purchase Coirfoam, Mr. Krishan waited for his friends final report and
recommendations.
The following questions consist of items related to the preceding passage above. Consider each item
separately in terms of the passage and on the answer sheet blacken space.
(a) If the item is Major Objective in making the decision; that is, one of the outcomes or results
sought by the decision maker;
(b) If the item is Major Factor in making the decision that is a consideration, explicitly mentioned in
the passage, that is basic in determining the decision.
(c) If the item is Minor Factor in making the decision; that is a secondary consideration that affects
the criteria tangentially, relating to a Major Factor rather than to an Objective.
(d) If the item is Major Assumption in making the decision; that is, a supposition or projection made
by the decision maker before weighing the variables;
(e) If the item is an Unimportant issue in making the decision; that is, a factor that is insignificant or
not immediately relevant to the situation.
Exercise 1-A
1-a 2-a 3-d 4-d 5-d 6-d 7-d 8-a 9-b 10-a
11-b 12-a 13-b 14-c 15-a 16-c 17-d 18-b 19-d 20-b
21-c 22-d 23-c 24-a 25-a 26-b 27-a 28-c 29-d 30-b
31-d 32-d 33-b 34-d 35-d 36-a 37-d 38-a 39-a 40-d
41-d 42-a 43-a 44-b 45-c 46-c 47-b 48-b 49-e 50-a
Exercise 1-B
1-b 2-e 3-d 4-b 5-c 6-a 7-b 8-b 9-a 10-a
11-b 12-d 13-c 14-a 15-c 16-a 17-a 18-b 19-e 20-e
Exercise 1-C
1-e 2-b 3-a 4-e 5-b 6-c 7-c 8-b 9-a 10-d
Exercise 2-A
1-d 2-d 3-a 4-d 5-d 6-b 7-a 8-b 9-b 10-a
11-c 12-c 13-c 14-a 15-b 16-a 17-a 18-c 19-b 20-d
21-a 22-d 23-d 24-c 25-c 26-d 27-b 28-a 29-b 30-c
31-a 32-d 33-d 34-a 35-a 36-c 37-d 38-d 39-c 40-c
41- a 42- b 43-d 44- d 45- d 46- b 47- b 48- d 49- c 50- d
Exercise 2-B
1-e 2-c 3-a 4-a 5-a 6-c 7-e 8-e 9-d 10-c
11-c 12-b 13-c 14-b 15-c 16-b 17-b 18-b 19-d 20-b
Exercise 2-C
1.a 2.d 3.b 4.c 5.c 6.e 7.d 8.b 9.a 10.d
Exercise 3-A
1-d 2-d 3-b 4-a 5-a 6-c 7-c 8-c 9-a 10-b
11-b 12-a 13-c 14-c 15-d 16-c 17-a 18-b 19-b 20-d
21-c 22-b 23-b 24-d 25-d 26-c 27-d 28-a 29-d 30-c
31-d 32-d 33-d 34-c 35-d 36-c 37-b 38-d 39-a 40-b
41-a 42-b 43-e 44-b 45-d 46-d 47-d 48-d 49-a 50-d
Exercise 3-B
1-c 2-c 3-c 4-a 5-c 6-c 7-c 8-a 9-c 10-b
11-c 12-b 13-c 14-b 15-b 16-b 17-b 18-b 19-b 20-c
Exercise 3-C
1-b 2-d 3-a 4-e 5-b 6-b 7-b 8-e 9-c 10-b
Exercise 4-A
1-a 2-a 3-a 4-c 5-c 6-c 7-a 8-c 9-a 10-d
11-c 12-a 13-c 14-a 15-d 16-b 17-b 18-d 19-d 20-a
21-d 22-d 23-a 24-a 25-d 26-c 27-a 28-c 29-a 30-d
31-a 32-c 33-d 34-d 35-a 36-b 37-b 38-b 39-b 40-c
41- d 42- c 43- a 44- b 45- d 46- a 47- b 48- c 49- d 50- d
Exercise 4-B
1-a 2-c 3-a 4-d 5-a 6-d 7-b 8-a 9-c 10-c
11-d 12-e 13- c 14-d 15-c 16-a 17-c 18-c 19-e 20-d
Exercise 4-C
1-c 2-b 3-a 4-d 5-d 6-a 7-b 8-e 9-c 10-d
Exercise 6-A
1-d 2-d 3-c 4-a 5-b 6-c 7-d 8-c 9-a 10-c
11-a 12-c 13-a 14-d 15-d 16-c 17-a 18-d 19-d 20-d
21-b 22-a 23-a 24-a 25-d 26-b 27-a 28-a 29-a 30-d
31-a 32-d 33-c 34-d 35-b 36-a 37-b 38-a 39-d 40-b
41- c 42- c 43- b 44- c 45- c 46- c 47- d 48- a 49- b 50- d
Exercise 6-B
1-d 2-e 3-c 4-b 5-d 6-c 7-d 8-c 9-d 10-c
11-c 12-a 13-a 14-a 15-a 16-d 17-e 18-b 19-b 20-b
Exercise 6-C
1-d 2-a 3-b 4-c 5-b 6-d 7-b 8-e 9-d 10-c
Exercise 7-A
1-a 2-c 3-d 4-a 5-b 6-b 7-d 8-c 9-d 10-d
11-d 12-d 13-d 14-d 15-a 16-a 17-d 18-b 19-a 20-d
21-d 22-d 23-b 24-a 25-b 26-d 27-b 28-c 29-c 30-b
31-c 32-d 33-d 34-d 35-a 36-a 37-b 38-d 39-c 40-a
41-d 42-d 43-c 44-b 45-a 46-c 47-b 48-c 49-e 50-b
Exercise 7-B
1-c 2-c 3-a 4-b 5-e 6-e 7-a 8-a 9-e 10-b
11-b 12-b 13-d 14-e 15-c 16-c 17-b 18-d 19-c 20-d
Exercise 7-C
1-e 2-d 3-a 4-c 5-b 6-d 7-b 8-a 9-b 10-d
Exercise 8-A
1-c 2-d 3-a 4-b 5-d 6-d 7-d 8-b 9-a 10-d
11-d 12-a 13-d 14-b 15-a 16-a 17-d 18-d 19-a 20-b
21-b 22-a 23-d 24-d 25-c 26-c 27-c 28-a 29-b 30-c
31-d 32-d 33-d 34-d 35-d 36-d 37-b 38-d 39-c 40-a
41-c 42- b 43- b 44- c 45- a 46- b 47- a 48- c 49- d 50- d
Exercise 8-B
1-a 2-a 3-e 4-d 5-c 6-e 7-e 8-e 9-a 10-e
11-b 12-c 13-c 14-d 15-b 16-c 17-d 18-c 19-b 20-a
Exercise 8-C
1-e 2-d 3-d 4-c 5-d 6-d 7-e 8-e 9-e 10-d
11-c 12-c 13-d 14-e 15-c
Exercise 9-A
1-c 2-b 3-d 4-c 5-d 6-a 7-d 8-a 9-a 10-c
Pankaj Gandhi’s Academy/Logical Reasoning 160
11-c 12-c 13-c 14-c 15-a 16-a 17-d 18-b 19-a 20-b
21-a 22-d 23-c 24-b 25-a 26-d 27-d 28-d 29-d 30-d
31-b 32-d 33-a 34-b 35-c 36-a 37-a 38-d 39-a 40-c
41-a 42-d 43-a 44-d 45-a 46-b 47-d 48-c 49-a 50-b
Exercise 9-B
1-b 2-c 3-c 4-b 5-e 6-d 7-d 8-b 9-b 10-a
11-b 12-a 13-c 14-b 15-b 16-d 17-c 18-b 19-a 20-d
Exercise 9-C
1-c 2-e 3-a 4-d 5-b 6-e 7-b 8-c 9-a 10-b
11-b 12-a 13-a 14-c 15-e
Exercise 10-A
1-c 2-c 3-a 4-b 5-c 6-a 7-d 8d 9-d 10-a
11-c 12-c 13-c 14-c 15-b 16-a 17-b 18-d 19-b 20-b
21-d 22-d 23-b 24-c 25-c 26-c 27-d 28-c 29-d 30-d
31-b 32-b 33-d 34-d 35-a 36-a 37-b 38-c 39-a 40-d
41- c 42- d 43- a 44- d 45- d 46-d 47- d 48- c 49- d 50- b
Exercise 10-B
1-a 2-e 3-e 4-c 5-b 6-c 7-a 8-c 9-c 10-b
11-e 12-b 13-b 14-d 15-b 16-b 17-c 18-c 19-a 20-c
Exercise 10-C
1-b 2-c 3-c 4-d 5-e 6-b 7-b 8-e 9-d 10-c
11-e 12-e 13-d 14-b 15-e
1. Ans. (a)
UA + UA = UA/PA. So conclusion (1) follows. However, conclusion (2) goes against the
argument according to which all books are latches.
2. Ans. (a)
E CHE
C H
We notice that only some eagles are customers. So conclusion (2) does not follow.
3. Ans. (d)
PA + UA = PA. Conclusions (1) and (2) are UAs.
4. Ans. (d)
UN + UN = Inconclusive
5. Ans. (d)
P. There is no clear relationship specified between the sets of radios and table lamps as a
result of which there would be a lot of possibilities. ―All radios are table lamps‖ is not always
true and hence the inference would be termed uncertain.
Q.
T R
R T
E.G. E.G
.
(1) (2)
6. Ans. (d)
P.
M
D
W
W
W
Q. PA + PA = Inconclusive.
8. Ans. (a).
O
P.
P P/O C
(1) (2)
Q.
P
O
C
C
C
9. Ans. (b).
There are two rules that need to be used in a ―missing premise‖ question. According to the
Common Factor Rule, the missing premise should consist of sets ―S‖ and ―B‖. (―F‖ has
occurred twice in the argument whereas ―B‖ and ―S‖ have occurred only once.)
According to the Rules for Testing the Validity of the Argument, UA + UA = UA.
S B B
F S F
UA + UA = UA
So the missing premise is ―All those who share sorrows are beneficial‖.
UA + UA = UA
LC G G
M LC M
UA + UA = UA
R GB GB
It W R It
W
UA + UA = UA
H L L
P H P
UA + UA = UA
17. Ans. (d). Statement 1 is not implied because all that the statement says is a person who runs
fast will not run for a long time. Exhaustion is not implied as a cause. Statement 2 is not
implied because the statement does not impose any value judgement on the action of running
fast being good or bad.
18. Ans. (b). Statement 1 is not implied because the statement says that fortunate heads do not
ache. 2 is implied, as the statement says that they are fortunate people whose heads have
never ached. Therefore the statement automatically implies that there are people who aren't
fortunate because if there are fortunate people, there should be unfortunate people as well.
19. Ans. (d). 1 is not implied. The statement merely says that it is best to use few words. It does
not say how many people actually talk little. 2 is not implied, as the statement makes no
mention of silence.
20. Ans. (b). 1 is not implied, as the statement does not say that mending has to be done. It states
that mending can be done anytime. 2 is implied when you say that it is never too late to mend.
22. Ans. (d). Neither of the statements tells us that a bomb exploded in a classroom.
23. Ans. (c). 1&2 combine to give this conclusion. The argument is a syllogism.
24. Ans. (a). Statement 1 says that talented people are imperative for progress and that progress
is necessary for India. From this statement alone we can arrive at the conclusion.
25. Ans. (a). since 1 says that every development has a reaction, we can conclude that " there
developments" to will have repercussions.
Option (a) is eliminated because the argument has two ―particular‖ statements for propositions.
(PA + PA = Inconclusive)
Options (b) (UA + UA = PA), (c) (UA + UA = PA) and (d) (UA + UA = PA) satisfy the rules for
testing the validity of the argument.
(b)
N PSN
P S
(c)
P
PSN
S
P
In
I.P In
C C
C
C
C
(c)
I.P
C
In In
In
In
C/I.
P
In
31. Ans. (d). Neither 1 nor 2 convinces us as to how making/ not making widow remarriage
obligatory will change the situation.
32. Ans. (d). 1 does not give a reason as to how marriages would shorten the life span. 2 gives a
reason justifying marriage whereas the statement should in fact be telling you what will happen
if a girl does not get married.
34. Ans. (d). Since neither of the statements give reasons for their arguments.
38. Ans. (a).1- judgement, since it is an opinion (must). 2 - judgement, since the sentence contains
an adverb(very). 3 is also a judgement since the statement involves the superlative degree
(most prolific). 4 is an inference as it is a cause-effect statement.
39. Ans. (a). 1 is a judgement since the statement has an element of compulsion (should). 2 is a
judgement for the same reason (must). 3 is a factual statement and 4 is an inference since it is
a cause-effect statement.
40. Ans. (d). 1 is a cause effect statement and therefore it is an inference. Indoctrination is given
as the cause. 2 is a categorical statement. 3 a cause-effect statement. 4 is a judgement
because the statement has an adverb in the form of 'only'.
So ―I work 12 hours a day‖ has two triggered events – ―I do not work hours a day‖ and ― I will
get a bonus‖
46. Ans. (c) as we do not have data in the passage to conclude or infer that the novae and
supernovae are always brighter than the sun.
47. Ans. (b). This is probably true. The last sentence says that it ―flares up in brilliance so that its
intrinsic luminosity for the first 30 days after the explosion is equal to 1000 million of our suns.‖
However, it is not clearly mentioned whether a nova is always less bright than a supernova.
But we can gather as much from the passage.
48. Ans. (b). This is probably true. The passage does quote CF Powell with regard to the novae
and supernovae. Therefore, it may be inferred that he is possibly an authority in this area.
49. Ans. (e). This is definitely false. the first line of the passage states that these stars increase
suddenly in brightness, and then gradually fade back to the normal levels. Therefore, it would
be definitely false to state that the supernovae self -destruct after getting very bright.
50. Ans. (a). This is definitely true. The passage states that the novae are more frequent than the
supernovae. In other words, the latter are rarer.
Exercise 1 (B)
1. Ans. (b). The argument assumes a direct correlation between being boring and being elected
the president of the ICPA. In other words, it assumes that all presidents of the ICPA would
inevitably be boring. Statement II gives the same idea in the form of a variation. Statement I
would be irrelevant as the argument does not make it clear that one has to be a member of
ICPA to be elected as the president. Statement III implies that all boring people are members
of the ICPA for which there is absolutely no justification in the given argument.
2. Ans. (e). The existence of hourglasses or sundials in various parts of the world is used as a
pointer to belief in the concept of time. According to option (e), All who built sundials or
hourglasses believed in the concept of time, and is therefore the best answer. Option (a) is
irrelevant, as the argument does not correlate reverence towards the concept of time with the
existence of hourglasses or sundials in all parts of the world. There is no evidence in the
passage to support option (b) and option (d). Option (c) would not support the argument either
as the argument is based on the existence of sundials and hourglasses not on the existence of
watch repairers.
3. Ans. (d). The given argument is in the form, If x, then y, z and p. The possible conclusions for the
given argument are
If not y, then not x,
If not z, then not x,
If not p, then not x,
If not y, not z & not p, then not x.
Option (d) is the only option which follows.
4. Ans. (b). There could be only one reason why the liberals oppose the appointment of John Galt
as the chairman of the Planning Commission – that he would take decisions that would serve
the interests of the far Right. Option (a) cannot be the answer, as the liberals would not have
opposed the appointment of Galt if they believed that he would make an unbiased and good
chairman of the Planning Commission. Options (c) and (d) are wrong as the author implies that
liberals oppose the appointment of Galt because they believe that he would work towards
ensuring that the Commission would be dominated by the far Right and not just because the
5. Ans. (c). The argument concludes that 24,000 to 30,000 people read 6000 copies of the
magazine Nerve which boils down to a ratio of 4 to 5 per copy. So the argument assumes that
the readership ratio of Nerve is the same as that of Belle. Options (a), (b) and (e) are
irrelevant. Option (d) is wrong as the argument assumes that the readership ratio per copy is
the same and not that the number of readers is similar.
6. Ans. (a). The argument says that the difference lies in the non-acceptance of perverted
violence by middle-class Americans, which goes to show that the depiction of sex on screen
was accepted by those Americans. Options (b) and (c) state just the opposite of what is stated
in the last sentence of the argument. Since the argument does not make it clear whether the
country in question is a democracy, option (d) can be ruled out. Option (e) can also be ruled
out as the argument just says that the American middle-class did not consider perverted
violence on screen acceptable but does not say that this group moulded the popular opinion.
7. Ans. (b). The argument obviously is based on the assumption that Pinka did not frame tests in
1996 and therefore concludes that Pinka has not framed those tests.
8. Ans. (b). Since the argument states that Leander Paes and Mahesh Bhupathi brought glory by
emerging victors, it is clear that it stands on the assumption that winning Wimbledon brings
glory to one's country. Option (c) is stated for a fact and is not an assumption.
9. Ans. (a). The author's conclusion that the protagonist need not be an epitome of virtue for the
literary assumption to be popular reflects his assumption that every literary composition would
have a protagonist. Option (b) is wrong as the argument states just the opposite - that popular
opinion is not based on portrayal of characters. Options (c) and (e) are not instrumental in
arriving at the conclusion. Option (d) is wrong as the argument says that Dr. Faustus became
very famous in spite of the protagonist being ignoble whereas the option states that such a
character always achieves more popularity.
10. Ans. (a). If fishing in estuaries is restricted only to commercial fishermen, it would undoubtedly
result in increased harvests. Limiting the total number of excursions per season for
recreational fishermen would not necessarily ensure that there would be an increase in the
harvests of commercial fishermen, as the recreational fishermen might end up netting most of
the shrimps. Options (c) and (d) would not be as effective as option (a), as the recreational
fishermen would still be able to net shrimps resulting in decrease in the harvests of commercial
fishermen. Allowing recreational fishermen to move out of estuaries into the South Atlantic
does not mean that they are not allowed to or that they are not going to fish in the estuary in
question.
11. Ans. (b). The argument rests on the assumption that UVA and UVB rays protection is
necessary for the skin. The value of the product would be severely undermined if UVA and
UVB rays protection is not required. So option (b) would be the best answer. Option (a) seems
to be a weakening argument but does not attack the major assumption underlying the given
argument; moreover, it talks about only one segment that is affected and does not question the
purpose of the product or its effectiveness in protecting one from harmful rays of the sun.
Options (c) and (d) also do not address the assumption.
12. Ans. (d). If most people do not have the infrastructure to use CD-ROMs they would not buy
them at all. So (d) is the best option. Options (a) and (b) can be ruled out as the company is
not likely to price their products high. Option (c) is a possible strengthening argument but the
argument does not make it clear whether the Encyclopaedia is a life lone source of knowledge.
Moreover this option does not mean that CD-ROMs are not popular or that no one would buy
them.
14. Ans. (a). Since the argument states the problems underlying baldness before the age of thirty,
it obviously assumes that baldness after thirty does not cause concern. So (a) is the answer.
The argument mentions that eating healthy foods can help one in counteracting some of the
risk of insulin-resistance related diseases, but does not hint or state that foods full of
carbohydrates are not healthy. So option (b) can be ruled out. One can counteract some of the
risk of contracting insulin-resistance related illnesses by keeping one's weight down, eating
healthy foods and avoiding cigarettes; all these factors contribute to counteracting the risk. As
option (c) mentions only one of these factors and as it speaks in absolute terms in saying that
you will not contract insulin related diseases, it can be eliminated. Option (d) is rather a fact
and not an assumption.
15. Ans. (c). The argument in the question follows the pattern wherein you take in something as
food and it makes you contract a disease. Option (c) is the only one which follows a similar
pattern. Option (a) mentions a stomach upset but not a disease. Option (b) is far from being
the answer. Since option (d) does not talk about what you take in as food, that too can be ruled
out.
16. Ans. (a). The main point of the argument is that when you are having a cosy dinner for two you
should go in for a Chardonnay and not a Cabernet. So the assumption is that looking like a
Dracula is not desirable at this moment. The argument does not imply that a cosy dinner for
two necessarily involves going on a date. Option (c) is possible but option (a) is more precise
as it addresses the Dracula part. The argument assumes that people roll wine, be it red or
white, over their front teeth and therefore recommends the use of Chardonnay instead of
Cabernet. Since option (d) mentions only red wine, it is not accurate.
17. Ans. (a). According to the argument, a woman who claims that a perfect marriage fell into her
lap is putting up a façade and is not mentioning what long-lasting love entails. Since there is
such a strong correlation between a perfect marriage and love, as stated in the passage, the
assumption is that a perfect marriage involves love. Option (b) can be eliminated as the
argument says that a perfect marriage is not like a well made up model. The argument is
specific; it is about a perfect marriage and not about marriages in general. Option (d) is rather
absurd. The argument says that an airbrushed photograph of a well made up model is not to
be compared with how you look the first thing in the morning; the implied message is that a
perfect marriage like the airbrushed photograph of a model requires a lot of background work.
18. Ans. (b). Since the argument about fitness conscious people and not about fitter people option
(a) can be ruled out. The calorie conscious and fitness conscious modern generation is much
slimmer, according to the argument. Therefore we can safely infer that if you are fitness
conscious, you will be slim. The argument does not equate calorie consciousness and fitness
consciousness with health consciousness, nor does it say that the fitness consciousness is a
part of it.
19. Ans. (e). It is not clear from the given argument whether the statements made are meant to
address everyone in the world. In other words, we cannot be sure that the statements are
generalisations. They could be addressed to a specific group. So none of the inferences would
follow. Option (b) might still seem right but we notice that the phrase ‗well endowed‘ could aslo
mean possessing a other things beside health and wealth.
20. Ans. (e). ―We ran the risk of boring people‖ is the statement given in the argument. That does
not mean it is boring these days. Option (a) can be ruled out. The argument uses a proper
noun; so we cannot be sure that Odissi is an art form. Moreover, option (b) is a sweeping
generalisation. We are not told about the result of the effort to make Odissi serve a social
cause. It might or might not be boring now.
Pankaj Gandhi’s Academy/Logical Reasoning 170
Exercise 2 (A)
1. Ans. (d)
P.
Go/GI/D
GI GI
Go D Go D
(4) (5)
Q. Refer to P for diagrams. ―All that glitters is gold or diamond‖ is true only in diag. (3)
3. Ans. (a).
P.
P P
P
B
F
B B F
F
Q.
F
B
B B
P
P P
F F
6. Ans. (b)
GP W W
H GP H
UA + UA = UA
7. Ans. (a)
FA H H
F FA F
UA + UA = UA
S C C
P S P
8. Ans. (b)
UA + UA = PA
9. Ans. (b)
NB NB PL P PL
P
UA + UN = UN
H S H
S C C
UA + UA = UA
Pankaj Gandhi’s Academy/Logical Reasoning 172
11. Ans. (c)
The statement defines the relationship between ―non-tribal‖ and ―uncivilized‖. Nothing can be
inferred about ―tribals‖ and ―civilized‖.
15. Ans. (b). If we infer that some cannot change the tide of time, we would imply that there are
others who can change the tide of time and that would go against the premise.
16. Ans. (a). When it is stated that forests are traditional sources of food and fuel, it is implied that
there are sources, which are non-traditional, or modern, which is what 1 says. The question
statement does not however, give any indication of food being essential for man.
17. Ans. (a). That there is an import bill implies that goods are being imported, which is what 1
says. The second statement, which says that mounting bills are dangerous, is not implied
anywhere in the statement.
18. Ans. (c). 1 states that waste is part of the scheme of nature and 2 states that renewal of life is
part of Nature's scheme. The question statement includes both under the scheme of Nature.
Therefore, both statements are implied.
19. Ans. (b). 1 is not implied by the question statement as the latter implies that energy crisis is the
only cause by terming it 'the cause'. 2 however, is implied, as, if there is a cause for tension, it
is obviously because there is tension in the first place.
20. Ans. (d). The question statement only says that those who neglect tomorrow would repeat the
mistakes of the past. This does not imply that we would forget the past; neither does it imply
that our past is full of mistakes. Therefore, neither of the statements is implied.
21. Ans. (a). 1 alone, which states that everyone should get food if a country should be said to be
completely free, can bring us to the conclusion that there can be no starvation in a completely
free country.
22. Ans. (d). The first statement merely states that parents have children. From this we cannot
conclude that the children love their parents. The second statement says that love is an
emotional bond. This statement too can not bring us to the given conclusion as it does not
imply that love exists between any given set of people.
23. Ans. (d). Statement 1, which says that better works are produced today, would not lead us to
the conclusion that there can be no comparison between the quality of work produced today
and yesterday. Neither does the 2nd statement help in arriving at this conclusion. The two
statements taken together too do not lead us to conclude that there can be no ground for
comparison.
24 Ans. (c). The first statement tells us that the lady was dissatisfied, and the second tells us that
such ladies are free to leave their husbands. Taking the two together, it is possible to arrive at
the given conclusion.
―Some cats are quadrupeds‖ does not follow as all cats are quadrupeds in both the diagrams.
(b)
A Q
Q C
A
A A C
A
A
A
Q
(d)
Q A
Q
A C
A C C
(c)
E E
D E.D/D
ED
(d)
D ED
D E
ED E D
D
D
D
(1) (2)
31. Ans. (a). The first answer provides a valid reason stating that it allows in freedom in choosing a
life companion, which in turn leads to a blissful marriage. The second answer is not forceful as
it does not say how love would tarnish the sanctity associated with institution of marriage.
32. Ans. (d). The first answer simply states that there is an increasing number of women who want
divorce. However, it does not say why that should lead to divorce laws becoming more
favourable for women. Wanting to do something is not reason enough to grant it. The second
answer passes a judgement and does not give evidence for supporting the statement that
women are emotional. It also does not support the correlation between being emotional and
seeking divorce on frivolous grounds.
33. Ans. (d). 1 is a judgement as it contains the adverb ‗almost‘. 2 is an inference, being a cause-
effect statement, 3 and 4 are factual statements.
34. Ans. (a). 1 is a factual statement. 2 has an element of compulsion in the phrase ‗are not to be‘
and is therefore a judgement. 3 and 4 are cause-effect statements and are inferences.
35. Ans. (a). 1 has a cause-effect pattern and would therefore be an inference. 2 is a factual
statement. 3 has an element of compulsion ‗will have to be‘ and is therefore a judgement. 4 is
an inference as it is a cause-effect statement.
P P P
M
E M E M E
―No mug is peg‖ and ― No peg is mug‖ follow only in diag. (2)
41. Ans. (a). This is evident from the first line of the passage. We are told that corporates are busy
innovating on severance pay packets. The given sentence would be a fitting opening sentence
to the passage as the first line tells us what they do to maintain a squeaky clean image even
after downsizing.
42. Ans. (b). The passage ends by talking about how they try to minimise the impact of income tax
on the recipient. So we can conclude that they would try to pay the money as royalty.
43. Ans. (d). The passage does not give any hint of a possible situation where the companies
would need their employees back. Hence it is irrelevant.
44. Ans. (d). The passage just tells us that the companies are innovating on severance pay
packets. We can infer that there is one competing ground for the companies. That they are
vying on every conceivable ground is not related to the ideas given in the passage and hence
cannot be an opening or a closing statement. Since the idea is not remotely parallel either it
cannot be considered a lateral argument.
45. Ans. (d). The passage does not imply that retrenchment is happening due to recession. So a
categorical statement like ―Retrenchment is a common feature of all companies in the wake of
recession‖ would be irrelevant.
For a disjunctive proposition like the above one, there are four possible conclusions as follows
If not y, then x
If not x, then y
If x, then y or not y
If y, then x or not x
Of these the last two do not apply here as none of the combinations possible follow that format.
The only negative statement among the four statements given is ―Cleo will not roll out of the
carpet‖ (If not x). This statement has to be followed by ―Cleo will be bitten by an asp‖ (then y).
So the answer is DC
49. Ans. (c). The argument in the passage is that the corporate sector is ready to battle the threat
of AIDS because they are now aware of the problems it causes. Of the given four options, (c)
attacks this argument the most by saying that companies have rarely gone beyond awareness
programs. This is the most weakening argument because it attacks the assumption, which is
that if you have awareness, the problem is as good as solved.
50. Ans. (d). A strengthening argument will give a reason as to why the sales of the investment
magazines have dipped. It would connect the fact that they concentrated on only the stock
market to a dip in sales. None of the given options give us this. Therefore none of them are
strengthening.
Exercise 2 (B)
1. Ans. (e). The primary assumption underlying this statement is that Whistler would not have
painted after 1900. Option (a) is a direct contradiction of the premise. The argument just
implies that Whistler would not have painted anything after 1900. That does not mean all
paintings done in or before 1900 were done by Whistler. So option (b) is wrong. Option (c) is
the same as option (b). There is no evidence in the argument to support option (d).
2. Ans. (c). The passage draws a causal relationship between brand recognition and increased
sales. This is obvious from especially the last line of the argument where the author identifies
the primary flaw as the inability of the viewers to recall the brand associated with the
advertisement, and says that this casts a doubt over the ability of the advertisement to promote
product sales. None of the options other than option (c) states this idea. Options (a) and (b)
are irrelevant as the argument does not concern itself with the credibility of a product or with
serious commercials. Moreover there is no indication in the argument, of the commercials in
question being flippant. Option (e) fails to identify the pivotal point of the argument while option
(d) is irrelevant.
3. Ans. (a). The given argument rests on the assumption that employing home nurses can cut
down the costs presuming that a small fraction of the costs for health care for the elderly goes
towards wages for these attendants. The criticism points to this loophole in the argument and if
the information given in option (a) were true, it would make the argument crumble to pieces as
costs for health care for the elderly would depend on how much the attendants have to be
paid. Options (b) and (d) do not address the cost part of the argument at all and are hence
irrelevant. The argument does not say that the elderly patients would themselves pay for their
health care. So option (c) is also irrelevant. Option (e) might seem to be right but if the wages
for health care workers forms a negligible fraction of the cost for health care for the elderly it
would not strengthen the criticism.
4. Ans. (a). The blacks, whites and hispanics seem to consider the Mayor‘s appointments
politically biased. Option (a) says that the appointments were based on merit and were not a
result of political prejudice. Option (b) does not make it clear that the appointments were not
tainted with political bias. Option (c) does not address the thrust of the argument. Option (d) is
irrelevant, as there is no indication in the argument of the appointments being illegal. Option
(e) is rather along the lines of a weakening argument.
6. Ans. (c). The assumption underlying the argument is that the birds would not be able to go on
their migratory route if the metal pieces are removed. That is the purpose of the experiment
undertaken by the scientists. Option (c) attacks this assumption by saying that the birds were
still able to go by the same migratory route. Option (a) does not address the assumption while
option (b) is a strengthening argument. Option (d) strengthens the scientists‘ point of view to
certain extent while option (e) also endorses the scientists‘ view by talking about a change that
has been effected by the experiment. Option (c) on the other hand says there is no change in
the pattern of the birds‘ flight.
7. Ans. (e). The argument is clearly based on the assumption that it would be possible to bring
post-graduates to the teaching career by increasing monetary benefits. That is the rationale
behind the HAPTA‘s single point decision to hike salaries and offer income tax relief packages.
None of the other options even remotely address the central concern of the argument.
8. Ans. (e). The assumption is evident from the premise which says war is a luxury only small
nations can afford. Option (a) is too general a statement; the premise talks about only the
luxury relating to war and not all luxuries. Moreover, nothing is said about the large nations in
the premise. Option (b) might seem close but the argument says that war is a luxury only small
nations can afford; it does not imply that small nations actually go to war nor does it imply that
all nations that go to war are small nations. Options (c) and (d) are not supported by the given
premise.
9. Ans. (d). ―If I do go home‖, we can conclude from the second statement that Pink does not
attend the class. This is not given in any of the options.
10. Ans (c). It can be inferred from the argument that 53% taxable income came from foreign
sources for corporations showing a net income more than $100 million. Also, 60% taxable
income came from foreign sources comprising of more than 10 countries. Hence we can infer
that the total taxable income, about 53% - 60% came from foreign sources.
11. Ans. (c). Options (a) and (b) bear no relevance to the passage. Option (c) is the kernel point of
the argument.
12. Ans. (b). The argument projects a super-speciality hospital as a high-profile hospital which is
way above kitchen clinics and private hospitals. So statements II and IV are assumptions
underlying the argument. Statement I is not an assumption, as the argument implies that a
super-speciality hospital or a central team of doctors is beyond the reach of a lay man and not
the other way around. Statement III is quite the opposite of what the argument says.
13. Ans. (c). The argument is about how the genie initially considers building a highway spanning
the Atlantic Ocean from Marseilles to New York difficult but later thinks it is far easier than
making out the deepest secrets of a woman‘s heart. Option (c) is therefore the right one. There
is no indication in the passage of the genie not being grateful. The passage just talks about
how he really compares the difficulty of one with the other. In fact he does grant a wish and
does not say he is not going to grant anything. There is no correlation given in the passage
between not trying to fathom a woman‘s heart and being a male chauvinist pig.
14. Ans. (b). The argument hints that electronic equipment is likely to be a part of the latest
‗clothing‘ by saying that electronic clothing is in. Only option (b) supports this idea. The need
for ironing is nowhere indicated as a cause for concern. Similarly options (c) and (d) neither
strengthen nor weaken the argument.
16. Ans. (b). The British consumer research magazine is cited as the authority on information
about radiation caused by cell phones. So the assumption is obvious; the British consumer
research magazine is considered a source that can be banked on. The ideas in options (a) and
(c) are rather stated in the argument while an assumption, by definition, is implicit. Option (d) is
irrelevant.
17. Ans. (b). While the argument talks about how modern x ray machines can damage film we
notice that there is a mention of holiday memories. It is not necessary that everyone at the
airport would be carrying photographs taken in the holiday. The argument thus assumes that
the person that the argument is addressed to is returning from a holiday. Option (a) states the
opposite of what is given in the argument. The idea given in option (c) is stated in the
argument and is not implicit. Hence it cannot be an assumption. The passage does not make it
clear whether modern x-ray machines are important parts of baggage checks at the airports.
All that we can gather from the passage is that it is used to peer into checked-in-baggage.
18. Ans. (b). The argument puts forth the frequenting of jail as sole advantage and qualification
that the speaker has. So the idea given in option (b) is the assumption underlying the
argument. Option (a) does not state the assumption precisely. Going to jail is not given as an
advantage. What is rather stated is that the speaker has gone more times. From the given
argument we cannot be very sure if Sohrabji has been to jail several times. For all we know
Sohrabji might have gone to jail twice and the speaker would have gone three times. Option (d)
is off the track.
19. Ans. (d). The argument talks about spiritual belief alone and not faith in oneself. So option (a)
can be eliminated. The argument mentions only one situation where faith is beneficial. So we
cannot infer that faith is always beneficial. The argument talks about believers and mentions
nothing about non- believers. It says believers are less likely to get depressed. That does not
imply that people who are ill and depressed are non-believers. So option (d) can be eliminated.
Option (c) sums up the gist of the argument.
20. Ans. (b). From the argument given, we cannot be sure that the money would have been spent
on primary education had it not been spent on a nuclear weapon. So option (a) can be
eliminated. There is no clear evidence in the passage to support option (c). Option (b) is right
as per the evidence given by the last line in the argument and especially by the phrase ―candid
juxtaposition‖.
Exercise 3 (A)
1. Ans. (d). The first statement cannot be inferred because we cannot be sure of what the "it" in
the question statement refers to in the first place. "It " most probably refers to the need for a
new work culture and not to "time saving devices". Therefore we cannot conclude that Indians
are slow and slothful. For the same reason, we cannot conclude the second statement either.
2. Ans. (d). The first statement cannot be concluded, as just because the Commission is
opposed to the proposal it does not mean that they are the ones who plan the expenditure.
There is no ground for concluding that helicopter operations must be subsidised. There has
been a proposal and that does not mean that there is a compulsion. So neither of the
statements can be a logical conclusion.
B P P
S B S
UA + UA = UA
4. Ans. (a)
N DM DM
C N C
UA + UA = UA
5. Ans. (a)
―No jar is a glass‖ is the only true conclusion for ―No glass is a jar‖.
6. Ans. (c)
―Some good things are not summer tonics‖ is true in one of the two possibilities.
7. Ans. (c)
The statement defines the relationship between non-toxic things and harmful things. Nothing
can be inferred about the set of beneficial things.
8. Ans. (c)
―All things harmful to eyes is strong light‖ is true in one of the two possibilities.
9. Ans. (a)
―All that glitters is not gold‖ is a rewording of ―Some things which glitter are not gold‖.
10. Ans. (b). The first statement cannot be inferred as the question just states that woman is the
embodiment of sacrifice. This just means that women make sacrifices. The statement doesn't
give us any idea as to the level of sacrifices made by men. So, just the second statement is
implied.
11. Ans. (b). The second statement alone can be inferred from the question. All that the statement
says is that the issue should not be considered too deeply. Obviously, it was being considered
so. It does not imply, like statement 1 says that something else was being considered deeply.
12. Ans. (a). The first statement alone can be inferred. If something can be conquered, then it is
obvious that it is weak. And religion has to do with man. Therefore, if religion is a conquest of
fear, then fear is obviously a weakness on the part of man. The second statement though,
cannot be inferred as we do not know from the given statement whether religion is something
private or public.
13. Ans. (c). Both statements are implied. If recreation is termed the highest kind of enjoyment,
then it is obvious from the superlative degree that there are other forms of enjoyment, and also
that recreation itself is an enjoyment.
Pankaj Gandhi’s Academy/Logical Reasoning 180
14. Ans. (c). If there is a call for production to be augmented, then it is implied that production is
insufficient. It is also implied that it is possible to augment production or else there would not
be a method to do the same. Therefore, both the statements are implied.
15. Ans. (d). The statements 1 and 2, taken in isolation or together, cannot bring us to the given
conclusion that great men have great faults. 1 just says that man is fallible, or that, he has
faults. This alone can not lead us to he given conclusion. Similarly, the second statement
merely says that actions make a man great. It does not tell us anything about great men.
16. Ans. (c). You are told that when you have feasts they are meant to be enjoyed and also that
there is no enjoyment in life. Taking these two statements together, we can conclude that life is
not a feast.
17. Ans. (a). The first statement alone, which says that concentration helps a man to memorise, is
sufficient to take us to the given conclusion that concentration is the key to a good memory.
18. Ans. (b). The second statement, which states that little minds cannot withstand misfortunes
alone is sufficient to take us to the conclusion that little minds will be subdued by misfortunes.
The first statement is not relevant to the given conclusion.
(b)
T
T S
B S
B
B
B
T T
B
S B S
(c)
T
T BST
S S B
B
T
BST
S
B
(a)
F
I PIF
P
(c)
I F PI F
P
23. Ans. (b). The first argument is not forceful because it does not address the issue at hand and it
does not give a supporting reason either. The second argument, on the other hand, is forceful
as it addresses the issue and gives a valid reason that President would be as good as a non-
entity without any active role.
24. Ans. (d). Neither of the arguments addresses the issue of whether politics can be called the
refuge of scoundrels or not. For instance, the first argument does not say whether the
scoundrels actually use politics as a refuge for the reason given. Similarly the second
argument does not say whether the scoundrels do not enter politics because people would not
accept them nor does it say they do not consider it a refuge.
25. Ans. (d). The first statement does not tell us how having a mixed economy would lead to a
competition between the private and public sector. Moreover, it does not tell why it is the best
type of economy for India. The second argument does not tell us why the mixed economy
cannot be the best economy nor does it talk about India specifically. This is what a forceful
argument would address.
27. Ans. (d). 1 is a judgement as it is an opinion (should). 2 is again an opinion (may) and
therefore a judgement, 3 too, is a judgement as it is in the superlative (best batsman). The
fourth statement is also in the superlative but it talks about the batting average, which is
quantifiable. So it is a fact.
28. Ans. (a). The first statement is a categorical statement. The second statement is a judgement
because of the word ‗must‘. The third statement is a categorical statement. The fourth
statement is a cause and effect statement.
29. Ans. (d). 1 is a judgement because of the word ‗should‘. The second statement has a cause
effect pattern. The third statement is a prediction and hence a judgement . The last statement
is a proverb and also has the superlative degree. So it is a judgement.
30. Ans. (c). 1, 2 and 3 are all categorical statements, and 4 is a cause-effect statement.
Therefore, FFFI.
31 Ans. (d)
R Q R Q
P P
(1) (2)
R
P Q PR Q
(3) (4)
R
R B
R
(4) (5)
―Some rags are bags‖ and ―Some bags are rags‖ are uncertain.
S
C C
P
P
S
―No fish is sparrow‖ and ―No sparrow is fish‖ follow in both the diagrams.
41. Ans. (a). The inference is definitely true since the passage tells us that in South Korea, one out
of every five people is named Kim. This amounts to 20%.
42. Ans. (b). The inference is probably true since we know that there were a lot of couples who
were living together but were not married simply because the law prevented them from doing
so. Therefore, it would be a fair possibility that they would marry once the law was struck
down.
43. Ans. (e). The inference is definitely false as the passage clearly states that 8% of the
population is named Park. This is eight out of hundred and not one out of eight.
44. Ans. (b). The inference is probably true as the passage does indicate that the law was written
as in-breeding was cause for concern. It is not, however, explicitly told in the passage that this
is the basic reason and so we cannot call it a definitely true inference.
45. Ans. (d). The inference is probably false, as, when there was a law preventing people who had
the same surnames from marrying, it was improbable that they would marry disregarding the
law. However the passage does not say explicitly that they did not marry. So the inference is
probably true.
46. Ans. (d). The first statement cannot be inferred from the premise as the latter only tells us that
rich countries average more than one main telephone line and not that rich countries alone
have telephones. And we cannot infer 2 as we have no information in the premise about the
less affluent countries.
47. Ans. (d). The first statement cannot be inferred, as the question statement does not give us
any idea of how tourists behave worldwide. The second statement cannot be inferred, as just
because Florence has drawn up a handbook, it does not mean that she wants her tourists to
necessarily conform to it. Perhaps she had come up with the handbook just to give tourists an
idea of good manners.
H So Si
Fr Sa H/F/So/Sa/S
i
(1) (2)
―All Harrys are Sintos‖ follows in both the diagrams. ―All Sintos are Harrys‖, however is true
only in diagram (2)
Exercise – 3 (B)
1. Ans. (c). Raoul‘s statement is the same as ―All books written by Camus are masterpieces‖.
Vadim‘s reply shows that he has understood the statement to mean ―All masterpieces were
written by Camus‖, which is why he says there are several masterpieces written by other
people. Option (c) is the same as ―All books found in the libraries are masterpieces written by
Camus‖.
2. Ans. (c). The argument concludes that students of St. Stephen‘s College get a better standard
of education than students of Christ College solely on the basis of the average marks. It is
evident that the argument assumes marks reflect the standard of education. The common
platform that is assumed in the comparison also goes to show that the way the performance in
exams is rated is the same in both the colleges. Hence statements I and III are the
assumptions. Statement II is irrelevant.
3. Ans. (c). The argument compartmentalises expression of lasting moral values and commenting
on political issues. The author regards an overlap aberrant. So the answer is clearly (c).
Option (a) does not cover the thrust of the argument entirely. Since the author does not pass
judgements on how much the expression of moral values is respected, option (b) can be
eliminated. Options (d) and (e) like option (a) do not cover the thrust of the argument.
4. Ans. (a). The opponent‘s point is made by citing movies where special effects have been used
in abundance. The author refutes the point propounded by his opponents by giving examples
of movies where there has been no use of special effects and the movies were still equally
successful. Only option (a) identifies the pattern in the argument.
6. Ans. (c). The argument here says that it is extremely important for women's colleges to become
co-educational if they have to survive. The reason cited is that very few college-bound women
would consider going to all-women's colleges. In other words, a large portion of the college
bound women will go to coeducational colleges. (c) is a weakening argument because it tells
you that the total capacity of women's colleges can take only 2 % of the college-bound women
whereas 15% of the college bound women have said that they would want to go to all-women's
college. So it means that women's colleges already have a surplus of potential students. So
they do not have to turn co-educational to survive. This is the option that most directly attacks
the argument.
7. Ans. (c). The argument believes that people have no solid reasons for distrusting a lawyer. IF
most people have very little direct contact with lawyers, they obviously will not be able to
inform the lawyers all the intimate details of their lives through a third person. No other option
comes even remotely close to questioning the assumption. For example, option (a) does not
explain why people do not trust a lawyer in the first place.
8. Ans. (a). The author ends the passage by saying that if a person cannot be an honest lawyer
he should resolve to be honest without being a lawyer. So the answer is clearly (a). Options
(b), (c) and (e) state the opposite of what the argument says. Option (d) is outside the scope of
the argument as the author talks about only honesty with respect to practising law.
9. Ans. (c). If the information given in option (c) were true, it would mean BOIL is under the
impression that their customers have stopped visiting pubs while they have actually passed
away or quit drinking. Even though these pub-goers have been replaced by young drinkers the
owners of the bars would still talk about a decline keeping the older pub-goers in mind. And if
there is an addition to the number of pub-goers due to the young drinkers it would explain the
results of the SPCA‘s survey as well. Since the respondents have reported that they visited
pubs and bars, options (a) and (d) would be irrelevant. Option (e) can also be eliminated, as
the argument is about the contradiction within the survey in question. Option (b) might seem
close but we are told that the compilations are often inaccurate. If they were not inaccurate this
time, this option would not help in explaining the contradiction.
10. Ans. (b). The speaker here believes that Pakistan is capable of winning the World Cup only if
the tournament is fixed. Since the argument does not imply that Pakistan is the only country
capable of winning fixed tournaments, option (a) can be eliminated. As the argument does not
talk about other countries option (c) can also be eliminated. The argument does not pass
judgements and hence option (d) can be eliminated. Option (e) would have been right had it
been ―Pakistan will not win the World Cup unless it is fixed.‖
11. Ans. (c). The argument tries to prove the point that third degree is the best method to use here
and polite interrogation would do no good, as there are no Cronjes to confess. So the
assumption is that third degree questioning would lead people to confess. There is no
evidence in the argument to infer that the cricketers in question are Indian. Option (b) is a
marginal assumption and does not address the central concern of the argument, which is that
they should be subject to third degree questioning; the argument does not just stop with saying
that these cricketers do not confess but forcefully tries to prove the point that they should be
put third degree questioning. There is no evidence in the passage to conclude that Cronje is a
hypocrite. Moreover, this is not the central assumption underlying the argument.
13. Ans. (c). Since the argument says that robots are still short of creativity and emotions, option
(a) can be eliminated. Nothing is mentioned in the argument about independent thinking.
There is no evidence in the argument to support option (d) either. As the argument says that
robots have the ability to exhibit only programmed behaviour and that even the most advanced
robots cannot be creative, we can infer option (c).
14. Ans. (b). According to the argument, a resource is created only when man finds a use for it.
Option (b) gives a case of some materials, which were not resources until man found a use for
it. No other option endorses the central idea of the argument.
15. Ans. (b). The argument assumes that nothing can be done to eradicate scarcity and says that
human knowledge and materials will always be limited. Option (b) addresses both these
concerns by saying that an extensive chart of the material resources of the world has been
prepared, which goes to show that the material resources are not limited and that human
knowledge has been used to make such a chart. Option (a) does not disprove the fact that
material resources are limited. Option (c) also talks about making optimal use of the resources
available but does not say that material resources are not limited. Option (d) talks about some
minerals that were not accessible earlier but are accessible now; nevertheless it does not talk
about the other material resources, nor does it say that the minerals that are being unearthed
are unlimited.
16. Ans. (b). The argument suggests different ways of stimulating memory and learning
continuously is projected as the best method to stimulate memory. Option (b) puts the same
idea in other words and thus endorses the argument. Option (a) would rather be a weakening
argument. Option (c) says nothing about how learning improves memory and is thus irrelevant.
17. Ans. (b). The kernel point of the argument here is that being a man is all about acting and they
are not rewarded for anything else. Option (b) says that action has indeed been inculcated as
a man‘s preoccupation. Option (a) can be eliminated, as it does not talk about doing things,
which is mainly talked about as a man‘s domain. Option (c) talks women and is thus irrelevant
to the given argument.
18. Ans. (b). The argument assumes that India has never bothered to spend enough on electricity
and education. Option (b) attacks this assumption by talking about the increasing importance
given to electricity and education. No other option talks about the emphasis placed on
education and electricity contrary to the perception given in the argument.
19. Ans. (b). The argument talks about the lack of awareness of the dangers caused by bacterial
contamination and the low hygiene standards. Option (b) gives evidence of the prevalent low
hygiene standards and the lack of awareness of bacterial contamination. Options (a) and (c)
are irrelevant while option (d) is rather along the lines of a weakening argument.
20. Ans. (c). Options (a), (c) and (d) are strengthening arguments while option (c) attacks the
primary assumption in the argument that venturing into different fields is beneficial.
Exercise 4 (A)
1. Ans. (a)
―Only able-bodied men are recruited‖ is a rewording of ―All who are recruited are able-bodied‖.
2. Ans. (a)
―Some Ys are Xs‖ is the only true conclusion for ―All Xs are Ys‖
4. Ans. (c)
―Only the educated can tackle this problem‖ is a rewording of ― All who can tackle this
problem are educated‖. ―Nothing can be inferred about ―uneducated people‖.
5. Ans. (c)
Nothing can be inferred about rash people from the given statement.
6. Ans. (c). Both the statements, if taken together, can bring us to the given conclusion. We know
that there was tension in the world and that the same was a cause for concern. Therefore it
can be concluded that the world should feel happy at the lowering of tension.
7. Ans. (a). The first statement tells us that if there is proper irrigation, then the yield is very high.
This alone will suffice to take us to the conclusion that irrigation is vital for increased
agricultural production.
8. Ans. (c). We are told that slogans are misleading, and that politicians indulge in them. These
two taken together will enable us to conclude that people are misled by political slogans.
9. Ans. (a). The first statement tells us that the price of the food is in direct proportion to the
inputs. This in itself is enough to conclude that the cheaper the inputs, the lower will be the
price of the food.
10. Ans. (d). Neither of the statements, taken in isolation or together, will lead us to the conclusion
that education has become poor, or, that it has deteriorated.
11. Ans. (c). We are told that human tastes are varied, but that they are cyclic in nature. In other
words, they can be predicted. If we take both the given statements, then we can arrive at the
conclusion.
12. Ans. (a)
Options (a) and (b) satisfy the rules for testing the validity of the argument. However the
propositions in (b) contradict each other. Options (c) and (d) are eliminated, as the arguments
in these options are inconclusive.
(a)
G G C
C F
F
(1) (2)
(3) (4)
―Some people who are charming are gluttons‖ follows in all the possibilities.
C
F GFC
G
(c)
―All gluttons are charming‖ follows in both the possibilities. So ―Some gluttons are charming‖
does not follow.
15. Ans. (d). Neither of the answers given relates to the reservation of jobs.
16. Ans. (b). The first answer does not talk about India specifically and is hence not forceful. The
second answer gives us a blanket statement; ―all nationalised sectors‖ would include
nationalisation of Indian agricultural sector as well. If they become inefficient due to
nationalisation it is reason enough to deter nationalisation.
17. Ans. (b). The first argument is not forceful, as it does not tell us how the prevalent illiteracy
would make adult education a success. The second argument though, is forceful as it
addresses the issue gives a supporting reason by talking about the lack of time to be spent on
education thus indicating that it is improbable for the movement to be a success.
18. Ans. (d). The first argument is not forceful, as it is ambiguous; the use of the pronoun ‗it‘ is not
very clear. The second statement does not give a supporting reason. It does not give
information about whether regional economic development is extremely important and thus a
reason enough to prevent dispensing with the public sector.
I
M PMI
P
―All intruders are passengers‖ is uncertain while ―Some passengers are intruders‖ is false in
both the possibilities.
Pankaj Gandhi’s Academy/Logical Reasoning 190
P
S ASP
A
―All actors are planets‖ follows in both the diagrams. ―All planets are actors‖ follows only in
diag. (2)
Y Y
S S
S P Y
P
P
―No prudent is young‖ and ―No young are prudent‖ are uncertain.
G B
CGB
G C
G
―All cows are goats‖ and ―All goats are cows‖ are uncertain.
―Coca cola is prohibited‖ follows in both the diagrams. ―Coca cola is not prohibited is false in
both the diagrams.
L
R CRL
C
―All canals irrigate the land of this state‖ is true in both the possibilities. ―Some canals irrigate
the land of this state‖ is false in both the diagrams.
25. Ans. (d). Since the doctor has advised him to take medicine and not anything that is bitter, the
first conclusion does not follow. Since the premises do not talk about sweet things, the second
conclusion would not follow either.
―All boys are generally liked by the children‖ and ―Some children generally like boys‖ are true
in both the diagrams.
E
H CHE
C
―All cocks lay eggs‖ is true in both the diagrams. ―Some cocks lay eggs‖ goes against the
argument.
―All lords are members of the upper house‖ and ―Some members of the upper house are lords‖
are true in both the possibilities.
30. Ans. (d). 1 is not implied as all that we are told is that the government wishes to promote the
people‘s wishes. This by no means implies that the interests of the two parties are different.
Neither does this mean that only democratic governments would wish to promote the interests
of the people. Thus, 2 is not implied.
31. Ans. (a). The first statement is implied as, if it is told that we must keep up the pursuit of truth,
then we are obviously pursuing truth. The second statement though, is not evident from the
statement.
32. Ans. (c). When you are told that human beings destroy humanity, then it is because humans
are destructive. Humanity is mankind itself, and so, in destroying humanity, man is man‘s
enemy.
33. Ans. (d). Just because the villagers feel that the city dwellers are in a hurry it does not lead us
to conclude that there is not much social life in the city. Neither can we conclude from the
statement that villagers do not like city life. All that we are told that is that villagers wonder
what the hurry is all about. That is not synonymous with dislike.
34. Ans. (d). We cannot arrive at either of the conclusions. The statement does not tell us that
social justice is essential or indispensable in a democratic state. The use of the pronoun ‗it‘ is
ambiguous here. It could refer to welfare state or to social justice.
35. Ans. (a). We can arrive at the first conclusion as we are told that the poverty line is computed
on the basis of minimum calorie requirement and that 48% of the population is below the
poverty line. Therefore, we can conclude that 48% do not get the required calories. However,
we cannot conclude that because of this condition, poverty is India's biggest problem. There is
no comparison given between poverty and other problems.
WP A
A
H WP H
UA + UA = UA
UA + UA = UA
DC U
U
B DC B
UA + UA = UA
IM C
C
S IM S
UA + UA = UA
―Only criminals adopt illegal methods‖ is a rewording of ―All who adopt illegal methods are
criminals‖.
I I
G
G C C
UA + UA = UA
41. Ans. (d). The argument is irrelevant because the passage talks about a specific product of
Compaq and not about the success of its price-war strategy. From the passage, we cannot
conclude that the strategy has paid off.
42. Ans. (c). This is a lateral argument because the passage does mention that the pricing is a
crucial factor ("..crucially, at a price-point lower than the existing models")
43. Ans. (a). The passage talks about the launch of Compaq's product. Therefore, we can place
this argument as an introduction to the passage.
44. Ans. (b). It is a downstream argument, or a conclusion to the passage, as the passage does
tell us that the product has been launched with price being a major consideration.
45. Ans. (d). This argument would be irrelevant, as we cannot infer how convincing the price tag of
the product is.
46. Ans. (a). This statement would be a logical conclusion since we are told that the economic
policies of the BJP are directed at strengthening the coalition than at being clear and
consistent.
47. Ans. (b). We are told in the passage that the economic policy of the BJP is far from being clear
and certain. Therefore, calling it well-defined would be contradictory to the intent of the
passage.
49. Ans. (d). The given statement is irrelevant since the passage has nothing to do with corporate
performance.
50. Ans. (d). This is irrelevant since the concept of economic nationalism is outside the context of
the passage.
Exercise 4 (B)
1. Ans. (a). The Wordmaster Typewriter Company assumes that since there is boom in the sales
of electronic typewriters in the market, there would be no hindrances or other adverse factors
that the sales of the increased number of electronic typewriters manufactured by them. Option
(a) attacks this assumption by citing the case of preference shown for a competitor‘s products
suggesting that the expected sales is likely to take a beating. Since the argument does not
indicate that investments in equipment and in a new factory would be unaffordable, option (b)
would not be weakening argument. Option (c) simply repeats the information given in the
passage and thus does not weaken the argument. Option (d) contradicts the basic information
given in the passage, which is the preference shown for electronic typewriters.
2. Ans. (c). The argument hints that it is not enough if the problem of pollution is handled on a
national basis. So it would be logical if the next statement talks about the problem being
handled on an international basis, as in option (c). Option (a) can be eliminated as the thrust of
the argument is about pollutants in general and not about nuclear power. The Chernobyl
nuclear accident is given as just case in point. Option (b) can also be ruled out, as the
argument talks about the problem on an international basis and just with respect to one nation
and its neighbour. The argument does not talk about using force but about having a say in a
neighbouring country‘s environmental policies. Option (d) does not talk about dealing with the
problem at hand and talks about only the future. Option (e) is incorrect because there is no
evidence in the given argument to suggest a need for an authoritative ruler. There is a call for
dealing with the issue on an international basis but not for the use of authoritarianism.
3. Ans. (a). The given argument talks about how pollution does not respect national boundaries.
Option (a) gives evidence of a problem which affects another country as well and is hence a
supporting argument. Options (b), (c) and (e) do not talk about the impact that the pollution has
on other countries. Option (d) talks about a solution to curb pollution but does not support the
assumption in the argument which is that the problem of pollution is not restricted to just one
nation.
4. Ans. (d). The argument says that it is not absurd to breed an intelligent human race purely on
the basis of evidence cited in the sags of Tiahuanaco, the inscription on the pediment of the
Gate of the Sun and the old religious scripts. It is clear that the argument assumes credibility of
these texts. The statement in the argument – ―With the theory of a visit to our earth by
unknown intelligences from the cosmos we could postulate that today we are similarly
constituted to those fabulous unidentified beings‖ – makes it clear that the foundation of the
argument is the theory found in those texts; so option (b) can be eliminated. Assumptions are
implicit. Option (c) states a possibility that is talked about in the argument on the basis of the
information given in the texts. The argument does not say human beings are in the process of
being formed by another people. It rather says that they were formed by another people and
there is hence a similarity.
5. Ans. (a). The given statement obviously implies that when it comes to love and war anything is
warranted. Option (a) conveys the same idea. From the given statement we cannot infer
anything about other times. So option (b) can be eliminated. Option (c) can be eliminated, as
the premise does not talk about people. Since the premise does not talk about the importance
of these events, option (d) can be eliminated. Option (e) is totally off the track.
7. Ans. (b). The given argument talks about the quintessence of a particular genre and the efforts
put in by the pioneering people. Similarly option (b) talks about the quintessence of
contemporary warfare and the pioneering efforts put in by people like Napolean, etc. No other
option mentions the quintessence of something and the efforts put in by the pioneers.
8. Ans. (a). The given argument assumes that the current passing percentage, which is 30%, can
be improved by trying out new methods. Option (a) attacks this assumption by stating that a
new method does not result in high passing percentage. Option (b) merely passes a judgement
and does not attack the assumption. Moreover, the argument does not say that the new
method was frivolous. All that is said that education was made more interesting and interactive.
Similarly options (c) and (d) pass judgements and do not give evidence to weaken the
assumption in the argument. Option (e) also states that it would be hazardous but does not
mention what exactly the hazard would be and how this particular approach would be
hazardous. It is a generalisation, which is not justified.
9. Ans. (c). Any survey assumes that the sample size is representative of the target population.
Similarly this argument concludes about all teenagers based on the survey conducted on a few
teenage college girls. Option (a) is irrelevant, as the survey is about the hairstyle that teenage
girls prefer. There is nothing to pertinent to who is more conscious of looks, etc. Options (b),
(d) and (e) also do not address the main concern of the argument.
10. Ans. (c). Ms. Dennis concludes that the engineering students are lazy on the basis of their not
doing the work that they have been assigned. So option (c) is right. Option (a) is irrelevant, as
the argument does not talk about other students. Option (d) is also irrelevant for the same
reason. Option (b) is a judgement and not the assumption underlying the argument. Option (e)
is also a judgement.
11. Ans. (d). The argument says that more emphasis is placed on saving the environment than
poverty and unemployment. Option (d) cites a case of people who are willing to pay more for
the sake of saving the environment. Option (a) weakens the argument by stating the opposite
of what is stated in the argument. Option (b) also talks about agricultural economy and not
about saving the environment. Option (c) seems right but there is no information given about
the reliability of the survey or about how many people were surveyed and whether they are
representative.
12. Ans. (e). Option (a) cannot be concluded that because the argument says that the
aim/milestone can be achieved when people are focussed and harmonise their energies and
not that they are easily achievable anytime. Option (b) is irrelevant, as the argument does not
talk about unity in general but about harmonising energies. Since the argument talks about
harmonising one‘s energies and not about working together, option (c) can also be ruled out.
Option (d) can be eliminated for the same reason.
13. Ans. (c). The given argument mentions a particular group of people who represent the
interests of the rich and that they have not been elected but selected. Similarly option (a) talks
about the cricket team which has been selected by a team consisting entirely of members who
represented the state of Maharashtra. So it is obvious that the selected team would represent
only their interests. No other argument talks about selecting or representing as the one in the
given argument.
15. Ans. (c). The question asks for the most basic assumption. When we observe the argument
carefully we notice that the argument assumes a causal relationship between controlled
behaviour and a perfect world. That is the crux of the argument and hence option (c) would be
the most basic assumption. Options (b) and (d) state marginal assumptions. Option (a) is
irrelevant because the phrase ―no people‖ is not used in the literal sense in the argument.
16. Ans. (a). From the above explanation it is clear that option (a) can safely be inferred because
of the relationship that is assumed between controlled behaviour and a perfect world. The
central idea of the argument is that the world would be perfect only if people‘s behaviour were
controlled. So the given option would be a logical inference. The given argument merely talks
about controlling behaviour and does not say whether behaviour is good or bad. So option (b)
can be eliminated. Option (c) can be eliminated for the same reason.
17. Ans.(c). The argument talks about lack of critical standards and discrimination amongst the
Indian critics implying that they have their bias for the West and for extreme parochialism.
Option (c) talks about Indian writers developing a critical frame of mind and that would mean
that they rid themselves of their biases. Option (a) is incorrect as the author says that those
who speak English go by the West and not that the press in general goes by the West. Option
(b) is incorrect for the same reason. Option (d) is irrelevant.
18. Ans. (c). The argument mentions confirming the news and double-checking. So it would be
logical to infer that the authenticity of the story is verified and that the reporter is questioned
regarding the accuracy of the content that he has collected. Option (a) can be eliminated as
the argument talks about double-checking the content that the reporter collects and about the
references that he has to come with to be hired. We do not have clear information about who
verifies the news. So option (b) can also be ruled out. Option (d) is not a complete inference,
as it does not talk about checking with the reporter as to the accuracy and authenticity of the
story. Option (c) is better than option (d).
19. Ans. (e). The kernel point of Jimmy Dogwallah‘s argument is that the prices of legal imports
would not be high in spite of the duties. None of the options question this idea. Option (d) is
apparently right but a closer scrutiny would make it clear that it merely states that the prices of
legal imports are still higher than similar products available in India. It does not compare the
prices of legal imports with the smuggled goods available.
20. Ans. (d). The argument says that when transparency is preferred it is weird on the part of
media to complain against the probe by the Commission of Inquiry. The fact that it is perceived
as odd goes to show that the argument assumes that the purpose of existence of the media is
defeated if they object to a probe. So option (d) is right. Option (a) states almost the opposite
of what is stated in the argument. Option (b) is irrelevant. Option (c) gets the idea wrong. The
argument is about the probe by the Commission of Inquiry and not probing the Commission of
Inquiry.
1. Ans. (d). From the statement, we do not get the idea that vigilance is essential. And, when it is
told that eternal vigilance is the price of liberty, what is implied is that we will have to be
eternally vigilant if we need liberty and nothing more. Thus neither of the statements is implied.
2. Ans. (b). The first statement is not implied, as we have no idea about the nature of our wants.
All that we are told is that it should be fewer than the present number. However, we are told
that the fewer our wants, the more we resemble the gods. This would mean that the gods have
no needs at all. Thus the second statement is implied.
3. Ans. (d). The statement only tells us that wit is the salt of conversation. This does not imply
that wit is necessary for a conversation nor does it imply that everyone likes witty
conversations.
4. Ans. (a). The first statement is implied as when we are told that variety is the spice of life, it is
implied that variety would make life spicy. But this would not imply that variety is important or
indispensable for life. Therefore the second statement is not implied.
5. Ans. (d). All that is implied is that none other than sincere people can recognise sincerity. So
the idea of compulsion implied in the first conclusion would not follow. Conclusion 2 is way off
the track.
6. Ans. (d). The first statement is not implied as we cannot say whether it is work which wears out
a man. The second statement too is not implied. We know that it is better to wear out, but it is
not said that if a man does not work he loses his charm.
7. Ans. (d). Statement 1 alone cannot lead us to the given conclusion as all that the former says
is that the speaker studied at school. The second statement too would not lead us to conclude
that the speaker stumbled upon the lesson at school. Even if we take the two statements
together, we cannot conclude that he stumbled upon this particular lesson. He could have
stumbled upon anything.
8. Ans. (d). Taking the first statement by itself, we cannot conclude that the government should reject
the demand for a tribal homeland. The second statement tells us that the government in
empowered to grant the tribal people a homeland. But this statement by itself too cannot take us to
the conclusion. Even if we combine the two statements, we cannot conclude that the government
should reject the demand for a tribal homeland.
9. Ans. (d). Neither of the statements, if taken alone, can lead us to infer that agriculture is denied
the benefits of technology. On combining the two, we still cannot conclude that agriculture
does not receive the benefits of technology.
10. Ans. (d). The first statement tells us that Panchayats are elected bodies. From this alone we cannot
conclude that the Election commission will hold the elections. The second statement tells us that a
new law with regard to the panchayat elections has been passed. But we do not exactly know what
the law says.
11. Ans. (c). According to he rectification rules, a UA is converted to a PN to make it true and a UN
is converted to a PA. Since the true premise is a PN here, any inference would be uncertain.
12. Ans. (a). For the true premise, which is a PA, an inference like ‗No Ys are Xs‘ is false.
13. Ans. (b). The true premise would be ‗Some servants are obedient‘ and hence ‗some obedient
are servants‘ would be true.
14. Ans. (c). The true premise would be a PN and hence the given inference is uncertain.
Wo
(c)
Wo WI E
E
WI
(1) (2)
WI
Wo WI E E
Wo
(3) (4)
(d)
WI E
WI Wo/WI/E
Wo
WI
WI
(b) LL SI/V/LL
V
SI
―All south Indians have long life‖ follows in all the diagrams.
SI SI/V/LL
V
SI
SI
(c)
20. Ans. (d). The first statement does not tell us why the presidential form of government is worth
adopting over the parliamentary form. There is no information given as to whether the
presidential form of government will work properly. The second statement does not tell us why
a concentration of power is undesirable for India. As neither of the statements gives us valid
reasons, neither of them is forceful.
21. Ans. (b). 1 is not forceful, as it does not give a supporting reason. It should be saying why
looking to the interests of the central government alone is undesirable. 2 is forceful as we are
told that the governor is a vital link between the state and central government and hence
indispensable.
22. Ans. (d). the first statement does not give us a valid reason justifying the need for a legal ban
as such. The second statement is not forceful, as difficulty in identifying such parties is no
reason for a legal ban or not going in for a legal ban.
23. Ans. (c). Both the statements, though contradicting each other, give valid reasons for and
against non-alignment. Therefore, both are forceful.
24. Ans. (b). 1 does not tell us how being a bigger country translates into good relations with
neighbours. 2 gives us a valid reason as to why it is impossible to maintain good relations and
therefore it is forceful.
25. Ans. (d). 1 is an opinion (should) and therefore is a judgement. 2 is a categorical statement
and hence a fact. 3 is a judgement because of the word ‗must‘. 4 also has an element of
compulsion (must) and is a judgement.
26. Ans. (c). 1 is a judgement (must). 2 is a judgement because of the word ‗should‘. 3 is a cause-
effect statement, consequently it is an inference. 4 is a prediction and thus a judgement.
28. Ans. (b). 1 is a prediction and thus a judgement. 2 has an element of compulsion (have to be
kept.) and is again a judgement. 3 is a judgement because of the word ‗forward‘, which is used
as an adverb here. 4 is a prediction and is a judgement.
29. Ans. (a). The first statement can be concluded from the statement as we are told that national
integration cannot be achieved where there are contrasts. So, it is obvious that contrasts
hinder national integration. But we cannot conclude that integration is essential for India, as
the question statement does not mention India nor does it say it is essential.
30. Ans. (d). 1 cannot be concluded, as the question has nothing to do with India. 2 too, does not
follow as the statement does not tell us what the opinions of the rich vis-à-vis liberalisation are.
B D
D B RBD
R R
―Some bracelets are diamonds‖ and ―Some diamonds are bracelets‖ follow in both the
diagrams.
H
L L
H
K
K
H
H
L L K
K
―Some locks are hooks‖ and ―Some hooks are locks‖ follow in all the diagrams.
J
F C RCJ
W R
―All rabbits are jackals‖ and ―Some jackals are rabbits‖ follow in both the diagrams.
―All cupboards are costly‖ and ―Some costly things are cupboards‖ follow in both the
possibilities.
T
C MCT
M
M R
H H
R
M
UA + UA = UA
UA + UA = UA
A
Na Ne A
Ne Na
UA + UN = UN
S D D
AO S AO
UA + UA = UA
P U U
N P N
UA + UA = UA
46. Ans. (e). The statement is definitely false as the passage tells us that there are occasions
which compel judicial intervention. Therefore, it is definitely false that the courts have no right
to intervene.
47. Ans. (b). The last sentence in the passage indicates that the court would intervene when there
is executive inaction. But we have no direct information that the court has intervened and the
forcefulness with which the case for the court‘s intervention is stated shows that it has most
probably intervened.
48. Ans. (a). The passage tells us that there are critics who are against judicial intervention. Thus
it would be definitely true to state that there are people who are against judicial activism.
49. Ans. (e). The passage explicitly mentions that there are constitutional obligations and statutory
provisions relating to child labour. Thus statements to the contrary are definitely false.
50. Ans. (c). The passage makes no mention of whether the decision of the judiciary to intervene
in governance is conscious or not. Thus any inference in this regard would be uncertain.
Exercise 5(B)
1. Ans. (a). The argument concludes that most people are satisfied, on the basis of the response
that they received from people who were stepping out of the major airline terminus. They
assume that all those people have travelled by the national airline and hence take their
response to be relating to their satisfaction with the national airline. Option (a) proves the
assumption wrong, as according to this option, a sizeable number of private airlines also use
the terminus. That would mean that the people who were responding to the survey would have
expressed their satisfaction with the private airlines and not the national one. Option (b) does
not make any difference to the forcefulness of the conclusion, as the argument talks about the
satisfaction expressed by people who responded. Option (c) is irrelevant, as the argument
says that most of the people were satisfied with the flight that they got off. Option (d) does not
undermine the assumption underlying the argument. Option (e) is rather a strengthening
argument.
2. Ans. (b). It is clear that the argument assumes only two possibilities and none other than these
two. Statements I, II and III talk about bludgeon, library and Professor Moriarty, stiletto, lounge
and Ms. Peel individually. That would mean that anyone could have committed the murder
anywhere using a bludgeon, anyone could have committed the murder in the library using
anything, etc. The argument, on the other hand, assumes that the murder could have been
committed in the library by Prof. Moriarty with the bludgeon or it would have been committed
Ms. Peel in the lounge using a stiletto.
3. Ans. (e). The author‘s argument tries to say that the laws are not legitimate as they encroach
on others‘ ability to deal with their property as they wish. Option (e) points to the loophole in
the argument by saying that the whole community has legitimate stake in preserving old or new
trees. Option (a) would not weaken the argument as it says altering the landscape usually does
little to enhance the aesthetic or economic value. If it does make a difference in this case, it
would be worth altering the landscape. Option (b) is a strengthening argument. Options (d) and
(e) are irrelevant.
5. Ans. (e). The author‘s argument centres on an implicit causal relationship. He mentions the
impact of the business shift made by a company on the automotive sector in the area. Option
(a) can be eliminated, as there is no question raised by the author. The facts given are direct
and are not stated in a convoluted manner. So option (b) can also be ruled out. The author
does not make any comparison. So option (c) can also be ruled out. The author talks about the
impact on just one sector. ‗Socialistic point of view‘ is too broad a phrase to be used.
6. Ans. (c). The argument assumes that there would be no snag in using the endowments for
reducing tuition fees and that the donors have given these institutions the right to use them in
any area. Option (c) undercuts this assumption by citing the restrictions specified by the
donors. No other option undermines the primary assumption made in the argument.
7. Ans. (d). The given argument has successive cause and effect statements with negation given
in each one of them. Only option (d) has such a pattern.
8. Ans. (b). The argument tries to disprove the existence of UFOs and flying saucers purely on
the basis of photographs that have been shown to lack authenticity. So the argument assumes
that this is the only way of coming to know about or concluding the existence of flying saucers.
Option (b) points to the flaw in the argument. Option (a) does not attack the basic assumption.
Option (c) would not be a weakening argument as it talks about only some people. Option (d)
talks about a hypothesis and is weak in itself. Option (e) is a judgement and does not address
the assumption of the argument.
9. Ans. (d). The argument advocates instilling a scientific way of thinking saying that it would lead
to healthy scepticism towards religion and tradition. It is obvious that the argument considers
such an attitude a favourable one. The argument also assumes it is possible for students to
assimilate a scientific way of thinking at young age. Statement I is incorrect, as the argument
assumes that an exposure to the scientific temper will lead to scepticism towards religion and
tradition. The pattern of the argument is like ‗If x, then y‘. That is not the same as ‗If not x, then
not y‘, which is the pattern of statement I.
10. Ans. (a). Napolean Bonafide refutes Cardinal Richloo‘s argument by confidently stating that it
would have been improbable for Louis IV to have not enjoyed a reputation in spite of the works
that he produced. He overlooks the fact Louis IV might have chosen not to proclaim his
authorship and talent. Option (b) is rather the assumption made by Cardinal Richloo and not
by Napolean Bonafide. Option (c) can be eliminated, as Napolean rather assumes that it is not
possible that Louis IV did not enjoy a reputation and not that he did not have the required
artistic gifts. Option (d) is also rather an assumption made by Cardinal Richloo. Option (e) is
not anywhere close to the assumption made by Napolean.
11. Ans. (a). The argument is quite optimistic about improvement in the two-wheeler industry.
Option (a) says that it is indeed highly probable, as the industry has had a track record of
bouncing back. Option (b) would be irrelevant, as we are not told whether Hero Honda is a
two-wheeler industry Option (c) would also be irrelevant, as we are not told anything directly or
substantially about the two-wheeler industry. Option (d) is completely off the track.
13. Ans. (d). Arguments A and D talk about things going hand in hand. In argument A, it is
technology, which is given as a part and parcel of life and in argument B, it is health that is
talked about as integral part of life.
14. Ans. (d). The given argument says that the ―conventional units of measurement are not suited.‖
So it can be inferred that they are inadequate and new units have been evolved to serve the
purpose. All that is given to us in the argument is that the conventional units of measurement
are not suited. That does not mean they are finite. There is no information given about that
aspect of the units of measurement. So option (a) can be eliminated. Option (b) is irrelevant as
it talks about global environment. The argument does not even hint that the units used so far
have been found to be faulty. So option (d) can be eliminated.
15. Ans. (a). The statement ―Yamma gave up power to fight corruption‖ Has an underlying
sarcastic tone. That is evident from the following statement, which has negative connotations.
Option (a) gives some positive information about Yamma thus refuting the author‘s statement.
Option (b) fails to understand the sarcastic tone used in the argument. Option (c) also does not
get the essence of the argument right. Option (a) is much stronger than option (d).
16. Ans. (d). The argument says that inventions, trademarks, etc. are called intellectual property
once they are given legal protection and the conclusion is that such protection ―is always
necessary‖ to ensure further progress of civilisation. So the assumption is that intellectual
property results in further progress of civilisation. Option (a) could be a conclusion. It is not the
backbone of the premise but rather what the argument states in the premise. Option (b) is
irrelevant as the argument does not concern itself with whether the patent rights are given to
make sure that the individual uses the invention for his own purposes.
17. Ans. (c). Since the argument says that the US has always been involved in Kashmir when seen
from the South Asian vantage point in spite of the sneering attitude shown by US officials, it
can be inferred that the US has an interest in South Asian affairs. Interest in South Asian
affairs is implied, as the argument talks about involvement in the form of Pakistan‘s cause. The
argument does not give enough evidence to support option (a). The argument talks about US‘s
involvement in Kashmir but does not say if it has incited rebellion in the area. So option (b) can
be ruled out. We cannot infer from the passage that US has always backed Pakistan.
18. Ans. (c). The argument talks about Tolstoy being classical in the same way that Homer‘s
Chaucer‘s and Spencer‘s are. To substantiate this statement, facts like his art moving to
instinctive rhythms and his preoccupation with questions of religion and belief are cited. So
option (c) can safely be inferred. We cannot infer from the argument whether these poets
influenced Tolstoy or whether the influence was deep. So option (a) can be ruled out. There is
no information given in the passage to support option (b). Option (d) leaves out another
important reason due to which Tolstoy‘s art is considered classical.
19. Ans. (c). Both these arguments project NIIT‘s ability to ensure that your business grows and
that you have an edge over the competitors.
20. Ans. (d). The argument merely talks about how it is difficult to stick to integrity given the
hardship that it entails. It does not say that not following the path of integrity will necessarily
result in wealth chasing a person. So option (a) is eliminated. The argument does not say that
it is imperative to lie nor does it say it is desirable for your family not to suffer from a life of
hardship. The argument does not give a definite cause-effect relationship between these two.
Thus option (b) can be ruled out. Since we are given no information about the nature of the
world or whether it is a world of riches and consumerism, option (c) can also be eliminated.
The argument says that the person who sticks to the narrow path of integrity is remarkable. So
option (d) would be a logical inference.
1. Ans. (d). The conclusion tells us that the President has already declared martial law. Neither of
the statements, taken in isolation or together, brings us to this conclusion. All that we are told
that is that the President can declare martial law.
2. Ans. (d). The conclusion is that the issue of leadership was decided. The first statement tells
us that this particular issue was quite important. This alone does not take us to the conclusion.
The second statement tells us that the same was to be decided by the majority. This too, is not
sufficient to lead us to the given conclusion. The two taken together too cannot lead us to
conclude that it was decided.
3. Ans. (c). The first statement tells us that political parties contest elections, while 2 tells us that
elections arouse political consciousness. Therefore, the two taken together can lead us to
conclude that political parties arouse political consciousness.
4. Ans. (a)
Options (a) and (b) satisfy the rules for testing the validity of the argument while options (c)
and (d) do not. K
K F.B
(a)
F.B P
P
F.B
(1) (2)
P K
P F.B K
(3) (4)
―Some four blade fans are not polar fans‖ follows in all the possibilities.
(b)
G G
F.B P F.B P
F.B
P F.B/G P
6. Ans. (c)
Options (a) and (d) are eliminated, as the arguments in these options are inconclusive.
S C.S/S/A
C.S
(b)
C.S
SA
SA
C.S
C.S
―Some college students are artists‖ remains a constant in all the possibilities.
7. Ans. (d)
Option (c) does not satisfy the rules for testing the validity of the argument. The argument in
option (a) is inconclusive.
(b)
D SD
M M
S
―Some surgeons are not musicians‖ does not follow as Surgeons and Musicians are mutually
exclusive sets in both the possibilities.
D
D M S
S M
8. Ans. (c)
The statement defines the relationship between ―non-whites‖ and ―black‖. Nothing can be
inferred about sets like ―non-blacks‖ and ―white‖.
9. Ans. (a)
The given statement is a rewording of ―All peace loving countries are third world nations‖.
After rectification, this statement is converted into ―Some peace loving countries are not third
world nations‖. The inference, which is a repetition of the above statement, is true.
13. Ans. (a). 1 is a categorical statement. 2 is a prediction and also has an element of compulsion.
3 is again a categorical statement. And 4 is a cause-effect statement and thus is an inference
14. Ans. (d). 1 has an element of compulsion (should) and is a judgement. 2 is also a judgement
as it uses the superlative (deepest feeling), 3 is a cause-effect statement and therefore an
inference, and 4 is an opinion and thus a judgement. Moreover it has superlative degree.
15. Ans. (d). 1 is a judgement as it is an opinion (should), 2 is a categorical statement, 3 is a
cause-effect statement and thus an inference, and 4 is a judgement as it includes an element
of compulsion.
16. Ans. (c). 1 is a categorical statement, 2, 3 and 4 are cause-effect statements and therefore,
inferences.
18. Ans. (d). Making health an end in itself does not translate into being health conscious.
Moreover it is not clear from the text whether it is the modern age that we are reading about.
And nowhere in the passage is it mentioned that very little people fall ill. Therefore, neither of
the statements follows.
20. Ans. (d). The given statements do not clearly equate the social objective of economic goals
with eliminating economic inequalities. Moreover, the first statement talks about equalities and
is not very specific in its reference to economic inequalities. So it might or might not refer to
such an equality. For this reason conclusion 1 cannot be inferred. Conclusion 2 is way off the
track as it talks about discipline and the relative importance, both of which are not mentioned
in the statements.
21. Ans. (b). Though a UA and a PA can be inferences for an argument, which has two UAs as
premises, we notice that the circle of lamps need not necessarily be inside the circle of milk. It
can be inside the circle of milk or outside the circle of milk or they might even intersect with
each other as long as milk encircles the set of goats. So inference 1 is valid and from the
above-mentioned possibilities, it is clear that inference would be valid, as that would remain a
constant in all the possibilities.
22. Ans. (a). Since prams include trams and no prams can carry passengers, it is logical to
conclude that no tram can carry passengers. Inference 2 would be invalid, as that would mean
that other trams may or may not carry passengers while we can clearly infer from the argument
that they can never carry passengers.
23. Ans. (a). The pattern of the argument and the pattern of the conclusions are the same as the
one in question no. 22.
24. Ans. (a). The set of bookworms is included in the set of first divisioners, which in turn is
included in the set of those who have been awarded prizes. So it is obvious that all bookworms
have been awarded prizes. For the same reason, the second conclusion is invalid as that
would imply that the other bookworms have not been awarded prizes and that goes against the
information given in the premise.
25. Ans. (d). From the second premise, it is clear that the circle of those who have to suffer in life
has to just intersect with or be encircled in the set of faithful people. It might or might not touch
the circle of servants. So both the conclusions would be uncertain.
26. Ans. (b). Both the statements are UAs. So it is clear that the missing premise has to be a UA
and since the term John has occurred twice, the missing premise would contain only the terms
shrewd and politicians. So the missing premise has to be option (b), as politicians would
include John.
27. Ans. (a). Here again, the missing premise has to be a UA for the same reason as above. Since
the term he has occurred twice, the missing premise has to have the terms ‗lucky‘ and ‗those
who have won the lottery‘. Option (a) would thus be the premise and ‗all who win the lottery‘
would include ‗him‘.
28. Ans. (a). The missing premise has to be a UA again. The term ‗she‘ has occurred twice and the
missing premise has to contain the other two terms. ‗All good-natured‘ would include her.
29. Ans. (a). Since the conclusion is a UN and the given premise is a UA, the missing premise has
to be a UN too. Since the term ‗he‘ has already occurred twice, the terms ‗efficient‘ and
‗sacked‘ should be used in the missing premise.
31. Ans. (a). The first statement is forceful as we are given a reason as to why corporal
punishment is not preferable to other methods. But the second statement is not forceful
enough, as it talks about no fear of punishment while the question is specifically about banning
of corporal punishment and not banning of all kinds of punishments.
32. Ans. (d). Neither of the statements gives us valid supporting reasons. We need to know why
science has to be made compulsory in schools as well as colleges too. Neither of the
statements convinces us in this regard. The second statement does not say why the study of
science would make them mechanical in outlook.
33. Ans. (c). Both the arguments are relevant to the question and they give supporting reasons.
Thus both are forceful.
34. Ans. (d). 1 is not forceful, as it does not give any valid reason at all. All it does is that it almost
repeats the question. 2 is not forceful as it does not give a supporting reason for its statement.
35. Ans. (b). 1 is not forceful as just because the judges are of great calibre, it does not mean that
they are independent. 2 is forceful as we are told why the judiciary is no longer independent.
36. Ans. (a). 1 is implied as we are told that he was stoical in facing misfortunes.
Thus it is implied that there were misfortunes. 2 is not implied as we do not know whether that
person faced the misfortune bravely. Stoicism and bravery are not the same.
37. Ans. (b). 1 is not implied as we are not told how many wise rulers are there. But we know that if
it is a wise ruler, who pays no attention to sycophants, then there are some rulers that do so.
Thus 2 is implied.
38. Ans. (a). We know that the mob is attempting to get out of the train. Thus, it is implied that they
were in the train. But 2 is not implied as we do not know for sure whether the train is moving or
not.
39. Ans. (d). All that we are told is that we should not condemn things we do not understand. We
are not told what are the things we can condemn nor are we told that there are people who do
not understand everything.
40. Ans. (b). The statement tells us that the US is an affluent nation. Therefore, it is obvious that
US is a nation. But we are not given any information about the affluence or the lack of it of the
other nations.
41. Ans. (c). The only statement that is relevant here is the ―either or‖ statement given. Statement
B (Kamal, the philosopher, will not stink) is ―not y‖ and should be followed by ―x‖- Kamal will
think (If not y, then x). This combination, i.e. BA, is provided by answer option (c). Consider
Statement D (Kamal, the philosopher, will not think) – ―not x‖. This should be followed by ―y‖ –
―Kamal, the philosopher, will stink‖ (If not x, then y). However, there is no option like DC. So
the answer is (c)
42. Ans. (c)
The only relevant statement here is the ―either or‖ statement given in the passage.
Statement A (The computer is not connected) is ―not x‖ and should be followed by statement B
(The work is delayed) which is ―y‖. The other possible answer would be CD (If not y, then x).
Since this is not given as an answer option, the answer is (c).
44. Ans. (c). The argument in the passage is that taking drastic gambles is more likely to fail than
succeed. (a) (b) and (d) are quite irrelevant to the argument. Option (c) talks about
revolutionary changes that are not required implying that they are more harmful than
beneficial. ―Revolutionary changes‖ would mean the opposite of a step-by-step and careful
approach.
45. Ans. (c). Economic nationalism would involve promoting indigenous goods. (c), which tells us
that the government has introduced a huge cut in import tariffs, and thus encouraging imports,
goes against this argument and thus weakens it. Options (a) and (b) are rather strengthening
arguments.
46. Ans. (c). The argument is lateral, as it is relevant to the passage. The passage is concerned
with the relation between the government at the centre and the bureaucracy. And it is probably
in order to ensure loyalty on the part of the latter that the former changes it according to
convenience.
47. Ans. (d). This argument is irrelevant, as we cannot make any inference in this regard from the
passage. We do not know if it was willing decision on the part of the bureaucracy or whether it
was forced to do so.
48. Ans. (a). This argument is an upstream one, as the passage elaborates on how exactly the
bureaucracy is related to the political scene. Thus it is an effective introductory remark.
49. Ans. (b). This argument is a downstream one, as the whole passage leads us to conclude on
the basis of the impact of the ruling party on the bureaucracy that the bureaucracy would be
jittery about its fate every time there is a change in the government.
50. Ans. (d). This argument is irrelevant, as whether or not the UF government was afraid to take
such a step is not mentioned in the passage. For all we know such a step might have been
unnecessary for the UF government.
Exercise 6 (B)
1. Ans. (d). Option (a) is a strengthening argument. Option (b) is irrelevant, as it does not talk
about the sale of books and how it is affected or not affected. Option (c) is also irrelevant for
the same reason. Option (d) comes closest to being a weakening argument as it hints that
people have come to spend more money on CDs and books rather than on any other form of
relaxation. That would mean that in spite of the CD-ROM literature boom, sales of books would
not be drastically affected. Option (e) is also more of a strengthening argument.
2. Ans. (e). Option (e) would mean that PMI has not been adapting to change/ software
development as much as is projected. Options (a) and (d) talk about a majority of the people
who are not interested in software development. That would be irrelevant, as PMI would still
make a significant difference to those who are interested. Option (c) would also not be a
weakening argument, as the argument just implies that whenever there is a development in the
software field, PMI helps one stay in touch with it. Option (c) is not as strong an undermining
argument as option (e). Moreover, we are not given any information about whether it matters to
have links with the other nations. The case might be such that software development happens
only in European nations.
4. Ans. (b). The given argument concludes that most of the heinous crimes were committed by
young novices whereas this option says that most of such crimes were committed by middle
aged men and not young novices. Option (a) cannot be a weakening argument as the
argument already talks about 69.2% of the accused. Option (c) does not help in attacking the
conclusion drawn about the young novices. Option (e) would be irrelevant, as there is no
information given in the argument to infer that the conclusion drawn is about the world. It might
be about just one small area.
5. Ans. (d). The nun claims that Mother Teresa deserves to be canonised on the basis of what
she observed as a miracle. Option (d) corroborates this fact by giving evidence to show that it
is indeed a miracle. Option (a) says nothing about Mother Teresa or her ability to perform
miracles. No time frame is given in the argument and hence option (b) would make no
difference. Moreover, this option also does not say anything about Mother Teresa‘s ability to
perform miracles. Option (c) seems close but there is no information given about the
authenticity of the reports. This option also does not say if the case in point is actually a
miracle. Option (e) is irrelevant.
6. Ans. (c). The nun claims that Mother Teresa performed a miracle. If the child is severely
handicapped, it goes to show that his survival was a mere statistical possibility and not a
miracle. Option (a) is not as strong as option (c). The unwillingness on the part of the doctors
to testify does not disprove her claim. If they had refuted her claims with evidence then this
option could have been a weakening argument. Option (b) is irrelevant, as it says nothing
about Mother Teresa‘s ability to perform miracles or about the nun‘s claims. Option (d) can
also be eliminated, as taking charity from people with dubious backgrounds says nothing about
her ability to perform miracles. Option (e) is also eliminated for the same reason.
7. Ans. (d). The parents assume that a dropout can also succeed in his life, which goes to show
that they feel education can be done away with and that would not entail adverse
consequences. Option (d) attacks this assumption by saying that education is indispensable.
Options (a) and (b) rather weaken Gates‘ position. To cut the Gordian knot is to take shortcuts.
Option (c) is merely a judgement on ―today‘s parents‖ and does not directly strengthen Gates‘
position. Option (e) is rather irrelevant as Gates places an emphasis on high school education
and not so much on ―expensive and competitive college education‖.
8. Ans. (c). From the ―Either or‖ statement we can infer that if Govinda does not sing, he dances
at the discos. (If not x, then y). From the If then statement we can infer that if Govinda does not
sing, then Rani acts in Baba‘s music video. (If not y, then not x).
9. Ans. (d). The relevant statement here is ―Midge will wear pink if Big Moose wears beige and
Dilton wears black.‖ There are two causes and one effect. It would mean that if both the
causes occur, the effect would follow. If the effect does not occur it would mean that both the
causes or that one of the causes did not occur. So option (d) would be logical.
10. Ans. (c). This conclusion follows from the first statement, using the same logic.
11. Ans. (c). The inference is rather obvious, as the argument says that in spite of the profusion of
ideas ―nothing ever gets done‖. Option (a) is not conclusively supported by the argument. We
are told about the politicians and bureaucrats but nothing specifically about the ambassadors.
Since the argument does not say anything about India, option (b) can be eliminated. The
argument does not assign a reason for not implementing the ideas. So option (d) can also be
eliminated.
13. Ans. (a). Option (a) says that most children feel disoriented when they grow up and implies
that it was due to a lack of an upbringing which did not keep them close to their own country.
Thus option (a) endorses the assumption. Option (b) does not say why missing out on real joys
of growing up in the open outdoors is so desirable and what the lack of it would result in.
Option (c) is also not as strong an option as (a). It does not say for example why real-life
adventures are more desirable than the ones in the book. Option (d) also does not give as
compelling a reason as option (a).
14. Ans. (a). The argument says that the apparent impression is that patriotism has lost its hold but
it actually has a firm base in the minds of the young Indians. Option (a) corroborates the same
point and is thus a strengthening argument. Option (b) talks about people who were
interviewed and since we are not told how many people were interviewed, it would not be an
effective strengthening argument. Option (c) would not help in strengthening the argument as
the themes of just two songs is not evidence enough to conclude that patriotism is prevalent.
15. Ans. (a). The argument assumes that the survey in question is a reliable one. Option (a) most
effectively undermines the argument by calling the validity of the survey into question thereby
hinting that the conclusion is not likely to follow.
16. Ans. (d). The argument claims to make heart, blood cells, etc in the laboratory. The
assumption thus is that ―parts of nature‘s programme‖ can be ―mimicked.‖ So option (d) is right.
Option (a) interprets ―spare parts‖ literally. Option (b) is rather a conclusion and not an
assumption. The argument talks about replacing the damaged tissues and not about creating a
human body. So option (c) is not an assumption.
17. Ans. (e). Option (a) weakens the validity of the research and thus is more likely to strengthen
the popular belief. Option (b) also strengthens the popular belief. Option (c) does not explain
why living independently stalls the ageing process. It also does not say that those who are not
living independently will age faster. So this option is irrelevant. Option (d) just hints that the
result of the research in question is likely to be true. But it does not say whether the 330
volunteers who were examined are representative of all elderly men and women.
18. Ans. (b). This option is a rewording of ―All those who enjoyed the movie are cowherds.‖ That
would mean that Santa who enjoyed the movie is also a cowherd. Option (a) talks about all the
people who wanted to see the movie. That would not necessarily include Santa whom we are
told ‗enjoyed‘ the movie. Moreover it talks about people who tend cattle which would refer to
tending any horned or hoofed animal and not necessarily a cow. Option (c) is ridiculous and
irrelevant. Option (d) is eliminated for the same reason.
19. Ans. (b). The argument hints that Bablu and Ulu are uncultured, as the professor says he is
cultured and would not deign to talk to them. In the latter‘s case, evidence is given about the
kind of language that he uses to substantiate the claim. Option (b) strongly posits the fact that
they are indeed uncultured. Option (a) simply reiterates the information given in the argument
and does not talk about Ulu either. Option (b) is better. Since the argument focuses on being
cultured and uncultured and not about the professor‘s and others‘ ―standards‖ in general,
option (c) would not help in strengthening the argument.
Exercise 7 (A)
1. Ans. (a)
Options (b) and (c) do not satisfy the rules for testing the validity of the argument. The
argument in option (d) is inconclusive.
2. Ans. (c)
Option (b) does not satisfy the rules for testing the validity of the argument.
(a)
H H
H
A G
A G A G
(b)
H
H
A G
A G
H
A
(c)
H AGH
G
H G AHG
G H
A A
(d)
3. Ans. (d)
Option (a) does not satisfy the rules for testing the validity of the argument and the argument in
option (b) is inconclusive.
(c)
S. W.S S/W.S/S.S
W.S S S.S
S S
(d)
S/W.S/S.S
W.S
S.S
―All soaps are water soluble substances‖ follows in both the diagrams.
4. Ans. (a)
Options (c) and (d) are eliminated, as these arguments are inconclusive.
S M P
S
P
M
(b)
P
M S MP S
P P
M S M S
―No S is P‖ is uncertain.
5. Ans. (b)
Options (a) and (c) are eliminated, as the arguments in these options are inconclusive. Options
(b) and (d) satisfy the rules for testing the validity of the argument.
R
F
R F
(b)
I
R F
(d)
F
I
R
6. Ans. (b). 1 is not forceful, as we are not provided with a valid supporting reason justifying a
government take-over of all schools. 2 is forceful because we are given a blanket statement
telling us that all government undertakings become inefficient and this would hold true for take-
overs of schools as well.
7. Ans. (d). Neither of the arguments gives supporting reasons for or against a ban as such. Thus
neither is forceful.
8. Ans. (c). Both statements give reasons for and against Military training. Thus both arguments
are forceful.
9. Ans. (d). Neither statement gives reasons for making school education compulsory. Therefore
neither of them is forceful.
10. Ans. (d). Neither statement gives reasons for imparting adult education on a war footing.
Therefore neither of them is forceful.
11. Ans. (d). Neither conclusion follows from the given statements. We are not told that the writer
does not expect rewards for his works. And we are told that the work of art is in itself a reward
but not that it is a thing of beauty.
12. Ans. (d). The statement doesn‘t make any reference to how effective the quality of education is
and therefore we cannot make any conclusions in this regard. Thus neither 1 nor 2 can be
concluded.
14. Ans. (d). The relationship between the first and second sentences of the statement is not clear.
We cannot definitely conclude that the ascetic's life is one of discipline. Thus we cannot
conclude 1. And it is not mentioned in the statement that poverty is generally praised. Thus
neither of the statements can be concluded from the question.
15. Ans. (a). We can see that the traditional role of the university is different from what it is now.
Thus the first statement can be concluded. But it is not mentioned that the university is a seat
of learning. And so we can not conclude statement 2.
S CTS
T
C
―Some sturdy things are tables‖ follows but ―Some cables are sturdy‖ is false in both the
diagrams.
R R CR
R
C P C P P
CR P C P
In diagram (4) there are four other possibilities. So ―All coins are paper currency‖ and ―Some
coins are paper currency‖ are uncertain.
H/A/F.R
F.R A
A F.R
H H
―All animals are fast runners‖ is uncertain. ―Some fast runners are animals‖ remains a constant
19. Ans. (a). ―Some good things are plants‖ is true in both the diagrams while ―Some plants are
good things‖ is false in both.
22. Ans. (d) Here again the given data is insufficient to find the triggered event.
UA + UA = UA
Sy Sc Sc
P Sy P
UA + UA = UA
R B B
T R T
UA + UA = UA
31. Ans. (c). The true premise will be a PN for which every inference will be uncertain.
32. Ans. (d). The given statement makes no reference to what has happened earlier on and so it
doesn‘t imply that in earlier years the position of science was unstable. The second implication
is interpreting the statement quite literally and is irrelevant. Thus neither of the statements is
implied.
33. Ans. (d). The first implication ‗Man is the product of evolution‘ would not be implied. The use of
the definite article ‗the‘ would imply that man (Homo sapiens) is the only product of evolution,
which is not implied by the given sentence. Statement 2 is also not implied since we are only
told that man is not the end of evolution. It is possible that even if man is not the end of
evolution, something else could be.
34. Ans. (d). The first implication would be wrong since all that we are told is that we lack
sportsmanship. The statement does not indicate that sportsmanship is necessary. Neither
does the statement indicate that sportsmanship will be present in future. Thus neither of the
statements is implied.
35. Ans. (a). The first implication would be right. When we are told that man must become an
integrated individual it is implied that as of now he isn‘t so. But the second statement would not
be implied since we do not know whether it is actually possible to make man integrated.
36. Ans. (a). The first statement would be implied since we are told that there was a ‗divergence of
opinions‘. Whereas the second statement would not be implied as nothing is mentioned which
will tell us what happened after.
37. Ans. (b). The second statement, which tells us that civil servants implement policies, is alone
enough to take us to the given conclusion.
38. Ans. (d). Neither of the statements by themselves would take us to the conclusion, which
explains the concept of human development. Even if we were to take both the statements
together we still can not reach the given conclusion.
39. Ans. (c). If we take both the statements in unison, we can arrive at the conclusion that a joint
stock company is a form of business organisation.
46. Ans. (c). The passage does not talk about all kinds of preventive medicine. Moreover it
indicates that certain things are overemphasised and does not say that everything needs to be
chucked. So nothing can be inferred about all kinds of medicine.
47. Ans. (b). The passage states that modern researchers have cast doubts on the 'fancies'
propagated in the name of preventive medicine. The implication is that there probably has
been research done on preventive medicine. Therefore this inference would be probably true.
48. Ans. (c). This inference is uncertain since there is no information in this regard. We can not,
from this passage, infer what exactly is required to win athletic tournaments.
49. Ans. (e). This inference is definitely false as the last line of the passage tells us that the
general quality of air is indeed affected by pollution.
50. Ans. (b). The passage states that a lot of nonsense has been talked about healthy life, one of them
being that we were told to take 18 chews. Implication being that it is not necessary to chew food for
so long.
1. Ans. (c). The consultant‘s emphasis on reducing the time taken to do the work goes to show
that he assumes that the workers are paid by the hour and not for the actual output. Since
nothing is said about the strength of the sheets in increasing efficiency, option (a) is irrelevant.
Options (b) and (d) are also irrelevant. The argument does not talk about increasing the
quality. So option (e) is irrelevant too.
2. Ans. (c). The professor concludes that the science graduates are more interested in art simply
because they attend art classes regularly. So option (c) is the assumption. Options (a), (d) and
(e) are judgements and do not address the assumption of the argument. Option (b) is nowhere
close to the assumption underlying the argument.
3. Ans. (a). The professor concludes that science students in the University are more interested
in art based on the students who attend her classes. So it is obvious that she has taken her
students to represent all science graduates in the university. Option (b) is irrelevant. Option (c)
does not explain what the special demand is. Had it said that the students were forced to
attend science classes, it could have been a weakening argument. Option (d) can be
eliminated, as the argument is about students being interested in art and not about the reason
that underlies the interest. So if the professor does not explain the reason it makes no
difference to the claim that is made. Option (e) is merely a judgement.
4. Ans. (b). The argument claims that there is nothing reprehensible about the poor spending
more on lotteries as they also win more. Option (b) attacks the argument by saying that only a
small fraction of the poor who buy lottery win money. Option (a) talks about the status of a man
after winning while the argument is about whether lotteries are favourable for the poor. Option
(c) is irrelevant for the same reason. It does not say that lotteries are favourable for the poor.
Option (d) is also irrelevant. It does say whether the nations that have lotteries are fair to the
poor. Option (e) still does not say that the poor who buy the lottery tickets do not stand to lose.
5. Ans. (e). The conclusion assumes that the statement ―All the members of the NPAC signed a
petition‖ means that only members of the NPAC signed the petition. ―All the members of the
NPAC signed a petition‖ is the same as ―All the people who signed the petition are members of
the NPAC. Only option (e) identifies this flaw.
6. Ans. (e). The Indian government seems to have assumed that passing the laws will help arrest
the decline in the population of turtles taking hunting to be the only cause that would result in a
decrease in the population. Option (e) cites another cause and thus explains the paradox.
Option (a) would not resolve the paradox, as the poachers who are released after some time
would still be forbidden from hunting turtles. Option (b) would rather mean that there should
not have been a decline and thus would not explain the paradox. Option (c) is irrelevant. The
argument leaves a question about why there was a decline between the period 1975 and 1990
and what happened before 1975 would thus be irrelevant.
7. Ans. (a). The argument claims to make a significant difference to the manufacturing of petrol
by cutting down on the use of tar for metalling the roads. Option (a) undermines the claim by
hinting that a very small amount of tar is used and thus would not make much of a difference to
the manufacturing of petrol. Option (b) does not attack the claim but merely posits a solution.
Option (c) would rather be irrelevant to the claim that a significant difference can be made to
the manufacturing of petrol by cutting down on the use of tar for metalling the roads. Option (d)
and option (e) also do not make any difference to the claim made.
8. Ans. (a). The answer is obvious due to the use of the phrase ―as a result‖.
10. Ans. (b). The argument claims that a comprehensive fraud-prevention plan will help companies
in heading off fraud. Option (b) says that such measures, which have already been
undertaken, have not helped in forestalling fraud. This is the only option which most effectively
and directly attacks the claim made.
11. Ans. (b). ―Technology is no longer a set of back-office systems‖ would mean that it was once a
part of back-office systems. The rest of the option can also be inferred as the argument says it
is now a critical part. Option (a) is irrelevant. Option (c) is incorrect as the argument says
technology is important but not that it is indispensable.
12. Ans. (b). The argument claims that the net as a medium for shopping presents considerable
competition for retail merchants. Option (b) supports this claim by talking about how it cores
over traditional purchasing methods. The fact that the businesses involved in e-commerce
have grown in number as suggested by option (a) does not necessarily mean that retail
business has taken a beating. Option (c) is absolutely irrelevant. Option (d) hints nothing about
retail merchants. It rather weakens the claim made in the argument.
13. Ans. (d). The argument says that when you do not have breakfast you end up craving for more
food resulting in putting on more weight. Option (d) captures the essence of the argument. The
argument says nothing about how many people keep eating between meals. So option (a)
cannot be inferred. The argument just implies that nibbling foods like biscuits puts one at a risk
if gaining weight. But it does not mean that it necessarily has to be fattening. In other words,
the argument does not say that only fattening foods make one put on weight. It also leaves
room for a possibility that foods, which contain a lot of sugar or carbohydrates, can make one
put on weight, etc. The argument does not say conclusively that skipping breakfast makes one
hungry. It rather says you will keep nibbling foods if you feel hungry.
14. Ans. (e). From the given argument we cannot figure out or infer whether doing things like doing
things like sprucing up roads and building a 100-bed hospital is a sign of a dedicated
politician. Moreover, the option is judgmental and such options are not likely to be the
answers. Option (b) also passes a judgement. Moreover, we cannot infer that the attempts
made by the PM to spruce up the roads etc. go to show that there was nothing in his home
constituency that one would stress upon to call it a developed town. There is no evidence in
the passage to infer option (c).
15. Ans. (c). The argument stresses a lot on ―going back‖ to the humble days of the bicycle. The
assumption obviously is that people have not been using bicycle. Since nothing is said about
environment, option (a) can be eliminated. Option (b) is rather a conclusion and not an
assumption. Option (d) states a marginal assumption and not the central assumption
underlying the kernel point in the argument.
16. Ans. (c). Option (a) cannot be inferred, as it is illogical to conclude that the TV helped him
grow up in the north. He landed there on his own according to the passage. The argument
does not exactly say that Lost and Found columns are being televised. It hints that they have
similar objectives. Option (c) can be inferred because the spokesperson says that Sony‘s
―Missing‖ has used the power of positive television to help a family. And the initial part of the
passage talks about another channel that airs a similar programme. These statements imply
that TV channels are now leaning towards new uses of the TV.
18. Ans. (d). The argument claims that starvation is never officially accepted and is labelled as
‗undernourishment‘ to give it the air of an illness. Option (d) talks about the government‘s effort
to pass off a starvation as an illness. No other option endorses the claim made.
19. Ans. (c). Option (a) is rather a judgement and not exactly an inference. There is not information
in the passage to support option (b). Option (c) can be inferred as the passage talks about how
crop boom is unfavourable for the farmers and that K.N. Nataraja had to throw his tomatoes
rather than sell it for practically ‗nothing‘. So it is clear that the price of tomatoes had gone
abysmally low due to the cash boom. The passage says ―farmers seldom if ever had it so bad.‖
So option (d) which is a generalisation implying that crop boom always spells ruin for the
farmers cannot be inferred.
20. Ans. (d). The argument highlights more than anything else the ability of Bio-ceramic material to
keep vegetables fresh. The assumption is thus that people are interested in a fridge that will keep
food fresh for long. The argument does not concern itself with good health and thus option (a) is
irrelevant. The ability of InfraRed rays to keep food fresh is stated as a fact and is thus not an
assumption. Moreover option (b) would rather be an inference. The argument is not about the
‗safety‘ of vegetables due to which option (c) can also be eliminated.
Exercise – 8 (A)
1. Ans. (c). Both the statements are forceful since both of them are relevant to the given
statement. Education should be free in Indian schools and colleges since people in this
country are poor and it should not because the people lose value for the things that are got
free of cost.
2. Ans. (d). Both these statements are too general and do not address the question at hand.
Simply because something is prevalent for quite some time it is not necessarily right. And
being job oriented is not the only justification for an education system to be right or wrong.
3. Ans. (a). Only the statement one is forceful. If the examination system is changed to an open
book style then it would save the children from the boring and pointless task of mugging up
answers. The second statement says that it would take them away from the real knowledge,
which does not necessarily follow.
4. Ans (b). Only statement 2 is forceful since it addresses the given statement in the practical
light while statement 1 assumes that the students are immature.
5. Ans .(d). Both the statements are not forceful since they are not directed necessarily towards
the question at hand. Vocational education guarantees jobs to a certain extent and not
completely besides this may or may not be the only way to solve the problem of
unemployment.
UA + UA = UA
―Only super powers have nuclear weapons‖ is the same as ―All countries that have nuclear
weapons are super powers‖
7. Ans. (d) P D
D H
8. Ans. (b)
S R R
H S H
UA + UA = UA
9. Ans. (a)
D
W
D d W d
UA + UN = UN
D – destructive; d – desirable
UA + UN = UN
F – fat; f – fast
L
C
L L
U
C C U
U
CL U
U CL
(4) (5)
B LHB
L H
―All locks are black‖ is true while ―All black things are locks‖ is uncertain.
13. Ans. (d). The information given is that All E and all I are B but we cannot say that these two
sets are necessarily subsets of each other.
B
E I
Pi
I P
concluded.
L
E H
16. Ans. (a). If X then Y. X is given so it has to be followed by Y because when X occurs then Y
occurs. Statement 2 says, not Y. Therefore (b) does not follow.
17. Ans. (d). From the statement ―If X then Y‖ we can conclude ―If not Y, then not X.‖ Y is given to
us from that we cannot conclude anything about X since Y is independent of X. X may or may
not have been the cause. But if X occurs then Y has to occur.
18. Ans. (d). We know that when it rains we will have a good crop. If X does not occur we cannot
conclude anything since it is a totally different situation and no information is available to us
from the given statement about what will happen when X does not occur.
19. Ans. (a). Clearly going by the rule we can say that the answer is (a) The premise given is If X
then Y. If not Y then as per the rule the answer has to be not X which is statement number 1.
26. Ans. (c) The U.A.-false converted to true is a P.N. i.e. Some problems are not knotty. We know
that for a P.N. all inferences are uncertain.
27. Ans. (c) Again a false U.A. is converted into a true statement P.N. from which all inferences are
uncertain.
28. Ans. (a) U.A. is converted to a P.N. to make the premise true, which is the same as the
inference. Therefore, the inference is true.
29. Ans. (b) The false statement is a U.N. So the true would be a P.A. i.e. Some wrong (W) is
acceptable (A). The inference, Nothing A is W, hence has to be totally false.
31. Ans. (d). Neither of the statements would lead to the conclusion. Statement 1 speaks of
smuggling gold as a crime and statement 2 says that customs officials have the power to arrest
smugglers. However from this we cannot conclude that because they have the power to arrest
they have actually done so.
32. Ans. (d). Both statements in the data speak of accords and of them being useless if not
implemented from which we cannot conclude that these accords have been implemented and
that too in good faith. Information given is not enough to reach such a conclusion.
33. Ans. (d) Interaction between the govt. and the people with innumerable spheres doesn‘t lead to
the conclusion that the govt. affects the major spheres of their lives.
34. Ans. (d) All that we are told is that the concept of development is complex. That does not mean
that scientists disagree about the characteristics of development.
35. Ans. (d) From the given data we cannot conclude for certain that the role of Traditional culture
has been ignored while bringing in reforms.
36. Ans. (d) In the data we have no clue about who founded the Pala Dynasty. The statements tell
us that Pala kingdom was widespread and that Dharmapala was the greatest ruler.
37. Ans. (b). With the help of rules we can eliminate only one option i.e. (c).
(a). The rose is not necessarily a useful plant as shown in the diagram below.
N
U R
(b). Roses are found in the nursery and the plants in the nursery are not useless. So rose
plants are not useless.
N
R UL
39. Ans. (c). 256. Options (b) & (d) can be eliminated directly with the help of rules.
(a). The given conclusion is not always true as seen in the diagram below and is hence
uncertain
Fo
Fr
D Fo
(c). Whatever is delicate (D) and fragile (Fr) does not belong to the set of flowers (Fo).
Fr
Fo
D
Fo
Fo
40. Ans. (a). Options (c) & (d) can be eliminated by rules. Option (b) goes against the rule of
Boolean logic. The conclusion states that, Some monkeys are trained animals when it was
already stated that all the monkeys were in the circus and therefore trained.
(a)
C
T
M
41. Ans. (c) From the passage given we cannot be very sure about whether the move by the Tata
group has lead to a debate on the integrity but it is likely that it could trigger off such a debate
so it can be called a far-fetched conclusion.
42. Ans. (b) The passage on the other hand tells us that the corporate houses have been funding
the election expenditures but covertly. So this statement is a contradiction.
43. Ans. (b) The passage says that election expenditure is supported by corporate houses. So this
statement, which says that they are funded by the individuals and the state, is a contradiction.
44. Ans. (c) The passage tells us that the Tata group has set up a trust to fund the political parties
and it is likely that the others would join them to make the group stronger though not
necessarily. Therefore it is far-fetched.
45. Ans. (a) This statement is a direct and logical conclusion of the passage since such a setup
would definitely go a long way in the providing the much needed transparency which we can
infer from the statements in the passage was formerly absent.
47. Ans (a) SRF is in the threat of a takeover can precede the given statement which mentions that
Arun has no option but to increase his stake in his company SRF Ltd. The given statement is
an indication of the reaction Arun has to the threat of a takeover so this statement has to be an
Upstream argument.
48. Ans (c) this is a lateral argument since it is in flow or in continuity with the overall meaning of
the statement given. In fact, it explains why he has no option but to increase the stake.
49. Ans (d) This statement is irrelevant to the given statement. It does not concern us if the
highest management can tackle a takeover threat level or not.
50. Ans (d) Again the statements about why he has no option left are irrelevant to the given
statement.
Exercise 8 (B)
1. Ans. (a). The argument is clearly based on analogy. The conclusion that is drawn makes it clear
that the argument has assumed that it is possible to get an idea of what percent of traffic violations
go unreported on the basis of the information available about petty crimes. Since the argument
does not compare the degree of danger posed by traffic violations and petty crimes, statement II is
not an assumption. The argument does not make it clear if the information about unreported crimes
and traffic violations is confined to just America. So statement III is not an assumption.
2. Ans. (a). The argument clearly assumes that a person who has complied with a small request
will also comply with a larger request inevitably. Option (b) is irrelevant. We do not have clear
information in the argument if the charitable organisations in question provide tax-exemption
benefits. We are also not told how many people are solicited. For all we know these
organisations might target very few potential donors. So option (c) can be eliminated. Option
(d) is rather accurate about how many people would grant the request while the argument does
not state anything specific regarding that. Moreover, the argument hints that the larger request
is made after the small request has been granted. Option (e) is irrelevant.
3. Ans. (e). The first statement in the argument is in the form of ‗If x and y, then z‘. So if z is not
served on Thursday, it would mean that x and y are also not served together. (If not z, then not
x and not y). We can infer from the passage that if the enchilados are not served on Thursday,
then biryani and manchurian were not served together. But its quite possible that one of them
is served on Thursday. So option (a) can be eliminated. There is no information in the passage
to conclude option (b). Option (c) can be eliminated for the same reason given to eliminate
option (a). The argument does not support option (d) either, for the same reasons cited above.
Moreover, the passage does not say conclusively biryani is served on all other days. It merely
talks about a condition when biryani and manchurian are both served together.
4. Ans. (d). Statement I is an assumption because the passage talks about rating the facilities
provided by the college on a scale of 1 to 10 and using the score for making one‘s decision
regarding opting for a college for further education. Since each of the facilities is scored on the
same scale and averaged, statement II is not an assumption. The passage simply talks about
choosing a college for further education and does not provide any hint about whether the
choice is personal or forced by parents/ peer pressure, etc. So statement III is not an
assumption.
6. Ans. (e). The phrase ―In much the same way‖ indicates that most of the public problems do
indeed reach a point at which, as the last statement in the passage says, ―government officials
can do anything about it.‖ So option (a) is an assumption. The argument says that by the time
the government officials react to a problem, it is past the point at which anything can be done
about it. The assumption hence is that political and military leaders (as suggested by the
analogy presented in the first few lines) react too slowly. Option (b) can thus be eliminated, as
it is an assumption. The statement – ―By the time an issue reaches the consciousness of
enough citizens to become a high priority item in the minds of the officials‖ – goes to show that
the assumption is that an issue first grips the attention of the citizens before it catches the
attention of the officials. So option (c) can also be eliminated. The argument dismisses the
government officials spending time on issues that have been already resolved. Hence the
assumption is that they do not hold any significance for resolving the current issues. However,
we do not have any information in the passage about whether the government officials
generally do not do anything to directly influence the course of public policy. We are just told
that problems tend to reach a point when the government officials cannot do anything about it.
7. Ans. (e). The statement defines an effect for the event when Graf wins. We are told nothing
about the effect for the event when Graf does not win. So none of the options can be
concluded.
8. Ans. (e). If Uncle Tony does not sign, we can conclude from the first statement in the passage
that Uncle Bill has not taken over as the Prime Minister. But this is not given as an option.
From the last but one statement we notice that since it is a rewording of ―If Uncle Tony has
signed, then the other two have signed too‖, we cannot conclude what would happen if Uncle
Tony does not sign. So the answer has to be option (e).
9. Ans. (a). The argument claims that it is very difficult to for transnationals to understand and
adapt to the Indian market adding that it is ―radically different‖. So if they were to enter the
market through joint ventures with those companies, who know about the ―radically different‖
market, it would be a profitable move for them. Option (b) cannot be the answer, as this option
does not say whether managers would have an idea about what is the best for the
transnationals and whether they would be at the helm of the company‘s most important
decision-making wheel. The argument already says that the Indian market is a very different
one. So we do not know if slashing the prices would be a winning move. Option (d) is
eliminated for the same reason. Moreover, the argument does not say that the transnationals
primarily target the children for their products. It rather talks about capturing the market in
general. We do not know if the move suggested by option (e) would work in the ―radically
different‖ Indian market.
10. The argument claims that outsourcing would be extremely beneficial to the company by citing
benefits like access to new skills, fresh insights and fewer administrative tasks. Option (e) cites
a negative consequence like inaccurate summing up of the situation, which would mean that
the positives that are cited would take a beating and hence outsourcing is projected as
unfavourable altogether. Option (a) would not be a weakening argument if outsourcing does
usher in benefits like fresh insights, etc. in spite of the higher cost, which the company might
not hesitate to spend. Option (b) is a strengthening argument while we are looking for a
weakening argument. Option (c) just says that the trend will probably not last for long and is
thus not an effective weakening argument Moreover it does not identify any serious
consequence that outsourcing would entail. Option (d) also cannot be a weakening argument
as it does not explain why the presence of an external consultant would ‗under-utilise‘ the
Pankaj Gandhi’s Academy/Logical Reasoning 232
internal resources and does not say that the external consultants would not be able to bring in
the benefits quoted in the argument.
11. Ans. (b). The argument claims that educated people also do not desist from child abuse.
Option (a) would be irrelevant. We are not told if the child abuse is caused by educated
people. Option (b) supports the claim by citing evidence of a brothel employing children where
graduates were found in a raid. Option (c) is irrelevant. Option (d) would also not strengthen
the argument, as it does not say that the lonely NRI wives are educated.
12. Ans. (c). The argument entices the reader into digital products by comparing them with toys
that one cherished in the childhood. The assumption is thus that people like revisiting their
childhood. The argument says nothing about women and option (a) is hence irrelevant. The
argument does not use the word ‗gift‘ literally with reference to the digital products. Hence
option (b) can be eliminated. Option (c) captures the central assumption underlying the
analogy of the passage than option (d).
13. Ans. (c). The author advocates the need for Sonia to interact with the crowd as she might meet
with several important people who would be of great help to her. We are told that leaders li ke
Manmohan Singh, etc. are self-effacing but we are not told f they spend their time in profitless
company. So option (a) can be eliminated. There is lack of evidence in the argument to
support option (b). The same is the case with option (d).
14. Ans. (d). Since the passage does not say that a software technology park should be located
within city‘s limits, option (a) cannot be inferred. The argument does not make it clear whether
coming up with a ―cyber corridor‖ is tantamount to making a remarkable progress in the IT
field. So option (b) can also be ruled out. From the given information alone we cannot infer if
Tidel Park is an extremely promising venture. Moreover this option is a judgement and not
exactly and inference. The argument touts various aspects of the Tidel Park with a heightened
emphasis on its advantage in terms of the location. So we can infer that its proximity to
international and domestic airport is likely to bestow the ‗cyber corridor‘ with an edge over the
others.
15. Ans. (b). The argument claims that educated people are incapable of making crucial decisions
while option (b) says that such people have an extensive knowledge base that helps their
decision-making. Option (a) is rather along the lines of a strengthening argument. Option (c) is
irrelevant. Option (d) is also a strengthening argument.
16. Ans. (c). The argument merely talks about the belief that the modern sense of humanism
induces, which is that human beings are self-sufficient and have no need for God. That does
not necessarily mean that this sense of humanism is incompatible with religious belief. Being
independent does not mean that one has renounced faith in God. The argument says that
Renaissance Humanism instituted a spirit of critical enquiry but does not say that it was
antithetical to faith in God. The spirit of critical enquiry instituted by Renaissance Humanism
would automatically take one towards a renewed learning and understanding of received ideas
with a critical frame of mind. So option (c) can be inferred. Option (d) is irrelevant.
17. Ans. (d). The argument claims that Zee music, in spite of going digital is still unsuccessful in its
attempts to be the leading music channel. Option (d) strengthens the argument by claiming
that Zee has a track record of low viewers and would be unable to compete with its rivals.
Statement I is likely to be a weakening argument if Zee is assumed to be represented by the
others to a certain extent. Statement II is irrelevant and so is statement III.
18. Ans. (c). The argument claims that Zee music would not be successful in spite of going digital
while statement III gives examples of certain other TV channels who have been successful in
this regard.
20. Ans. (d). The argument claims that something must be done to curb the increase in the
temperature contributed by certain production processes and industrial growth, assuming that
such a phenomenon would be undesirable. Option (a) states that such a phenomenon would
be desirable and need not be curbed. Option (b) does not address the central concern of the
argument, which is global warming. It talks about only one factor that causes global warming
and how it cannot be compromised upon. Option (c) talks about another factor that causes
global warming but we are not told clearly in the option that the increase in the carbon-dioxide
levels is caused more by automobiles than by industrial growth. Moreover this option also does
not say that industrial growth does not cause global warming.
Exercise 9 (A)
1. 1 is clearly a judgement as the person is stating his opinion. 2 is a statement of fact. Once we
have figured these two we can directly choose (c) as the correct answer as it is the only one to
contain this combination of judgement followed by fact. 3 is a clear inference that follows the
if…then pattern. Statement 4 can be classified as a judgement due to the use of the
superlative adjectives that are used. Hence the correct answer is (c).
3. 1 is clearly a judgement as it uses the superlative. The second is stated categorically, hence it
is a fact. Now we can eliminate options (a) and (b). 3 is a judgement as it gives a person‘s
particular perspective. Hence the correct answer is (d).
4. The first statement is a fact as it is categorically stated. The second statement is clearly an
inference as it follows the if…then pattern. Hence the correct answer is (c)
5. This is a categorically stated fact. The second is definitely a judgement as it uses the word of
compulsion i.e. must. Hence the correct answer can only be (d).
6. Here the answer is (a) as only statement 1 follows. The second statement does not follow in
the case where the statements are represented as concentric circles.
7. The answer is (d) as both the statements do not follow. This is because we do not have
complete information about the relationship between lovers and wood. The statements can
only be uncertain.
8. Here the answer is (a). If we take lions and foxes to be intersecting circles we can represent
ants, insects and foxes by the same circle. In this case we do not know the relationship
between lions and human being .It may intersect with the universe of lion. In which case the
first statement follows.
9. Here following the same logic as in question 6 we see that only 1 follows, hence the answer is
(a).
11. The given is a UA. The correct statement will thus be a PN for which all inferences are
uncertain, hence the answer is (c).
12. The given statement is a variation, which can be converted into a UA. The correct statement is
thus a PN. Hence the answer is (c).
13. This is a UA. It must be made true. Hence it is a PN for which all inferences are uncertain.
Hence the answer is (c).
15. This statement is a variation of a UA. Hence when converted into a true statement it becomes
a PN. Here the inference is also a variation, which is a form of PN. Hence the inference is true.
Hence the answer is (a).
16. Here the given premise is the UA- His vocabulary is good. The conclusion is a UN. Hence the
missing premise following the rules must be a UN. There is only one UN in the answer options.
Hence the answer is (a).
17. Here the correct answer is (d). This is because both the premise and the conclusion are UA.
Hence we are looking for a UA. Both the answer options (a) and (b) are UA and can directly be
eliminated because they are both the same. (Only x is y is a variation that means all y is x).
Hence we can directly arrive at the answer.
18. The premise is that all doctors save lives, which is a UA. The conclusion is also a UA. Hence
the missing premise must also be a UA. There are three possible options, but we can arrive at
the answer by sheer elimination when we see that (c) is a variation that when reduced to a UA
is the same as (a). Hence the answer is (b). This method saves time and is effective.
19. The given premise is a UA –laws discipline lives. The conclusion is also a UA. Hence the
answer must either be (a) or (b). (b) is eliminated as it is possible that the set obeys occurs
outside the universe of law. Hence the answer is (a).
20. Here the given premise is he reads newspapers, which is a UA. The conclusion is also a UA.
Hence the missing premise must also be a UA. The answers can either be (a) or (b). (a) is
eliminated as it is possible that politically conscious occurs outside the universe of he. Hence
the answer is (b).
21. The use of the word gradually implies that the first inference follows but we cannot come to any
conclusion about the necessity of a world society as the statement does not refer to it at all.
Hence the correct answer is (a).
22. Here both the statements cannot be implied, as the statement does not refer to who Gandhi
was, neither does it say what role the conscience plays. Hence the answer is (d).
23. When the statement says that we need something it implicitly means that we do not have it.
Hence the first statement follows. When the requirement is for a code of justice it directly
implies that justice can be codified. Hence the answer is (c) as both follow.
24. We cannot imply the first statement, as we are given no indication about other farmers. The
second is true as it means that some farmers may not be receptive, as opposed to ours. Hence
the answer is (b).
25. When we say violence in any form the implicit assumption is that violence has several forms.
Hence the first follows. The second has no bearing on the given statement hence the answer is
(a).
235 Pankaj Gandhi’s Academy/Logical Reasoning
26. The given data obviously do not lead to the given conclusion. Even when taken together the
data only means that the border dispute is troublesome for the government but even then all
disputes must have a solution. Thus the answer is (d).
27. The data do not lead to the conclusion when taken in isolation. But when taken together they
mean that the Budha was a great teacher who lived in India. This cannot lead to any statement
about the teachings of Budha. Hence the answer is (d).
28. The two statements in isolation do not lead to the given statement. Even when taken together
they only refer to a general situation between friends, they do not specifically deal with a
mother and her daughter. Hence the correct answer is (d).
29. The statements taken in isolation do not lead to the given conclusion. Even if taken together
they are ineffective as the data only speaks of accidents and the authorities that investigate
them, it does not refer to the causes of accidents at all. Hence the answer is (d).
30. The two pieces of information do not have any bearing on the specific statement given. They
are only generalisations. Hence the answer is (d).
31. Using the rules we can eliminate (a) and (d). Among the other two options we see that (c) does
not follow as there is no relationship defined between poor people and politician at all. Hence
the correct answer is (b).
32. This is an easy question as by simply using the rules we can eliminate all options except (d).
Hence (d) is the correct answer.
34. Using the rules taught in class we could eliminate all other options. The only one that follows
according to the rules is (b).
35. The correct answer according to the rules and the Venn diagrams is option (c). Option (a) also
satisfies the rules but it is not always necessary that the set of doctors and introverts would
intersect. So the given conclusion is uncertain.
36. The first is forceful as it provides a reason and is specific. The second does not provide a valid
reason and it is vague- we does not necessarily refer to Indians. Hence the correct answer is
(a).
37. The first statement is forceful as it provides a reason and it is specific. The second is a
generalisation that does not refer to cow slaughter at all. Hence the correct answer is (a).
38. The first statement is vague – what does these refer to? The second is equally vague. It does
not specify which tests. Hence the answer is (d).
39. The first is forceful as it gives a valid reason and by using the umbrella term of professionals it
encapsulates teachers and doctors. The second does not provide any clear reason and it is
not specific- what is meant by such discrimination? Hence the answer is (a).
40. The first is forceful as it is specific to India and provides a valid reason. The second statement
is also forceful, as it is a blanket statement. Hence the correct answer is (c).
41. Ans. (a). Hillary comes to know about Bill's generosity (If X) then…
Look at statement 3 (then Y) Bill will not give Paula any gift.( this is not present in the answer
options) Look again at the passage. "Bill will not give Paula any gift" is the effect (Y) in
statement no 2 so (notX) will be "Bill does not gift Monica". From that we can get X that is
Answer option (a) - Bill gives Monica a gift.
43. Ans (a) If Shah Rukh Khan (sk) acts in a Govind Nilhani (GN) movie… is not given anywhere
as the cause X. Taking it as - Not Y…from that we know Y would be Sk does not act in a GN
movie, this is given in the last line of the passage. The corresponding X to it is Aamir Khan
does not act… and the Not X will be Answer Option (a) Aamir Khan will act in it too.
44. Ans. (d) Every statement is linked to the other one therefore the answer will be all of the above
45. Ans. (a) If Shantanu decides not to teach deductive reasoning is not given as X anywhere in
the passage. Taking it as Not Y …Y becomes Shantanu will teach deductive reasoning which
is present in two statements in the passage. From the corresponding remainder we get Answer
option (a) Havovi teaches Reading Comprehension but not Inductive Reasoning
46. This statement is probably true, as it is safe to assume that 42% can be taken to mean most.
Hence the answer is (b).
47. This can be probably false as we see that they are at least planning a new survey. They
cannot be completely callous. Hence the answer is (d).
48. We do not have any reason as to why the government has raised the poverty line. Hence the
answer is uncertain (c).
49. This is definitely true as the passage clearly states that the government plans to take a fresh
survey. Hence the answer is (a).
50. This is probably true as the passage only state that this is what officials believe. So we cannot
call it definitely true. Therefore the answer is (b).
Exercise 9 (B)
1. Ans. (b). The argument claims that relying on computers is unsafe just because some teenage
hackers happen to be capable of breaking the security code. The assumption is thus that it is not
possible to have computers where the security codes have not been broken. Since other methods
of record keeping are not talked about as safer in the argument, statement III is not an assumption.
2. Ans. (c). The argument claims that money spent on marriage counselling to prevent family
dissolution would greatly help curtail the problem of teenage violence and abuse assuming
that it is family dissolution that aggravates the problem of teenage violence and abuse. From
the given argument it is also clear that assumption I is also implied because the argument
stresses on conducting marriage-counselling programs to prevent dissolution of marriages with
the hope that such programs are capable of achieving the desired results. Statement III is far
from being an assumption as the argument says nothing about who would be employed as
marriage counsellors.
3. Ans. (c). The argument says that the money which is obtained by way of the ten percent
surcharge during 1 p.m. to 6 a.m. would ultimately go into the hands of the fleet owners who
lease autos to the drivers as they would simply raise the per shift charge for these hours. That
would leave the drivers of autorickshaws with no incentive to work during the hours mentioned
above thus affecting the availability of autorickshaws in those hours. Clearly, the argument
assumes that a majority of the drivers lease their autorickshaws from these fleet owners and
do not have their own autorickshaws. Option (c) attacks this assumption by stating that the
opposite is true. No other option helps in attacking this assumption in the argument.
5. Ans. (e). If Theroux did not win the prize, we can conclude from the second ―If, then‖ statement
given as premises that Naipaul did not receive more votes than McEwan and that Rushdie did
not receive more votes than Seth. (The given premise is in the form of If x or y, then z, for
which a logical conclusion would be ―If not z, then not x and not y‖).
6. Ans. (d). The relevant premise is in the form ―If x and y, then z‖. The question says y occurred
but z did not occur. If z did not occur we can conclude that x did not occur because x and y
occurring together would result in the occurrence of z. Now that z has not occurred it goes to
show that one of them or both of them did not occur. In the case of this particular question it is
x which would not have occurred as we are told clearly that y occurred.
7. Ans. (d). This option would clearly explain why the sales of foreign sauces and noodles have
not come down in spite of the recent changes in foreign exchange regulations and currency
status that have increased the prices of imported items. Option (a) would not explain why the
current increase in the price has not affected the sales of imported noodles and sauces.
Option (b) does not explain why people in India are willing to pay the higher price. Option (c)
also does not explain why the sales have not been affected so far but merely talks about what
is expected in the future. Option (e) addresses the issue at hand more specifically by talking
about what the consumers choose and why, while option (e) does not give a reason for the
preference shown for imported noodles and sauces but talks about only failed efforts to
persuade people against buying these products.
8. Ans. (b). The statement made by the thespian is that most actors are also failed actors, which
implies that few actors are successful in their profession. No other option addresses the
thespian‘s statement even remotely.
9. Ans. (b). From the second ―If, then‖ statement given, we can conclude that if the drums are
beaten, then there is a war between the kingdoms, as the logical conclusion for the given
premise would be ―If not y, then not x‖.
10. Ans. (a). If the information given by option (a) were true, it would mean that there is no
increase in the number of cases of workplace sexual harassment but merely an increase in the
number of reported cases as opposed to unreported cases. Option (b) would not really explain
the paradox as this option could also leave room for the possibility that there is an increase in
the number of cases of workplace sexual harassment. Option (c) cannot explain the paradox at
all. If sexual harassment cases have been on the high priority list, it would mean that they
would not approve of an apparent increase in the number of such cases. Options (d) and (e)
do not address the issue at hand at all.
11. Ans. (b). If the information given by option (b) were true, it would mean that there is no need
for the number of body spray users to dwindle, as they no longer have a detrimental effect on
the ozone layer. Option (a) says that many people are unaware of the harmful effects of body
spray on the ozone layer. So at least the few who know should have stopped using whereas
the passage says that the number has not abated. Option (b) explains the apparent paradox
without leaving any lacunae in reasoning. Option (c) is irrelevant.
13. Ans. (c). The argument assumes that conditions conducive to the success of free secondary
education would be present in the other states too. Option (c) attacks this assumption by
saying that free secondary education would not be made use of and the literacy rate would not
shoot up. Since the argument does not present the idea of free secondary education as an
obligation or compulsion but as a suggestion, option (a) can be eliminated. Option (b) is also a
judgement and does not explain why the program would not result in overall higher literacy.
Option (d) does not say that adults in other states are uneducated nor does it say that people
who are not educated will not be able to appreciate or encourage literacy.
14. Ans. (b). The passage says that privacy and deception can be lost sight of while using web
chat. So the implied message is that one ought to be circumspect about the use of web chat.
The passage says that web chat is a pleasant way of making friends, exchanging information,
etc. but we are not specifically told that it is user-friendly. Option (c) cannot be inferred from
the given passage as nothing is said about other methods of entertainment. Option (d) is a
generalisation and we cannot generalise based on just one example.
15. Ans. (b). The argument claims that allopathic medicines come with troublesome side-effects
while ayurvedic medicines are talked about as very effective. The assumption is that ayurvedic
medicines do not have anything undesirable about them. Option (b) attacks this assumption by
stating the flip side of ayurvedic medicines. Option (a) would not be a weakening argument as
it does not say that allopathic medicines can be taken without a doctor‘s prescription, nor does
it undermine the effectiveness of ayurvedic medicines as such. Option (c) is more of a
strengthening argument. Option (d) is also a strengthening argument.
16. Ans. (d). The argument says that there was shift was a shift in focus due to the financial
feasibility for the existence of the business hinting that the company was short of funds.
Options (a), (b) and (c) explicitly mention the huge amount of money that would be required to
sustain the business and thus the answer is (d) which includes all three.
17. Ans. (c). Option (c) talks about two important factors influencing an actress‘ career that have
been instrumental in making heroines take up novel roles. Option (a) is a mere judgement.
Option (b) does not talk about what kind of roles Hollywood actresses take up and hence
cannot explain the shift. Option (d) does not say that a movie is all about doing away with
stereotyped roles and thus cannot be a supporting argument.
18. Ans. (b). If a bad loser does not take a defect lightly and remembers failure as too strong a
challenge, option (b) suggests that they might put themselves under undue pressure to
perform well which would actually have the opposite effect. Option (a) does not talk about any
serious consequences arising out of fussing or enacting dramatics disproportionately. Option
(c) also does not explain any serious damage arising out of not having any motivation and is
thus not an effective weakening argument. Option (d) does not say that bad losers thwart their
own success and is thus not a weakening argument.
19. Ans. (a). The second group dismisses imaginations as ―fruitless dreaming‖ implying that
nothing useful comes out of it. Option (a) attacks this assumption and talks about how people
work towards a goal that they have in mind. Options (b) and (c) would strengthen the belief
held by the latter half. Option (d) does not explain if anything useful comes out of their being
―ahead of their times‖. Thus it is not as effective a weakening argument as option (a).
Exercise 10-A
1. In this question the correct answer is (c) as both the conclusions directly follow from the
passage. The passage states in the first sentence that plants and animals adapt themselves to
environments while the second statement points to the second conclusion.
2. The correct answer is (c) as the passage speaks of women‘s emancipation, which is the same
as women‘s freedom. The last sentence of the passage clearly states that she has succeeded
because of her patience. Thus both the conclusions follow.
3. The correct answer here is (a). It is implicit that when the passage speaks of the two fold
phases of the business cycle that there are several steps in the development of business.
Thus conclusion 1 follows. However the second statement cannot be concluded from the given
passage, as it is a generalisation. The passage only addresses what happens when a
business develops it does not state that business must develop.
4. The correct answer is (b). Although the given passage states that we must adopt a positive
view we cannot conclude that we are not sympathetic to their demands. Hence this conclusion
does not follow. The second conclusion can be seen as valid based on the first sentence that
talks of a quest. Hence conclusion 2 follows.
5. The correct answer is (c). It is understood that if students are not really educated where real
education involves critical thinking that the first conclusion follows. Similarly one can conclude
that if education means mere memorising that is not to be considered education. Hence both
conclusions follow.
6. The first conclusion clearly follows but in the case where all the sets are represented by
concentric circles it is not possible to say that all harmful things are bomb. Hence the correct
answer is (a).
7. Here it is not possible to conclude that she is virgin as it is possible that the set she may exist
outside the universe of virgin. Similarly it is possible that she exists outside the universe of
flower. Hence both the conclusions do not follow and the answer is (d).
8. In the case where all the sets are represented by concentric circles and the set E is a disjoint
set we see that both the conclusions do not follow. For if all mangoes are ripe it is impossible
to say that some are not ripe. Again if no mango is eatable it is impossible to conclude that
some are not eatable. Hence he answer is (d).
9. Here we can represent the statements one and two with concentric circles. We can have a set
Girls that partially intersects with Boys and we can represent Father as intersecting with any of
the others sets as long as it does not intersect with the set of girls. Hence we cannot conclude
that no player is a father, neither can we conclude that no father is a player. Hence the correct
answer is (d).
10. All the sets can be represented by concentric circles in which case it is possible to conclude
that all women are pigs but it is not possible to conclude that all pigs are women as there may
be some pigs that are not women. Hence the answer is (a).
11. This is uncertain as we are only given information about the set of unskilled, we do not take
the semantic meaning into consideration. Hence the answer is (c).
12. Similar to the previous question this is uncertain, as we do not have information about the set
of employed. Hence the answer is (c).
Pankaj Gandhi’s Academy/Logical Reasoning 240
13. This is a PN. Hence the inference is uncertain. The answer is (c).
14. This statement is also a PN. It can be read as All wishes are not horses or Some wishes are
not horses. Hence according to the rules the inference is uncertain. The answer is (c).
15. Using the rules we know that for a UN this conclusion is false. Hence the answer is (b).
16. Here we see that the first premise and the conclusion are both UA. Hence the missing premise
according to the rules must also be a UA. Hence answer options (c) and (d) can be eliminated,
as they are PA. We cannot eliminate option (b) directly because we have seen that only x is y
is a variation of the UA. We have to choose the answer between the two options (a) and (b).
We see using Venn diagrams that option (b) does not follow as it is possible that the set He
exists outside the set of Sings well. Hence the correct answer is (a).
17. The premise given to us is India is a poor country, which is a UA. The conclusion is also a UA.
Hence the missing premise must also be a UA. Here again options (c) and (d) can be
eliminated, as they are PA. Among option (a) and (b) we can eliminate (a) using Venn
diagrams. As it is possible that India exists outside the universe of the set Agricultural Country.
Hence (b) is correct as it follows from the diagram.
18. The premise that is given – He is imaginative is a UA and the conclusion is also a UA. Hence
we know that the missing premise must also be a UA. Option (c) is clearly eliminated. When
we check option (a) we see that it is possible that He exists outside the universe of write
poems in which case the conclusion he can write poems does not follow. When we check
option (b) we notice that it is possible that He exists outside the universe of Write poems,
hence the conclusion does not follow. Here the correct answer is (d) none of the above.
19. In this question the premise – animals are not rational is a UN and the conclusion is also a UN.
Hence according to the rules the missing premise must be a UA. Hence options (c) and (d) are
eliminated. Among (a) and (b) we see that (a) does not follow as Animals exists within the
universe of experience. Hence we cannot conclude that no animal learns from experience.
When we check using Venn diagrams we see that (b) follows. Hence the correct answer is (b).
20. In this question we are given a premise that is a UA and a conclusion that is a UN. Hence
according to the rules we know that the missing premise should be a UN. Hence the answer
options (a), (c) and (d) are eliminated, as (a) is a PN, (c) is a UA and (d) is also a UA. By
sheer elimination using the rules we see that the correct answer is (b).
21. The first statement is not implied, as it is a generalisation that has nothing to do with the given
statement. The given statement says that hurting someone is tantamount to hurting oneself it
does not suggest that hurting anyone is bad. The second statement can clearly be eliminated,
as we cannot presuppose that this is the reason for the statement. We have to see if it is an
implication of the statement, not whether it is the possible reason for the statement. Hence the
answer is (d) i.e. both the statements are not implied.
22. The first statement cannot be considered an implication, as it is a generalisation that has
nothing to do with the question statement. Ignorance of law cannot be used as an excuse,
does not imply that one should not be ignorant of the law. One can or cannot be ignorant the
statement only deals with the fact that one should not use ignorance as a scapegoat. The
second statement again is simply a generalisation that has no bearing on the given statement.
Hence the answer is (d).
23. We only know that the incidence can be reduced, this does not imply necessarily that the
incidence is high …even if it is low it can still be reduced a little. Hence 1 does not follow. The
statement 2 follows as the very structure of the given statement indicates a connection
between smoking and lung cancer. Hence the answer is (b).
25. It is apparent that when the statement mentions dental infection as a common human disease
there is no denying the fact that it is a disease is a direct implication. 1 follows. The use of the
word still leads to the implication that it is an old disease. Hence both are implied i.e. the
answer is (c).
26. In these questions it is a better idea to focus on the data first and then check to see if the
statement follows. If we do this we clearly see that both the statements taken together lead to
the conclusion – visions imply an element of unreality but the vision of a European peace order
is now a reality. Hence we can conclude that the European peace order is no longer a vision.
Hence the answer is (c).
27. From the given data all we can conclude is that nuclear wars are highly destructive hence they
should not be fought. From this data we cannot conclude that they cannot be won. Hence both
the data do not lead to the conclusion. Hence the answer is (d).
28. We see that both the statements lead to the conclusion. We know that people suffer due to
increase in expenses and we are also told that the increase has been abnormal or steep.
Hence (c) is the correct answer.
29. We are given information about the Food adulteration department and that several samples
are taken for testing. This does not lead to the specific conclusion about the adulteration of
vegetable oil, neither does it mention the role of the department. Hence the answer is (d).
30. Here again the data provided is of a general nature, even if the statements are taken together
we cannot reach a conclusion based on any single individual‘s interest. Hence the answer is
(d).
32. Statement 1 is a judgement – the indicator here is ‗must‘ which signifies compulsion or a
forcing of one‘s personal opinion on someone else. Statement 2 is a universal truth. Statement
3 is a fact as it is a categorical statement. Statement 4 is a proverb but has an implicit cause
and effect pattern.
33. Statement 1 is a fact, we may be tempted to think of it as a judgement due to the common
sense or proverbial nature of the statement but if you remember the priority order this
confusion can be solved. Statement 2 is clearly a fact. Statement 3 is clearly an inference. It
even follows the if…then pattern. Statement 4 is also factual information. Hence the correct
answer option is (d) FFIF
35. The first statement is a proverb and is therefore a judgement. 2 is a categorically stated fact. 3
has an implicit cause effect pattern and 4 is clearly an inference. The correct answer thus is (a)
JFII.
37. (a) UA +UA should lead to UA or PA. But 5 is a UN. So it does not follow. (b) UN+UA should
be either a PN or a UN. 5 is a UN. Hence it may be the answer. (c)UA + UN should be PN /UN.
But 2 is a UA. Hence it does not follow. (d) UN+UN is inconclusive. Hence the correct answer
is (b).
38. (a)PA+ PN is inconclusive. (b) UA+UN gives UN /PN, 1 is a PN. Hence this may be a possible
answer. (c) UN+UA gives UN/PN and 6 is a UN. This may be a possible answer. (d) UA+UA
should be UA/PA. 6 is a UN. This does not follow. We can eliminate among the two possible
answers by using Venn diagrams. (b) does not follow as it is possible that no politician is poor
therefore the conclusion some are not poor is invalid. Hence the correct answer is (c).
39. Here by using the rules we see that the possible answers are (a) and (c). (c) does not follow
when we check with Venn diagrams as Beautiful may occur within the universe of Flowers so
the conclusion is invalid. Hence the correct answer is (a).
40. By using the rules we can only eliminate option (c). (a) can be eliminated as it is possible that
apples and balls though contained in the universe of Round may not intersect. (b) is eliminated
as it is possible that the set of Ball intersects with the set of Apple. Therefore the answer is (d).
41. This is implied in the passage although it is not directly stated hence it is a lateral argument
i.e. (c).
42. This statement has no bearing upon the passage, hence (d).
43. This accords with the gist of the passage and can function as the opening statement. Hence it
is an upstream argument i.e. (a).
44. The passage does not deal with private channels. Hence it is irrelevant (d).
45. We have no clue about the tone of newscasters hence it is irrelevant. (d).
46. The correct answer here is (d) as the inference for Either x or y is If not x then y which is BD.
47. The relevant statement in the entire rigmarole is the last one as the others are not in the form
of propositions. Hence the answer is (d) as it follows the form If not y then x.
48. There are two relevant propositions- the second and the fourth. The answer options reveal that
they are concerned with the disjoint statement. Hence the correct answer is (c). It fits the
pattern of If not x then y.
49. (a) functions as a supporting argument. (b) and (c) are also supporting arguments. It is only
(d), which suggests that there may be some sections in the party that may discourage
investment. Hence the weakening argument is (d).
50. The statements (a) and (c) are not strengthening. They are not very relevant. It is only (b) that
fits the bill as it agrees that the manager cannot always use subtle tactics, he may require an
arbitrary front. Hence the answer is (b).
1. Ans. (a). The argument begins with the line and centres around the fact that technology is
considered wasteful, ―as though only immediate technological application can justify any
scientific endeavour‖. The latter part of the argument also says that scientific research might
be useful someday. So the assumption is that there is a time gap between the scientific
research and the discovery of a practical application for it. Since the argument says nothing
about the cost part, statement II is not an assumption. Though the argument does assume a
time gap it does not hint that the gap would necessarily be one between two generations. The
argument does not conclusively say that it will not be of any use to the people who are
currently alive.
2. Ans. (e). The analogy given in the last part of the argument makes it clear that the argument
compares a new born baby with a new scientific discovery that has no immediately apparent
practical application. No other option suggests the same idea.
3. Ans. (e). The argument says Changu and Mangu also paid their admission fee for the IIM(L)
hinting that they are also last years‘ Mathematics faculty and have gained admission into
IIM(L). Option (a) can be eliminated for this reason. Option (b) is not an implication but is
stated for a fact. Option (c) says that paying your fee at the IIM(L) implies that you have gained
admission into the IIMs in general, which cannot be inferred from the passage. Option (d)
would mean that all those who make it to the IIMs are Mathematics faculty, which is not implied
by the passage. (―All Xs are Ys‖ does not mean that ―All Ys are Xs‖)
4. Ans. (c). From the second hypothetical statement given in the passage we can infer that if
―Muharaam‖ is not a hit, then Tabu does not act in the film, as when Tabu acts in the film,
―Muharaam‖ will be a hit.
5. Ans. (b). Since the author does not suggest any solution for the problem, option (a) can be
eliminated. Since the author uses a simile in the statement - ―They (standing for violence and
lawlessness) are like a river, which..…overflows in every direction..‖ - we can infer that violent
people are comparable to an overflowing river, according to the author. The author does not
attribute a cause to the existence of violence and lawlessness. So option (c) can be eliminated.
Since the author says nothing about or about what would accomplish anything, option (d) can
be eliminated. Since the author does not pass a judgement or suggest corrective measures,
option (e) can also be eliminated.
6. Ans. (c). The author assumes that sun is not the source of light, which he thinks, is there
―anyway‖, implying that there is some other source of light in the daytime. The argument does
not compare the relative usefulness of all things on earth. So the superlative degree used in
option (a) and the idea given are not the assumptions underlying the passage. Since the
argument does not confine the natural sources of light to just the sun and the moon, as
suggested by the definite article used in option (b), this option can be eliminated. The
argument does not say that the sun and the moon are equal in the intensity of the light that
they give us. So option (d) can also be eliminated. The argument is not about giving more light
to be useful, but rather about giving light ‗when it is necessary.‘
7. Ans. (a). The information given in the argument goes to show that the concept of ‗exclusive
stores‘ and ‗destination shopping‘ proved to be a fiasco. So if they were to sell their products in
a multibrand environment, it would be beneficial to the shoe giants. No other option identifies
the basic flaw in the premise, which is exclusivity.
8. Ans. (c). The argument in question talks about selective knowledge leading one astray as it
entails suppressing some knowledge also. Option (c) similarly talks about selective advertising
that suppresses the defects of a product leading the buyer astray. Option (a) talks about
selection but not about suppression of anything. Option (b) talks only about the negative part,
which is the disaster, but says nothing about selection or suppression. Options (d) and (e) are
also eliminated for the same reason.
10. Ans. (b). The argument claims that the dark side of Venus must be exceedingly cold because
one face of it always towards the sun. This claim is ―corroborated‖ by the experiment
conducted by Pettit & Nicholson who found it to be lower than that of the stratosphere. So the
assumption is that any temperature lower than that of the stratosphere could be termed
‗exceedingly cold.‘ The analogy drawn between Mercury and Venus is to posit the possibility
that Venus also keeps one face always towards the sun. The argument does not use the
analogy to say anything about the size or the distance from the sun. So option (a) can be
eliminated. The argument does hint that since the Venus rotates very slowly, the temperature
of its darker side is colder than stratosphere. From this statement we cannot say that the
assumption was that ―If Venus were to rotate faster, the temperature of its darker side would be
higher than that of stratosphere.‖ It is as good as saying that the premise ―If x, then y‖ implies
―If not x, then not y‖. The passage does not give any information to support options (d) and (e).
11. Ans. (e). The passage merely states that there are certain derogatory terms in the English
language for which there are no male equivalents. That, however, does not mean that women
actually are the quintessence of these qualities. So option (a) is eliminated. The passage says
nothing about men and whether they did or not deserve the equivalents of these terms. So
option (b) is also eliminated. Option (c) cannot be inferred either, as these terms might have
come into existence without the male equivalents as a sheer accident and not due to some
deliberate manipulation by male chauvinists.
12. Ans. (b). The argument mentions the various adverse effects caused by improperly designed
chairs and hints at the need for a chair that would not result in back pain. Option (b) talks
about companies switching to ergonomically friendly chairs implying that the facts given in the
argument are likely to be true and hence properly designed chairs become imperative, as
suggested by the argument. Option (a) does not talk about eliminating the use of improperly
designed chairs to solve the problem and hence cannot strengthen the argument. Option (c)
covers only one aspect of the passage and hence it can be eliminated. Moreover, option (b) is
better when compared with option (c). Option (d) also does not cover the impact that it would
have on the working of an industry or the seriousness of the pain itself.
13. Ans. (b). The argument claims that Blazing Blazers lend a depth, elegance and distinctive tone
to the wearer and the shade he chooses. The assumption is thus that any Blazing blazer can
lend depth and elegance to the wearer. The word ―All‖ is used in option (b) would mean the
same as any. The argument implies that all blazing blazers lend depth etc. That does not mean
that all with ‗depth of character‘ wear blazing blazers. Acquiring depth, character and elegance
as claimed by the passage is different from acquiring ‗depth of character‘ as expressed in
option (c).
14. Ans. (d). The argument hints that ‗The Leopard‘ is read more by the people travelling to Sicily
than by the people of Sicily. Which is why the argument quotes the truth of the cliché and says
that it is still one of the most under-read masterpieces of the last century. Option (d) mentions
that few Sicilians have read ‗The Leopard‘ while most of its readers appear to be tourists thus
corroborating the information given in the argument. Since the name does not indicate the
content, option (a) is irrelevant. Option (b) talks about clichés and not about the readers of ‗the
Leopard‘. Hence it is irrelevant. Option (c) would weaken the argument.
16. Ans. (b). The author of the argument expresses concern over the possible downtrend in the
sale of books taking a cue from the recent software slump. Option (b) says that in spite of the
apparent increase in sales of certain books, the overall sales of books are on the decline.
Option (a) talks about suburbs only and is thus not an effective strengthening argument. Since
the argument is about the sales of books and not just book reading options (c) and (d) would
be irrelevant.
17. Ans. (c). The argument claims that the role of families and communities is becoming an
increasingly an important one as people turn to families and communities for support. Option
(c) directly undermines the argument by citing the case of some people who socialise with their
peers at work rather than turn to families for support. That would mean that families and
communities do not play as important a role in the mental health of a person as projected in
the argument. Option (a) is irrelevant. Option (b) talks about India while the argument is not
specific in its reference. The information given seems to pertain to everyone and not just
Indians. Option (d) is more of a strengthening argument.
18. Ans. (c). The last statement in the passage makes it clear that ―we‖ are guilty and therefore we
cannot look their parents in the eye. The rhetorical question makes it clear that the assumption
is that we cannot look them in the eye and mete out justice to them until we do not have a clear
conscience. Option (a) can be ruled out easily as the argument says that we have shaken
hands and not that we are going to shake hands. The argument merely says that the parents
asking for justice, which is not the same as avenging their loss.
19. Ans. (a). The given argument claims that Ray is the embodiment of the romantic style, which is
described as ‗flamboyant‘ and ‗visceral‘. Option (a) says that the romantic style was
predominantly peaceful and unobtrusive with subdued colours suggesting that Ray who used
the other extreme style is not likely to represent the quintessence of romanticism. Option (b)
does not explain why taking too much from Italy makes one unfit to be called a true romantic.
Moreover, the argument does not hint anywhere that Ray borrowed a lot from Italy specifically.
Option (c) misinterprets the term used in the argument. The word is used to stand for
romanticism and not romance. Option (d) talks about just one case where Phatak scored over
Ray. But that does not effectively undermine Ray‘s skill at using the romantic style.
Additionally, this option does not correlate capturing the rural essence of India with being a
quintessential romantic.
20. Ans. (c). The argument puts conversion in a positive light claiming that Stokes‘ conversion has
begun the end of caste-ridden prejudices in the area. If it were true that conversion poses a
sure threat to rigid caste beliefs and people who hold such beliefs, the claim made in the
arguments stands vindicated. Option (a) does not say that Stokes‘ granddaughter has had
extensive experience with people who have fought against caste bias through conversion. Her
extensive experience with people who are religious converts would not help in corroborating
the claim and is thus irrelevant. The argument says that Stokes did not challenge the system
head on and not that he did not pose a threat to the caste politics. So option (b) can be
eliminated. The fact that Stokes did not challenge the caste system head on does not mean
that he is a believer in non-violence. So option (d) is irrelevant too.